Links 3/17: Relinkquishment

During the 1500s, the Caribbean was teeming with Jewish pirates, who named their ships things like Queen Esther and Shield of Abraham and had swashbuckling nicknames like “The Pirate Rabbi”.

This month in bad graphs: Family Inequality on the weird way the New York Times confuses the trend in car accidents, and Stuart Buck on how some of the hype about rising white-working-class mortality comes from graph that exaggerates its point by using two different y-axes.

A Miami lawyer’s pants burst into flames while he was arguing in court. One hopes the judge instructed the jury to ignore the kabbalistic implications.

Tesco finalizes deal to give all unsold food to charity.

Illegal immigration through Mexico is down by almost half since last year. Some of Trump’s crueler policies might be interpretable in the context of trying to scare people out of illegal-immigrating.

Serum BDNF concentrations show strong seasonal variation and correlation with sunlight. Interesting as more evidence that BDNF matches everything we know about depression. So either the BDNF scientists are cooking the books really hard, it’s one of those things which is correlated in some complicated non-causal ways, or it’s a key to the mystery.

Jerry Coyne’s negative review of Cordelia Fine’s new book on the biology of sex/gender. Stuart Ritchie’s negative review. Greg Cochran’s negative review. Positive reviews from PZ Myers (though he possibly admits he gets his science wrong while also criticizing “the humanity” of anyone who points it out?) and of course the New York Times.

A long time ago I hosted an SSC-meetup-ish-thing in the offices of Quixey, a Silicon Valley company with a lot of connections to the rationalist and effective altruist movements. Unfortunately, Quixey is shutting down. There’s something kind of crazy about starting from nothing in 2009, getting valued at $600 million in 2015, and shutting down in 2017, but I guess that’s business. Or something.

Since Sweden etc are some of the happiest and most developed countries in the world, can we just copy their model?

In British naval parlance, ships with around 90 guns were called second-rates; since these were a little bit weaker than the flagships their name became a generic term for anything that was not quite as good as something else.

The Awl: I Talked To Some Trump Voters Too. “Except for roughly 7,200 articles on the subject, there has been scant effort made by the mainstream media to understand the kind of voters who say Trump speaks for them. So I set out on a road trip to the part of America most coastal elites don’t think about, except when they’re reading one of the fourteen daily pieces in the mainstream media where a journalist visits a town most coastal elites don’t think about.”

Related: I said a few months ago that Trump was considering choosing some exciting candidates for the FDA who might be true libertarians and really change things. Needless to say, Trump did not do that. Big Pharma is thrilled; I hope people think long and hard about the significance of an industry deeply relieved that they are not going to be deregulated. Watch for Gottleib to use vaguely libertarian rhetoric while continuing the crony capitalist system, drug prices to continue to rise, and liberals to declare this proves that libertarianism always fails.

Still related: lots of people have compared Trump to Andrew Jackson; what’s surprising is that he seems self-aware about it (or at least his handlers are). See eg Trump Adds Portrait Of President Jackson To Oval Office and Trump To Lay Wreath At Andrew Jackson’s Tomb. Pundits suspect “dog-whistle” for anti-Cherokee sentiment. Just as long as he doesn’t imitate Jackson’s attitude to the Supreme Court.

And related: That Time When Dick Cheney And Donald Rumsfeld Ran A Universal Basic Income Experiment For Nixon.

Leek & Jager: maybe most published scientific findings aren’t false.

Uber self-driving car progress report: humans still need to take control about once per-mile, with little progress made over the past few months.

TheOutgroup.org: visualizations of political polarization by graphing networks of pundits on Twitter. Recommended!

Somehow I stumbled across New-Culture.org, which I’m tempted to mock as the silliest and most contentless hippie thing I have ever seen. After more consideration, I think that I’m in favor. It seems like a rallying-flag to try to create a community, and contentless rallying flags can be a good thing if they attract the right people while preventing a community from being dependent on possibly-falsifiable statements.

Federal research bodies have started a “vast” deregulation of social science research, in the sense where you might no longer need to get the approval of fifty different ethics bodies before giving participants a written survey about how much TV they watch or whatever. As somebody with a couple of IRB horror stories myself this is actually inspiring me to think about doing more research. Kudos to everyone involved.

Tim Pool was the journalist who took up an InfoWars offer to go to Sweden and see for himself whether it’s plagued by migrant crime. Now he’s reported his results including a YouTube video and a Reddit AMA. Interesting since it’s one of the closest things yet to the “adversarial collaboration” model of journalism. Unfortunately, by the time it reached the mainstream media any signal had already been lost: Breitbart reports that he discovered Sweden was very dangerous, while Salon reports that he discovered Sweden was perfectly safe. He himself says that “what I found was interesting and in reality ‘closer to the middle’ in regards to the left/right narrative.”

Related: claims that 70% of French prisoners are Muslim are inaccurate (though the likely real number, 40%-50%, is still about five times their representation in the population).

Prison brutality: guards throw mentally ill inmate in scalding hot shower; leave him there until he dies. No punishment given.

80000 Hours does their research thing to try to identify the world’s most important problems.

A while back I blogged about how the government forced UC Berkeley to take down its library of free public videotaped lectures from its website because having audio was discriminatory against deaf people. Now lbry.io has mirrored them and put them up on their own website.

New startup plans a 150-seat battery-powered electric plane. Article focuses on the global warming impact, which makes me wonder about what I would expect to be electric planes’ big advantage – are they silent?

An unusually beautiful graph showing just how important genetics can be in various life domains.

New law proposal: lay tubes for underground broadband while you’re building roads, so that your city can have broadband later without anyone having to dig anything up. This sounds so sensible that there’s no way it can possibly happen.

Cost disease update: Navy team challenges itself to avoid the usual failure modes of military bureaucracy, designs new ramjet missile in six months with a $900 engine. “They were even able to buy the parts with a credit card, avoiding the time-consuming defense acquisition process.”

If UK countries were matched to US states in proportion to their percent of their respective nations’ population, then Scotland would be Texas, Wales would be Pennsylvania, Northern Ireland would be New Jersey, and England would be the other 47.

Remember how everyone was talking about how Trump must have inspired an anti-Semitic crime wave among his supporters? And remember how some of the incidents were traced to an anti-Trump socialist working at a leftist magazine? Well, the rest of them seem to be the fault of an Israeli Jew who may have a personality-altering brain tumor. The Atlantic has a pretty good postmortem of the whole affair.

How entomologists have become the first line of defense against delusional parasitosis. Warning: lots of creepy bug pictures.

A summary of the arguments for why multigenerational mobility is not as low as Clark thinks. I may be misunderstanding this field, but it seems to me that the randomized lottery-style experiments show there’s not much long-term transmission of wealth through non-genetic means (which makes sense since only one person can get an inheritance). But transmission of wealth through genetic means is heavily dependent on assortative mating, since three generations out your descendants only have an eighth of your genes anyway. I wonder if anyone has looked into whether the places that have been found to have unusually low intergenerational mobility (medieval Venice?) are the ones that have the most assortative mating.

Low-trust society: Russian store owner tries to hand out free bread to the poor, becomes widely suspected of plotting something.

Matt Levine quotes JP Konig on the Somali shilling (h/t Alex Guzey):

Old legitimate 1000 shilling notes and newer counterfeit 1000 notes are worth about 4 U.S. cents each. Both types of shillings are fungible—or, put differently, they are accepted interchangeably in trade, despite the fact that it is easy to tell fakes apart from genuine notes. This is an odd thing for non-Somalis to get our heads around since for most of us, an obvious counterfeit is pretty much worthless. The exchange rate between dollars and Somali shillings is a floating one that is determined by the cost of printing new fake 1000 notes. For instance, if a would-be counterfeiter can find a currency printer, say in Switzerland, that will produce a decent knock off and ship it to Somalia for 2.5 U.S. cents each (which includes the cost of paper and ink), then notes will flood into Somalia until their purchasing power falls from 4 to 2.5 U.S. cents … at which point counterfeiting is no longer profitable and the price level stabilizes.

SJWs in tech hound a top programmer out of the Drupal community for being into BDSM. On the one hand, the Drupal community leader has been hinting that there are aspects of the case he can’t reveal publicly. On the other, I feel like if you are firing someone for something you cannot make public, you should say so, instead of stating a clear reason for firing him and then mumbling about secrets when people tell you that your reason is stupid. Also a nominative determinist aspect: the guy leading the purge is named “Purer”.

Facebook: Capitalist Quotes That Inadvertently Support Communism

A profile of Nathan Robinson and Current Affairs. If you’re not reading them you’re missing out; you can only get about 50% of the material from listening to me yell at the parts I don’t like.

Some people have reevaluated STAR*D data with symptom clustering? And found differences? In the efficacy of antidepressants? Will have to look at this one more closely sometime.

Did you know: Ten-year-old Ayn Rand was best-friends-forever with Vladimir Nabokov’s little sister, and they would meet at Nabokov’s mansion and have adorable ten-year-old-girl political debates with each other.

A Florida company suing Buzzfeed for defamation has filed a response to a motion to dismiss titled “Six Ways Buzzfeed Has Misled The Court (Number Two Will Amaze You). You can read it here.

Marginal Revolution: Last year, 35% of colleges saw international student numbers go up, 26% saw no change, and 39% saw them go down. New York Times publishes this with the headline “Amid Trump Effect Fear, 40% of Colleges See Dip in Foreign Applicants”. This may be even more dishonest than that other NYT headline.

Breitbart claims Venezuelans are now literally using rare Pepes as currency.

Nobody knows why the ancient Romans needed quite so many mysterious dodecahedrons.

The poor woman is just trying to clean the leaves!

This entry was posted in Uncategorized and tagged . Bookmark the permalink.

752 Responses to Links 3/17: Relinkquishment

  1. Markus Ramikin says:

    That article about free bread reads like an example from an Ayn Rand novel.

    • MartMart says:

      After a comment like that, I simply had to read the link.
      First thought: How is it that black bread is more expensive than white bread now? In the 80’s it was 1/3rd the price of white bread. Very strange.

      Second thought: How much of the differences between the makers and the moochers that AR described can be attributed to high/low trust societies? Galts gulch was a small, tight knit, high trust community. Various producers out in the world also knew each other thru some sort of informal network, and appeared to trust each other. Her various moochers appeared to not trust each other at all, even when they were part of close cabals.
      I don’t remember her ever making the case that individualism will result in high trust, she more or less assumed it. I remember Heinlein at least making that argument, but as usual with him, it was more of a nice sounding quip.

      • Besserwisser says:

        I’m assuming black bread is more expensive than white bread now because less people are eating it now. Economies of scale and such, black bread becomes more expensive relative to white bread. Plus, buyers of black bread might perceive it as a healthy alternative, making them willing to pay more so sellers charge more.

        • DBDr says:

          Registered to reply to this comment:

          Historically, dark bread with the bread of the poor. Usually made with Rye flour, sometimes sunflower seed, has general associations with starving pleasantry.

          In the modern era, white wheat flour has become massively cheaper, and the ingredients used to make dark bread (Molasses, rye flour, etc.) have become much more expensive.

          Also, making white bread is stupidly easy, whereas it is possible to fail to make dark bread.

          • Squirrel of Doom says:

            the ingredients used to make dark bread (Molasses, rye flour, etc.) have become much more expensive.

            This is to make a point rather than to be nitpicky:

            I’d be very surprised if those ingredients have not also become a lot cheaper, just not as massively so.

            The point being that we are so vastly better off materially now that it’s hard to even imagine.

      • Alex Zavoluk says:

        Well, one issue she does describe and emphasize is that among moochers and looters, you have to pretend that your choice of policies is driven by the fact that you care about the poor or whatever. So you can never give your actual reasons, but everyone is still doing everything they can to get what they want, so you’re constantly politicking and covering up your actual intentions. Whereas in Galt’s Gulch, if you want something, you either go make it yourself, or you honestly trade with someone else.

        • MartMart says:

          Trying to reconcile that idea with the experience of Scandinavian countries (of which I know very little), there appears the possibility that Soviet economy wasn’t so much ruined by communism but rather that the pogroms of the revolutions damaged the cultural fabric enough to reduce societal trust.
          Does that make me sound like a reactionary?

          • Scott Alexander says:

            My impression is that it was something about communism itself which damaged social trust – they found a really big social trust cliff just over the West German/East German border.

          • Ilya Shpitser says:

            I like cultural determinism (does that make me sound like a reactionary? Oh no!).

            I think what happened in Russia has to do with the culture that was already firmly entrenched by 1917, and whose roots (imo) go quite a bit back to the mongols, Ivan the Terrible, and the sack of Novgorod.

            I think Russia by 1900 was already a deeply culturally screwed up place. The relationship of both the rank and file and the elite to the state and the czar in Russia was incredibly weird by Western European standards.

            If I may make an analogy to a person, Russia is a person that was abused and beated as a child, and grew up into a psychopath that perpetuates the cycle of abuse on others.

          • Alex Zavoluk says:

            Social trust or not, I would expect the dual problems of incentives and innovation to ruin any and all attempts to implement large scale communism.

            edit–I’m not sure what Scandinavia has to do with the issue.

          • MartMart says:

            Scandinavian countries, according to the more liberal commentators on the web at least, are less ayn randian but at the same time have a high trust societies. They also seem to be a non miserable places to live.
            Somehow, contrary to rand, their well developed safety nets do not result in degrading trust.
            Perhaps then, she is wrong entirely, and Soviet misery was caused mostly by degraded trust and less so by economic realities of communism. Post revolution soviet society was/is remarkable low trust. I’m not sure how the trust was prior to the revolution (I suspect also, because revolution). But after the revolution there were a series of pogroms as anyone remotely associated with non soviet regime was rather brutally removed from society, to the extend of declaring any peasant with more than one horse to be an enemy of the people.
            That had to lower the overall trust level, which had to make life more miserable for everyone independently of the economic system in place.

            So, you know, maybe the soviet dream is possible if everyone can just stay friends, or something like that.

          • johnWH says:

            First, I’m kinda new here, but looking through the comment sections the last couple weeks I’ve noticed that Rand seems to be a common representation for libertarian ideas. Which strikes me as a little odd. I don’t think she’s a very good representation of libertarianism — more of a caricature. I think Smith, Hayek, and Friedman are much better exemplars, but whatever.

            Regarding this comparison between the Soviet Union and the Scandinavian countries: I don’t think you’re really comparing apples to apples. The Soviet Union was, at least an attempt at, a centrally planned economy where whole swathes of industry were managed from the top down. They quickly learned how impractical that was and so the Soviet system moved away from that over the decades, but thats still basically what it was. I think Pete Boettke’s research on this subject is probably a good starting point to learne more. The widespread poverty of the USSR really was, it can be shown, directly related to this system of central planning. It also might worth asking: to what extent were the pogroms a necessary result of central planning?

            The Scandinavian countries, though they have large welfare states, are pretty much free market in every other respect. In certain respects, like international free trade, the fraser institute’s index of economic freedom ranks some of the Scandinavian countries ahead of the USA. Perhaps these rankings are wrong; that’s certainly debatable, but I think this image of the Scandinavian countries as some sort of socialist paradise is pretty misleading.

            So maybe the key to attaining high trust is big welfare state + more or less free trade and low business regulation. Though I do think history/culture plays a big role, and in that respect, Russia probably was worse off than a lot of western countries at the time of the Revolution; But I certainly don’t think the soviet system made it any better.

          • rlms says:

            I think Friedman represents libertarianism much more than Rand does (here at least), since she’s dead now.

          • Besserwisser says:

            Rand might simply be more well-known than some of the other examples and more easily criticized precisely because she seems like a caricature.

          • Nancy Lebovitz says:

            Another major difference between the USSR and the Scandinavian countries is that USSR had a centrally planned economy while the Scandinavian countries have market economies with strong safety nets.

          • suntzuanime says:

            My impression is that it was something about communism itself which damaged social trust

            Possibly has something to do with everyone being encouraged to rat each other out to the authorities for thoughtcrime? How does social trust hold up in non-communist totalitarian states?

          • johnWH says:

            I meant Milton, not David, but he’s a good example too.

            Rand’s definitely among the most well-known for sure, but I would hope that people interested in engaging libertarianism charitably would seek out better sources — though I admit I’m being a little picky

          • On the question of socialism and trust, a point I made in Hidden Order and posted, in condensed form, on my blog may be relevant.

          • Fossegrimen says:

            @JohnWH, Nancy:

            Soviet Union had vs Scandinavia has is apples to oranges. The Scandinavian countries were pretty central-plan based from ~1920 to early seventies at least. The majority party in Norway were working to “change the constitution so we can join the Soviet Union” at least through the 40s.
            As late as the early 80s, telephones were “awarded” from the state and you could typically wait a couple years to get one. The Norwegian state is still by far the biggest player in the Norwegian stock market and used to own a lot of businesses outright.

            During the 70s, somebody noticed that the businesses run by the state were doing worse than the ones run by the market and a privatisation drive has been going on ever since, often with the State as a hands-off majority stock owner. After the fall of the Berlin wall, when one noticed how bad things had been in the USSR, the privatisation caught some serious momentum and by 1996, it was mostly done. Rail services are still run by the state.

            The bottom line is that except for the Pogroms, everything about communism should also count for Scandinavia more or less for the entire lifetime of communism, so that a theory that explains USSR and does not include pogroms should also explain Scandinavia up to around 1970-1990.

          • markus says:

            In the swedish case it´s definitly wrong to compare Sweden 1920-1970 to USSR 1920-1970.

            1920 Swedish taxes was 9,7% of GDP and the state didn´t control much of the economy outside of that. During the coming 60 years the state would command a growing share of the economy, both with taxation and by state-owned companies, the latter mainly by expanding into formerly private (but small) fast growing sectors like telecommunications and electricity and in some cases by starting or bying companies in main swedish industries like steel and mines.

            In the eigthies the process began to reverse and the nineties saw significant change and today total tax stands at 44% (GDP) after a all time high of 50% in 1990.

            Recommended reading for anyone interested about the swedish example is “The capitalistic welfare state” by Andreas Bergh. Other than some common ideological roots for the swedish social democratic party and the soviet communist one you will not find any similarities between Sweden and Soviet reagarding politics or economy.

          • Alex Zavoluk says:

            > I think Friedman represents libertarianism much more than Rand does (here at least), since she’s dead now.

            I’m pretty sure “Friedman” was referring to Milton here, who is also dead.

          • Whatever Happened To Anonymous says:

            For whatever it’s worth, I’m from outside the US, and I’ve heard of (Milton) Friedman, way more than Rand outside the internet.

          • Alex Zavoluk says:

            I think there was probably very low social trust in the USSR, but it seems a lot of it was probably the result of other problems, like the secret police arresting anyone and everyone on any suspicion at all, or just because they hadn’t hit their quotas, and encouraging everyone to report their family and friends. And, as in Rand’s stories, everyone has to constantly being demonstrating their loyalty to a state that you may or may not actually agree with.

            Scandinavia, as others mentioned, other than the welfare state, isn’t as socialist as it’s made out to be. They’ve been reducing spending, and freeing up the economy for decades.

          • Matt M says:

            Yeah, the libertarian spin on the nordics is much more “they had free markets and oil, therefore they ended up with a bunch of money and decided to spend it on social welfare programs” rather than “they instituted social welfare programs and that led to them having a bunch of money”

            If you notice that Bill Gates owns a Bentley, you can’t simply conclude that buying a Bentley is a great path to wealth.

          • rlms says:

            Only Norway has a lot of oil.

          • Mark V Anderson says:

            The books I’ve read about the Russian revolution gave me the impression that Bolshevism was able to take over the country to a large extent because of enormous mistrust between the peasants and the nobility. Thus it was very hard for a moderate regime that tried to represent all of society to succeed. The Bolsheviks claimed to represent the peasants and the workers (although it didn’t turn out that way), and said they’d get revenge on the nobility. In a very low trust society like Russia, the people are much more likely to support whoever will put their group on top, rather than a group that purports to be fair to everyone.

            Thus I think Communism is more a result of low trust in Russia rather than the cause. Although 75 years of Communism probably did make it worse.

          • cassander says:

            @Mark V Anderson says:

            . The Bolsheviks claimed to represent the peasants and the workers (although it didn’t turn out that way), and said they’d get revenge on the nobility.

            They didn’t claim that, Lenin came to power with the greatest political slogan in history, “Land, Peace, Bread.” The “land” portion amounted to, more or less, flat out encouraging the peasants murder their landlords and rich neighbors and take their land.

      • Nancy Lebovitz says:

        I like the shopkeeper’s explanation that it’s about a low-kindness culture. I’m not sure that kindness is exactly the same thing as trust, though they’re related.

      • SEE says:

        It wasn’t individualism per se that allowed people to trust each other in Rand’s philosophy, it was rationality. If someone has demonstrated that he rationally pursues his own self-interest, you can expect that he will continue to do so, since doing so is in his own rational self-interest. Therefore you can rely on it in pursuing your own rational self-interest.

        To put it in other terms more common in the rationality community, if everyone has demonstrated that they will cooperate in an Iterated Prisoner’s Dilemma, then the rational course of action is to cooperate with them, which will maintain the equilibrium of everybody cooperating, to everybody’s personal advantage.

        The Randian “looters and moochers” are people who have proven they will defect; they are thus not rational in the Randian sense, because their successes depend on a steady supply of cooperators, while their actions punish cooperators. Similarly, a conspiracy of the defectors cannot trust each other for the simple reason that they’re all proven defectors.

        The Randian way to handle the “looters and moochers” is to defect against them in response — by, for example, leaving for Galt’s Gulch to live in an all-cooperator community. The all-defector society left will then collapse. The error of Dagny Taggert in the setting of Atlas Shrugged is her expectation that the “looters and moochers” (or at least most of them) are educable and will learn to cooperate when facing the consequences of their defection, rather than either irrational or self-destructive such that they will always defect. As long as she has that belief, it is perfectly moral/rational/selfish for her to remain in the world trying to cooperate with them; when she figures out otherwise, it’s time for her to leave, because it is immoral/irrational/altruistic (in the sense of self-sacrificing) to cooperate with defectors.

        • MartMart says:

          So her point is that socialists are necessarily defectors and can’t/won’t cooperate?
          So the only way to ensure cooperation is to be self interested, and attempt at charity will necessarily lead to defections.

          It seems far to simplistic to be correct, but I can’t find a weak spot in the logic, at least right away.

          • Le Maistre Chat says:

            Maybe state charity recipients won’t cooperate, but Christian charity works because the recipients believe God will punish them for defecting?

          • wysinwygymmv says:

            I don’t understand where there is any logic to have a weak spot, there. The conclusion doesn’t follow from the premises, which haven’t been proven in the first place.

          • SEE says:

            No, not exactly.

            Rand doesn’t object to charity per se — there’s a bit she wrote (in “The Ethics of Emergencies”, one of the essays in The Virtue of Selfishness) that explicitly states that it’s perfectly moral to go and help out a neighbor who’s “ill and penniless” and give him food and medicine “if one can afford it (as an act of good will, not duty)” to help him recover. “But this does not mean that one must support him from then on, nor that one must spend one’s life looking for starving men to help.”

            When she objects to “altruism”, she specifically means the ethic placing the welfare of others first, especially as a Kantian categorical imperative. Her position is that the usual (and impossible) standard of altruistic moral codes is that you should always cooperate — even with a known defector — and accordingly such codes serve simply to empower defectors at your own detriment and the detriment of all other non-defectors. Instead, you should be “selfish” — which is not a case of always defecting, but instead always pursuing one’s own interests (which you cannot do successfully without being consistently rational).

            Socialism is then to Rand the institutional economics of “altruism” — a economy organized such that you either must cooperate with other people regardless of whether or not they defect, or join the class of defectors exploiting the system. And accordingly supporters of socialism are either people confused by indoctrination into altruism such that (even though they see it will empower defectors) they believe they must go along because it’s “right”, or people planning to live as defectors who want a steady stream of suckers. And the latter will prosper at the expense of the former.

          • hlynkacg says:

            So her point is that socialists are necessarily defectors

            I don’t think her point is “socialists are necessarily defectors” so much as “socialism rewards defectors” leading to more defection.

    • cassander says:

      I’ve never been fond of Rand’s solutions, but she was always spot on in her diagnosis of the problem.

    • howardtreesong says:

      Further down that page, there’s an interesting article saying that the vast majority of Proton rocket engines are defective and need to be totally reworked.

  2. Besserwisser says:

    Of course mainstream media looks at a study showing sexual dimorphism and argue against it. That’s just the way their bias goes. Case in point, they’re also biased against MRAs.

    • Aapje says:

      The methodology used in the article is rather poor, as Google tailors search results to the user. So it is quite possible that the selection of articles is based on Google playing to the biases of the author.

      Besides tailoring the results, Google uses a pagerank algorithm that favors articles that are linked to more, which probably results in controversial articles getting ranked higher, as I suspect that more people link to them.

      So the methodology is not really suitable to form strong conclusions on media bias.

      • Besserwisser says:

        I don’t know if there is a clear bias for articles which oppose the author’s viewpoint. For all we know, there are actually more anti-MRM articles out there than the article makes out to be. Also, do you really think anti-MRM articles are more controversial than pro-MRM articles?

        But I agree that the methodology isn’t perfect. The sample size is rather low for one thing and it’s really hard to quantify what’s pro, what’s anti and what’s neutral, even though I have the feeling a lot of the articles probably don’t leave those assumptions very questionable.

        • Aapje says:

          I did not claim that the bias would be either way. A preference for ‘hate reading’ could result in bias to anti-MRM articles. A preference for pro-MRM articles could lead to a bias in pro-MRM articles. A preference for both could lead to a bias against neutral articles. Etc. We have no way to tell, which is why the methodology is poor.

  3. luispedro says:

    “””Cost disease update: Navy team challenges itself to avoid the usual failure modes of military bureaucracy, designs new ramjet missile in six months for $900. “They were even able to buy the parts with a credit card, avoiding the time-consuming defense acquisition process.””””

    In any other public sector context, this would be great news. The idea that a small group of people with $900 could build a missile in a few months is not a pure positive, though.

    • 1soru1 says:

      Pretty sure al Qaeda and ISIS already knew to stay away from fixed-price contracts with private sector weapons contractors.

      Wonder how much total tax-payers money could be saved given the argument:

      In some areas, no matter how hard we try, we are not going to be able to establish a free market with multiple sellers competing on price. Or perhaps trying too hard would be an example of big government social engineering. Or maybe we simply don’t wan’t to succeed, because missiles.

      So we are going to do stuff by hiring people how know how to do stuff, and paying them to do that stuff.

    • homunq says:

      Actually, the $900 price is just for one off-the-shelf component, not the entire missile.

    • gbdub says:

      My understanding is that they didn’t actually “develop a missile for $900”. They developed a booster rocket to flight test the missile (ramjets need to be accelerated to high speed in order to work) for $900 using off the shelf parts and a motor used by (or based on one used by) hobby high-power rockets.

      As both a rocketry hobbyist and an aerospace engineer, this interested me greatly and sounds like a lot of fun. My own manager has kicked around the idea of using hobby rockets to boost sub scale models to get aerodynamic data – supersonic wind tunnel testing is insanely expensive.

      • dodrian says:

        It looks to me that they developed both the booster rocket and ramjet missile engine with off the shelf parts and at low cost.

        However, you’re right in that the $900 figure specifically refers to just the model rocket engine, rather than the whole shebang.

        Though, after seeing Scott’s comment on the NYTimes immigration figures I’m now even more skeptical of anything I read in popular news :-/

      • Progressive Reformation says:

        The famous (infamous?) Gerald Bull – the supergun guy – made his name as a rocket designer for the Canadian military-industrial complex (don’t laugh!). His big idea: buy a cannon and shoot models of the rocket out of it into a big warehouse full of paper screens in order to get aerodynamic data. Worked great, apparently, and was a heckuva lot cheaper and simpler than the wind tunnels his bosses wanted to use.

        • Jiro says:

          Just because some plan was cheap and worked well, doesn’t mean it was a good idea. Some plans are riskier than others. And if you look at the result of one instance of the plan, you might not see that the plan only worked because you were lucky enough to avoid running into the risk.

          (Related to this, see CatCube’s post below.)

          Of course I have no way to know that that is true here, but it’s not obviously false, either.

    • bean says:

      What I really want to know is what this ramjet missile is supposed to do. If they’re talking about 3-4 years to the fleet, then it can’t be all that complicated. Also, the Navy has some pretty strict standards about dangerous items it lets aboard ship. I suspect it may be a new target drone, which would be largely immune to such problems. In any case, the $900 seems to have been just for the rocket motor, not the rest of the project.
      Overall, this is interesting, but not a revolution in anything.

    • CatCube says:

      Doing it one-off isn’t actually all that impressive. My organization typically has a three month or so cycle to let a contract–I mean, that once we have the design (which takes a few months to years, depending on the complexity of the project), it takes our contracting office three months to go through all of the bureaucratic nonsense to actually advertise the project, receive the bids, conduct a Source Selection (if necessary) and sign a contract.

      On a project that was an emergency (we had a trash rack fail and it was starting to feed debris into a hydroelectric turbine) we did the design and let a contract in 20 days, because it was a priority for everybody to get it done in 20 days. When it’s being done one-off like that, it’s not terribly difficult to short-circuit everything because there’s usually enough attention to make sure that nobody is playing games or taking kickbacks or something. If you try to do it for every single project, when somebody inevitably lines their pockets due to a lack of oversight everybody screams “How could you short-circuit your processes that prevent this?”

      Of course, this means that we have a bunch of employees spending months slogging through bureaucratic nonsense, and I’m not sure this actually prevents more corruption that it costs, but Congress has decided that this is how we will do things, so that’s how we do them.

  4. The original Mr. X says:

    Low-trust society: Russian store owner tries to hand out free bread to the poor, becomes widely suspected of plotting something.

    Reminds me of the Roman nobleman Spurius Maelius, who during a famine bought up a load of grain and sold it at a low price to the commoners. For this he was accused of plotting to make himself king and murdered by another nobleman.

  5. g says:

    The paragraph about Tim Pool’s trip to Sweden is currently missing a “Breitbart reports that” or similar.

  6. Callum G says:

    Loosely RE: Tim Pool

    Maybe we should give research grants to genuine evidence based adversarial collaboration? It seems a lot of MSM funding is moving towards an online clickbait format and that’s reducing the quality of the discussions around certain issues. This could change if an organisation funded worthy projects and had a quality control/review process. Sure, it would be too slow for the day to day news, but if helped overcome a financial burden for longer form and less biased media that would be a good thing. Plus the stamp of approval would be a drawcard to readers in and of itself.

  7. TheEternallyPerplexed says:

    Who wants to be a millionaire in a low trust society:

    contestants on the Russian version of the show quickly became wary of asking the audience anything, because they’d almost always give the wrong answer. You may assume that this is the result of a government health agenda built around the regular consumption of vodka, but you’re wrong. As it turns out, it’s believed that Russian audiences would intentionally try to sabotage contestants on the show.

    I don’t buy their explanation (unmerited fortune), though.

    • onyomi says:

      Helmut Schoeck would have something to say about this.

    • Error says:

      This sounds like the current state of most of the Internet.

    • mupetblast says:

      Hard to imagine that happening in the U.S.

      I wonder if something as mundane as the phrase “don’t be a hater” is suggestive of Americans’ greater tolerance for inequality and cultural love affair with the rich.

      • onyomi says:

        You call it “tolerance for inequality” and a “love affair with the rich.” We call it not being petty, mean-spirited, and envious.

        I do recall, however, being told by a Russian once that Russians are very suspicious of their own wealthy based on the premise that it’s nearly impossible to get rich in Russia without being corrupt, therefore all rich people are prima facie suspect. This is not true in the US, so Russians theoretically would not be as suspicious of rich Americans. This doesn’t explain why they’d want to sabotage someone getting rich on a gameshow, though.

        • Besserwisser says:

          Who says they didn’t get in the game show via connections?

        • wysinwygymmv says:

          You call it “tolerance for inequality” and a “love affair with the rich.” We call it not being petty, mean-spirited, and envious.

          “Tolerance for inequality” seems like a totally non-judgmental factual explanation — like, wealth inequality is a thing that exists and people who aren’t against it tolerate it. “Love affair with the rich” doesn’t seem like a particularly negative characterization. Neither seems to be a bad faith assumption about the motives of the other person.

          On the other hand, “petty, mean-spirited, and envious” seems obviously very morally judgmental and are obviously bad faith assumptions about the motives of the other person.

          To me, it seems like you’re the one poisoning the well here.

          • Whatever Happened To Anonymous says:

            “Love affaire with the rich” definitely sounds like a negative characterization.

            Almost agreed on “tolerance for inequality”, except that the word tolerance implies that there’s something that’s bad and needs to be, well, tolerated, while I just don’t think there’s anything wrong with it.

          • Spookykou says:

            Even given that I agree with you that both descriptions apply moral judgment(I’m torn on tolerance(Lactose?)), do you think one is more obvious or overt, more likely to ‘poison the well’?

          • wysinwygymmv says:

            @Whatever Happened To Anonymous:

            “Love affaire with the rich” definitely sounds like a negative characterization.

            It’s obviously a tongue-in-cheek way of saying “admiration for rich people.” So you can infer that the speaker believes it’s bad or not so good to admire rich people. But do you think it’s bad to admire rich people? If not, then how is it a negative characterization?

            Almost agreed on “tolerance for inequality”, except that the word tolerance implies that there’s something that’s bad and needs to be, well, tolerated, while I just don’t think there’s anything wrong with it.

            Right, that’s why you tolerate it. The fact that other people don’t is the only reason that anyone would ever use the word “tolerate” in the first place. No one talks about “tolerating vanilla ice cream.”

            Compare to “tolerance for homosexuality.” Does the phrase imply that there’s something bad about homosexuality, or merely that there are people who think there’s something bad about homosexuality? And if the latter, how is the mere use of the phrase a criticism in homosexuality in any way?

            So in this instance, we have Lefty McLefterson saying: “Hey, here’s a weird explanation for why libertarians admire rich people and don’t have a problem with wealth inequality,” to which Righty O’Right responds: “oh yeah, well you’re a petty, mean-spirited, green eyed monster who just wants to tear down everyone more successful than you are!”

            The former is a pretty fair characterization of libertarian views — you explicitly admitted as much! — whereas the latter is a rhetorical ploy to shame people into shutting up.

          • Whatever Happened To Anonymous says:

            It’s obviously a tongue-in-cheek way of saying “admiration for rich people.” So you can infer that the speaker believes it’s bad or not so good to admire rich people. But do you think it’s bad to admire rich people? If not, then how is it a negative characterization?

            I think there’s more to it than that. After all, do you, a presumably lefty person, think there’s anything wrong with admiring rich people?

            So in this instance, we have Lefty McLefterson saying: “Hey, here’s a weird explanation for why libertarians admire rich people and don’t have a problem with wealth inequality,” to which Righty O’Right responds: “oh yeah, well you’re a petty, mean-spirited, green eyed monster who just wants to tear down everyone more successful than you are!”

            But lefty mc-leftperson didn’t say that, they specifically used the term “love affaire” and “tolerance”. If you’re arguing that the response was disproportionate, then sure, I make a point of not telling petty, green eyed leftists how envious they are, because it’s uncivil and frankly a bit crass, but I think the line between “light hearted disapproval” and “putting foward bad-faith assumptions” is not as clear and broad as you make it out to be.

          • The original Mr. X says:

            It’s obviously a tongue-in-cheek way of saying “admiration for rich people.” So you can infer that the speaker believes it’s bad or not so good to admire rich people. But do you think it’s bad to admire rich people? If not, then how is it a negative characterization?

            If I called you a “dirty faggot”, I think it would be fair for you to describe this as a negative characterisation, even if you, personally, don’t have a problem with homosexuality.

          • wysinwygymmv says:

            @Whatever Happened To Anonymous:

            I think there’s more to it than that. After all, do you, a presumably lefty person, think there’s anything wrong with admiring rich people?

            Yes, of course — there are a lot of downsides to admiring rich people. Personally, yes, I think the downsides tend to outweigh the upsides.

            That’s irrelevant to whether the phrase “love affair with the rich” is an insult or accusation of bad faith. It is neither one. It is an accurate characterization phrased in such a way as to convey the speaker’s disagreement.

            I see no problem unless you think mere disagreement is insulting or accusation of bad faith, in which case there’s clearly no hope for you.

            If you want to convince me there’s a problem here, you’re going to have to do a lot more work than just repeat the words and say “c’mooooooon….”

            But lefty mc-leftperson didn’t say that, they specifically used the term “love affaire” and “tolerance”.

            Yes, I clearly paraphrased. Good observation.

            Since you haven’t actually adduced anything about the phrases “love affair” or “tolerance” that seems to me like an insult or a bad faith assumption, you are giving me exactly zero reason to change my mind about this.

            In fact, I went to great length to explain why “tolerance for inequality” can’t possibly be an insult or assumption of bad faith, and yet you still cite it as problematic. Why? What possible phrase could convey the same meaning while being less insulting to people who actually are tolerant of inequality?

            I think the line between “light hearted disapproval” and “putting foward bad-faith assumptions” is not as clear and broad as you make it out to be.

            I don’t think I made it out to be clear or broad in general. I said it’s clear and broad in this instance. But this makes me think that it’s more clear and broad than you personally tend to think, because you seem to want to characterize very tame (while still disapproving) accurate descriptions of common libertarian sentiments as somehow on the same spectrum as ad hominem, unsubstantiated accusations of pettiness, mean-spiritedness, and envy. There is a pretty clear, broad distinction between ad hominem and not ad hominem.

          • wysinwygymmv says:

            @The original Mr X:

            If I called you a “dirty faggot”, I think it would be fair for you to describe this as a negative characterisation, even if you, personally, don’t have a problem with homosexuality.

            Do you seriously think this is even remotely comparable to the situation we’re discussing?

            “Love affair with the rich” is a reasonable characterization of a common libertarian sentiment (admiration for the wealthy) phrased in such a way that it’s clear that the speaker does not agree with the sentiment. But the sentiment itself is not insulting.

            “You dirty faggot” is not a reasonable characterization of a common liberal/non-libertarian/whatever sentiment.

            “Love affair with the rich” is not an obvious personal attack — it’s only maybe a personal attack if you already agree with the speaker’s values.

            “You dirty faggot” is obviously a personal attack and nothing else.

            If accurately characterizing your beliefs while disagreeing is the equivalent to you of calling you a dirty faggot then I don’t know how to help you.

          • Whatever Happened To Anonymous says:

            Yes, of course — there are a lot of downsides to admiring rich people. Personally, yes, I think the downsides tend to outweigh the upsides.

            Wait, realy? which?

            Since you haven’t actually adduced anything about the phrases “love affair”

            Well, to me, it implies the idea of irrational support and subservience to a group different than their own. Basically, by saying libertarians are not actually “all about liberty” rather than just patsies for rich people. I mean, I’m pretty bad at this subtext thing, but people get offended by much less than this.

          • Gobbobobble says:

            accurately characterizing your beliefs

            I love coming by SSC so that left wingers can tell me what I believe in my heart of hearts.

          • mupetblast says:

            As the person who wrote the original post kicking off the discussion, I admit to having a ‘tude in the case of “love affair with the rich” but not “tolerance for inequality.” Pretty clearly there IS alot of tolerance for inequality in the states. It’s something Americans simply don’t much care to combat. Maybe for good reasons, but was just sayin’.

            The ‘tude I displayed was easily one-upped by “petty, envious and mean-spirited.”

            Incidentally it wasn’t a veiled attack on libertarians. Whatever phenomena is at work in the US on this issue goes way beyond the liberty-oriented right.

          • IrishDude says:

            @wysinwygymmv

            “Love affair with the rich” is a reasonable characterization of a common libertarian sentiment (admiration for the wealthy)

            As a libertarian, I don’t have a general admiration for the wealthy. I do have admiration for those who do a great job at making other people better off, and those who have a knack for that often become become wealthy. But some wealthy got there through cronyism or inheritance, and the former I have distaste for and the latter I’m indifferent to.

            When it comes to admiring, it’s more relevant to me why someone is wealthy than that they’re wealthy.

          • Whatever Happened To Anonymous says:

            As the person who wrote the original post kicking off the discussion, I admit to having a ‘tude in the case of “love affair with the rich”

            I knew it!

            but not “tolerance for inequality.” Pretty clearly there IS alot of tolerance for inequality in the states. It’s something Americans simply don’t much care to combat. Maybe for good reasons, but was just sayin’.

            That’s fair. I did say it’s basically correct, it’s just that…well, I don’t see it as tolerance since I really don’t think there’s anything inherently wrong with inequality (nor am I American, for the matter, but like whatever).

            The ‘tude I displayed was easily one-upped by “petty, envious and mean-spirited.”

            As I said before, I agree.

          • onyomi says:

            I was going to address this question in a new OP to avoid any appearance of making it an ad hominem against mupetblast, but since it’s now received so many replies, I’ll say at least some of what I think here.

            Believe it or not, I censored how I originally wanted to phrase it so that it would take something closer to the form of a description of how most Americans feel about this in general, and less of an expression of my own personal moral revulsion toward this sort of thinking (after all, my initial impulse was to assume that Mupetblast thinks Americans have a “love affair with the rich,” which may also not be accurate if he just means that from e.g. a Russian or European perspective, Americans seem to love the rich). If even describing commonly held moral feelings as part of an explanation for cultural differences is called “poisoning the well,” then I’m pretty sure that forecloses discussion of many important issues of the sort Haidt has explored.

            Obviously “love affair with the rich” is an insult: it implies Americans have a weird, worshipful fascination with the wealthy that is harmful and unnatural. But what might look that way to some people Americans might call “admiring, instead of begrudging, success.”

            In isolation one could possibly make a case that “tolerance for inequality” is just a neutral descriptor, but I think that, in context, it is very clearly a negative. After all, it is paired with a related statement which Mupetblast admits was meant snarkily. Consider the following statement: “you big city folk and your love affair with homosexuality and tolerance for single motherhood and abortion.” In this case, how do you think the speaker feels about single motherhood and abortion?

            Moreover, yes, I do have a problem with merely “tolerating” homosexuality, because even out of any particular context, “tolerating” implies that one is somehow being put upon to accept something bad or unpleasant (and yes, I think “accept” is a more neutral term than “tolerate”). If I said, in isolation, for example, that “Swedes have a high tolerance for censorship,” would you think I have a neutral attitude towards censorship? Or does my use of the word “high” (and, indeed, the usually negative “censorship”) imply something about what I think is normative (a lower tolerance for censorship than most Swedes have)?

            Imagine you have two neighbors, Alice and Bob. Recently, Alice’s business has been really taking off and she has started renovating and landscaping and has bought a new car. Meanwhile, your hours just got cut, your basement flooded and needs expensive repairs, and Bob was laid off. Bob comes over and says “I can’t believe how you tolerate that Alice! She’s renovating her kitchen while your basement is in urgent need of repair. That’s some nerve!”

            What is the correct response to Bob? Assuming Alice is an honest businesswoman and has not previously wronged you in some direct way, the American perspective (which I understand not all Americans actually share; I am stereotyping here) is that you should say: “tolerate?? What’s there to tolerate? What business is it of mine? Sure, I’m a little envious that she’s doing so well while I’m having some trouble; I hope one day I can be as successful as Alice and I guess it would be really nice if she were to help me out financially, but since I don’t even know her very well, I can’t really expect it.” After all, what is the alternative to “tolerating” your neighbor’s success? Keying her car? Marching over and demanding she pay for your basement repairs?

            Moreover, if even your successful neighbor doesn’t owe you anything just because she’s more successful, how much less do you have a claim upon the wealth and success of people you’ve never met, living far away from you, just because they are within the same sovereign boundaries? You aren’t “put upon” to “tolerate” their success. Assuming they didn’t do anything corrupt or directly harmful to you in the attainment of that success, it’s really none of your business.

            This doesn’t mean I don’t ever feel envious. Of course I do. I know what that feeling is like to think “ugh, stupid so-and-so, always so freaking hardworking, how does she do it?? My life sure would be easier if I didn’t have to compete with superwoman, “I publish a brilliant new book every year” over here…and worst of all, she’s a nice person! Couldn’t she at least be a little haughty so I could hate here??” But I don’t think this is a good impulse of mine. I think it’s a bad attitude.

            I very much agree with Helmut Schoeck that the “crabs in a bucket” attitude (the story being that crabs in a bucket can’t get out because each time one starts to climb high the others all pull it down) is quite natural to humanity–maybe even the mean to which people revert in the absence of strong cultural pushes to the contrary–and that a culture’s ability to suppress that attitude and, ideally, replace it with one of “let’s get ahead together” or “one day, that will be me” is a key factor in success.

            And people who have read my posts know I have no “love affair with the rich.” Anyone who is rich because of special privileges, intensive lobbying, competition-crushing regulation, I want them knocked off their pedestal by a nimble competitor yesterday. But that doesn’t mean I assume anyone who is successful is corrupt, nor that the successful in my society owe me something just by virtue of their success, nor that they should feel lucky a mob hasn’t yet shown up at their door with torches and pitchforks.

            And this is why I feel a strong moral revulsion in reaction to statements like “the American poor see themselves not as an exploited proletariat but only temporarily embarrassed millionaires.” Because it encapsulates precisely what I see as the most destructive prevalent attitude in the world.

            So if it came off as “poisoning the well,” then I apologize. I am trying to describe my opinion on the matter in as neutral a language possible given the fact that I have very strong moral and emotional reactions to the topic and think that those reactions are, in some sense, the topic (because we’re talking about differing moral intuitions of e.g. Americans and Russians).

          • houseboatonstyxb says:

            @onyomi
            But a following stage does become Zero Sum. Alice offers your landlord a great sum for your house; the landlord accepts — and you’re out. She demolishes the house to plant redwood trees. Her sister buys Bob’s house — etc etc — til neither of you guys can afford to live in that town.

            This is one thing that makes us fear growing inequality.

          • suntzuanime says:

            If you can tolerate inequality, you build more houses. If you can’t, you regulate development out of existence and then complain about greedy landlords.

          • The original Mr. X says:

            @ wysinwygymmv:

            Do you seriously think this is even remotely comparable to the situation we’re discussing?

            “This individual is homosexual” is not an inherently offensive statement, but it can become offensive if phrased in an offensive way.

            “This individual admires wealthy people” is also not an inherently offensive statement, but it too can become offensive if phrased in an offensive way.

            So yes, I do in fact that that the example I gave is more than remotely comparable to what you were saying.

          • houseboatonstyxb says:

            @suntzuanime

            If you can tolerate inequality, you build more houses. If you can’t, you regulate development out of existence and then complain about greedy landlords.

            Zero-Sum vs No-Zero-Sum has a fuzzy borderline, which fluctuates, and different people encounter it at different income levels. Local real estate is Zero-Sum for most of us; as Annie’s and Amelia’s incomes increase and they bid against each other for the local houses, we can see the price of local houses rising.

            I’m avoiding the term ‘gentrification’, which suggests richer people moving in from outside the neighborhood. I’m trying to show a reason why ordinary people see rising inequality as a danger per se.

            Arguments about Alice perhaps building a factory nearby (cough! buy earplugs!), or high-rise condomiums, or irrigating the adjoining desert — or about the greatest good for the greatest number — are not to the point here. I plan to talk about where else the fuzzy borderline shows up, and about scale.

  8. noahmotion says:

    I went to Hebrew school and Sunday school for years as a child, and I heard not word one about Jewish pirates. It was all “learn the language this” and and “Israel is the best that” and “let’s make Hamentaschen or latkes, depending on the time of year”, but not a single mention of Jewish pirates. What a travesty!

    • Nornagest says:

      Did you at least learn about the Khazar Khaganate?

    • BBA says:

      What’ll really blow your mind is how in the ’20s Jews dominated basketball.

    • Mark V Anderson says:

      I went to Hebrew school and Sunday school for years as a child

      What, both? You got me curious. Sunday school normally means Christian education. Were your parents one of each, so you went to both? I never liked Sunday school — I am really glad I didn’t have to learn two religions.

      • AnonYEmous says:

        Sunday school is also a Hebrew thing. Probably because you can’t work on Saturday if you believe – and surprisingly many do.

        Not sure why he differentiated it – I guess he went on another day and Sunday, but still.

  9. Creutzer says:

    The opposition to the motion to dismiss in the Buzzfeed case is even better. It’s entitled “Six Ways Buzzfeed Has Misled The Court (Number Two Will Amaze You) … And a Picture of a Kitten“, the latter of which was, indeed, filed as Exhibit 41.

    • onyomi says:

      These are just jokes, but I do think something like this in a more serious vein might be a good antidote to a common failure mode I see in, among others, Scott Adams:

      Scott Adams says “Smart, well-informed people disagree on nearly all major issues. So being smart and well-informed doesn’t help you grasp reality as much as you would hope. If it did, all of the smart, well-informed people would agree. They don’t.”

      While I agree that tribalism and other cognitive biases can lead to more disagreement than ought to exist, he misses this major point: “major issues” are “major issues” precisely because they are the things smart people disagree on. That is, we may vastly underestimate how much consensus actually exists because people don’t much talk about things everyone agrees on, like the color of the sky, obviously terrible philosophical arguments, etc.

      • Nancy Lebovitz says:

        The sky is blue less than half the time.

      • Murphy says:

        ya, it’s like if there were 100 pairs of countries with long running disagreements over borders.

        Someone comes along with a neat XYZ negotiation strategy for settling that kind of disagreement and 99 of the pairs settle their differences and stop having border spats.

        20 years later someone comes along and says “the Northians and the Southians have been fighting for decades. So as we can see the XYZ negotiation strategy clearly doesn’t work”

        And people will believe it because every evening there’s film of Northian towns being shelled.

        Nobody bothers to have news segments about the 198 countries where people aren’t kicking the shit out of each other.

        It does imply something separate though: any reasonably popular, reasonably long running dispute which has a large fraction of the population the opposing side probably has some kind of fairly solid underpinnings even for the side you hate.

      • Ilya Shpitser says:

        Look, I don’t think there is any reason to take Scott Adams seriously. Scott Adams isn’t playing the same game as you, he’s shilling. He’s not a policy/statistics kind of guy, he’s all about trying to paint pictures in your head, waggling his eyebrows suggestively, etc.

        • mupetblast says:

          “all about trying to paint pictures in your head…”

          Exactly. He is an animator after all.

        • AnonYEmous says:

          Uh huh.

          The guy more or less admits to being a “master persuader” and a hypnotist. He cops to it and somehow people still take him seriously.

        • onyomi says:

          But I think this general notion is fairly widespread, regardless of what you think of Scott Adams. Maybe not explicitly as Adams states it, but in a kind of hand-wringing attitude which makes one feel that, since seemingly no contentious issue ever gets resolved, maybe we should just give up.

      • Peffern says:

        So this is just a weird form of survivorship bias, right?

  10. Freddie deBoer says:

    A profile of Nathan Robinson and Current Affairs.

    Includes bonus affirmation that I am in fact not white nationalist Richard Spencer.

    • Aapje says:

      Let’s take the scientific approach here:

      White skin: both
      Short hair: both
      Studied English: both
      Criticize the left: both
      Hated by lots of people: both
      Favors violence against the other tribe: only Freddie

      So the sciency facts which are totally not cherry picked show that they are very similar, although deBoer is much more extremist.

      • Protagoras says:

        Hmmm. I’m not sold. Is there any way you could include some confusing and irrelevant but impressive looking mathematical formulas? We are trying to be scientific here, after all.

        • Aapje says:

          Euler’s formula:

          e^ix = 1 + ix + (ix)^2/2! + (ix)^3/3! + (ix)^4/4! + (ix)^5/5! + …
          = cos x + i sin x

          So there is a fundamental relationship between the trigonometric functions and the complex exponential function. deBoer is bipolar, so he can be modeled with a wave function. Spencer has a belief system that will probably run out of control if he tries to implement it, so he can be modeled as a complex exponential function.

          Euler’s formula shows that we can describe a combination of sinusoidal functions as a sum of complex exponential functions and thus we can describe deBoer as a sum of Spencers.

          So: similar, but not exactly equal.

          QED.

          • Protagoras says:

            All right, clearly I can’t argue with that.

          • Error says:

            You, sir, are awesome.

          • The original Mr. X says:

            ^ Comment of the week right here.

          • PSJ says:

            How the fuck are there three comments applauding someone for making fun of a major mental disorder??? What happened to this blog’s comment section? Also, Freddie has written against political violence in the past (although his blog seems to have changed, I’m sure someone can find the citation though)

          • Skivverus says:

            How the fuck are there three comments applauding someone for making fun of a major mental disorder???

            …sufficiently sophisticated trolling, something something Poe’s Law?

            One of the problems with widening the Overton Window, perhaps: one has to go farther for sufficiently unbelievable snark.

          • herbert herberson says:

            Also, Freddie has written against political violence in the past

            The last time DeBoer came up here, a solid majority of commenters were super pissed at him because although he wrote at some length and to substantial controversy with his peers about how street violence was pointless and counter-productive, he admitted in the comments that he didn’t actually feel personally bad when Dick Spencer got punched, nor did he really consider it morally wrong.

          • Salem says:

            Freddie has written against political violence in the past

            Kinda. He said he was against political violence by leftists because it’s unlikely to achieve its aims, but also said he would support it if it were likely to succeed. In other words, he’s in favour of political violence – just not right now.

          • Error says:

            How the fuck are there three comments applauding someone for making fun of a major mental disorder

            Because he was creative, hilarious, and entirely inoffensive about it. It reminded me of the convoluted Kabbalistic references from Unsong. I don’t even know who deBoer or Spencer are, but I’d laugh just the same if the same post had been directed at me.

            Given that the supposed target posts here, I predict with ~80% confidence that he’ll find Aapje’s post funny, too.

          • rlms says:

            @Salem
            I think that description (in favour of political violence, just not right now) describes pretty much everyone. Most people can think of some situations where they’d agree with political violence (Nazi Germany is the classic one).

          • Creutzer says:

            How the fuck are there three comments applauding someone for making fun of a major mental disorder???

            I’m pretty sure that the commenters above, like myself, simply don’t read it as making fun of a mental disorder. Sure, the mental disorder features in the joke, but that doesn’t automatically mean it’s the object of ridicule. If this joke has any object of ridicule, it’s Spencer for having a belief system “that will probably run out of control if he tries to implement it, so he can be modeled as a complex exponential function”. But that’s just because something negative is being said about his belief system – not because he’s being mentioned in a joke. Being mentioned in a joke isn’t tantamount to being ridiculed.

          • CatCube says:

            Maybe I’m just exposing a gap in my knowledge, but what is the “major mental disorder” here and who has it?

          • PSJ says:

            It’s not that it was specifically framed as a negative as much as it was called out in a post explicitly attacking him which still seems sufficiently rude for the standards here. I would personally be fairly angry if someone did that to me, and would prevent me from talking openly about my mental issues which is already a problem in today’s culture. As clever as the comment was, it doesn’t seem justified in the context. (for the record, I am bipolar and have no particular idea whether Freddie is or not, but if someone has a depressive mood disorder it’s probably not best to bring it up while being mean to them already)

          • CatCube says:

            Ah. I missed the word “bipolar” in Aapje’s comment, and I was trying to relate the “scientific approach” to mocking people with some disorder that makes them precise to a socially-crippling degree.

          • Salem says:

            @rlms:
            Maybe. But consider the following messages:
            1. Don’t lie.
            2. Honesty is generally good for society.
            3. Don’t lie, because you’ll probably get caught, but if you can get away with it, go right ahead.

            It’s true that (1) probably comes with some implicit exceptions, and it’s probably true that most people who say (2) really mean (3). But it doesn’t follow that (1) collapses into (3). And if you say (3), I’m going to count my spoons.

            There are more reasons to oppose political violence than how it affects your own faction’s chances of success. Like morality, or procedural justice, or any of the other standard reasons deBoer explicitly rejects. Von Stauffenberg he is not. This is a democracy, he’s free to bring about whatever change he likes, if only he could persuade people – which he can’t. He reminds me of nothing so much as the craven “intellectuals” who cheered on the manganello and olio di ricino.

          • Eponymous says:

            Ouch! You just got Eulered.

          • Nancy Lebovitz says:

            This is going to seem off-topic, but it isn’t.

            I’m not fond of Social Justice, but I was reading a very Social Justicey novel called Dust Girl. A biracial young woman who’s slowly dying from breathing the dust during the Dust Bowl is desperately trying to make a go of her dead(?) parents’ hotel. Some rich white people with the boring name of Hopper show up, and it looks as though they will spend enough to make a difference.

            Gurl fgneg rngvat rirelguvat va fvtug. Bs pbhefr gurl qba’g cnl. Gurl ner ernyyl tvnag ybphfgf, naq nofbyhgryl greevslvat.

            Gur anzr vf Ubccre, trg vg?

            That was when I realized I could be bought, and all it took was a really good pun.

          • The original Mr. X says:

            Vg jbhyq or rira orggre vs vg ghearq bhg gung gurl jrer whfg abezny ybphfgf, naq gur gjvfg jnf gung gur tvey va dhrfgvba jnfa’g n uhzna ng nyy, ohg na vafrpg.

          • Aapje says:

            @PSJ

            I don’t think that I made fun of people who are bipolar. AFAIK it is diagnosed by the existence of strong mood swings, so pretty much by definition people who are bipolar have them. I didn’t say any negative about those mood swings in my comment, but merely acknowledged them. Modelling the mood swings as a wave function seems to me to merely be mild observational humor, not ‘morality humor’ that trades on condemnation/dislike (which was the kind of humor that I used for the ‘violence’ comment, which was actual ribbing). But different people have a different sense of humor, some of which is culturally defined, so YMMV.

            The only reservation I had when writing the comment was that as far as I know, deBoer has not talked about it here, but he does write about it on his blog, so it was public information. The last time he talked about it on his blog he said that he is getting proper medical treatment since he got a solid job, so I didn’t think he would be upset with seeing this.

          • Aapje says:

            I just noticed that Freddie has restarted his blog and on it he posted:

            I’ve gotten a lot out of writing online, but it has had downsides, especially concerning people targeting my employment. Online politics, are not good for my mental well-being. As someone with poor impulse control and bipolar disorder, it’s best to limit my political engagement in digital mediums that favor immediacy over thoughtfulness. I also have found much better ways to utilize my political energy in recent months. Since moving to New York I’ve gotten involved in my own union, in a tenant’s union, and in local education politics, along with attending many protests. This has been wonderful for my mood and sense of political purpose. Online politics leave me discouraged and unhappy; offline politics make me hopeful and energized. So I intend to keep my political engagement squarely offline.

            So now I feel bad about my jokes. My apologies to Freddie if it displeased him.

          • wysinwygymmv says:

            @Salem:

            Like morality, or procedural justice, or any of the other standard reasons deBoer explicitly rejects.

            Both morality and procedural justice can be used unjustly and for evil. This is proved by myriad historical examples — the Hindu caste system is an easy one to reach for.

            Acknowledging this and consequently not ruling out political violence does not make someone evil or immoral.

          • herbert herberson says:

            Maybe it’s an unreasonable assumption to think that many of the people complaining about DeBoer and others’ personal feelings towards antifa aren’t pacifists or otherwise in opposition to America’s constant wars. But I’m pretty sure that most of them are not, and its hard not to think of such people as laughably wrapped up in self-(or elite-)serving deontologies.

          • Gobbobobble says:

            @herbert

            One can be opposed to random street violence without being morally required to be opposed to organized, ostensibly regulated, state-level violence.

            Sure, maybe the US hasn’t done a great job lately of following The Rules, but you do see people complaining about, for example, military drone use from time to time.

            One can be opposed to Violence-As-A-Chaotic-Free-For-All without needing to go all the way to Violence-Is-Never-Justified.

          • 1soru1 says:

            One can be opposed to random street violence without being morally required to be opposed to organized, ostensibly regulated, state-level violence.

            But it seems implausible to have such a position without it being fundamentally based on the judgement that one works and one doesn’t.

          • Skivverus says:

            But it seems implausible to have such a position without it being fundamentally based on the judgement that one works and one doesn’t.

            Not necessarily. The judgment might be “organized violence allows for clear ideas of when, where, and who it will happen to (or from), minimizing the number of people who need to watch out for it; chaotic violence does not.”
            No doubt you can expand the definition of “works” further to cover this, but it does not seem implausible to me for “not involving people in (the potential for) violence where possible” to be a goal independent from whatever other goal one was considering using violence to solve.

          • rlms says:

            I think the commonly made distinction is between justified and unjustified violence. Scott had a post where he suggests organisation matters (Be Nice, At Least Until You Can Coordinate Meanness), but I think that is a minority view. In 1943, both formal military action and less organised violence against Germany are acceptable. In 2017, neither is.

          • Gobbobobble says:

            In 1943, both formal military action and less organised violence against Germany are acceptable.

            Well, formal and less organized violence against the German state apparatus, anyway. I don’t think it would be morally justifiable for, say, a concentration camp escapee to walk down the streets of Bielefeld shooting any citizen they came across. Even if they did literally vote for Nazis.

            One of the (again, ostensible) touchstones of organized warfare is that non-combatants don’t get dragged into the fighting. With street fighting, especially of the punch-a-political-opponent variety (I’m told gangs have various apparel that serves the same purpose as uniforms), the distinction is lost.

            The whole point of isolating violence over to people in uniform is so that they can know what they’re getting into and the rest of us can go through life with as little of it as possible. This arrangement has been so successful that things like strategic bombing and terrorism become viable tactics.

          • Nyx says:

            “One can be opposed to random street violence without being morally required to be opposed to organized, ostensibly regulated, state-level violence.”

            Well, I don’t really see a huge distinction. Is it morally superior that the Nazis wore uniforms and had bureaucrats and regulations when murdering Jews, as opposed to the disorganized grassroots pogroms that took place in other times and places? I mean, if tomorrow the United States government arrested Richard Spencer and executed him for fomenting treason against the Constitution, would you give that a big thumbs up because the state has a legitimate right to exercise violence? Probably not. You’re uncomfortable with the legitimization of the punching of Spencer because you don’t see him as a legitimate target of violence, not because you’re quibbling with the manner of that violence.

          • IrishDude says:

            One can be opposed to random street violence without being morally required to be opposed to organized, ostensibly regulated, state-level violence.

            One big downside to state violence is those who decide to use it are making others pay a significant portion of the costs, making the state less careful in what violence they engage in compared to people that fully internalize the cost of their violence.

            I’m less likely to bomb others if I have to pay the full cost of the bomb myself rather than forcing my neighbor to pay for it.

          • Gobbobobble says:

            I mean, if tomorrow the United States government arrested Richard Spencer and executed him for fomenting treason against the Constitution, would you give that a big thumbs up because the state has a legitimate right to exercise violence?

            No, because we live in a system with this funny little thing called an (Ostensibly) Independent Judiciary. It would require a trial and there is definitely not enough evidence to convict him of treason (at least that I’m aware of. If he’s selling nuclear secrets to the Russians, sure, fry the bastard. But prove it first.).

            An extrajudicial killing or a kangaroo court would of course not be considered moral. It would be a perversion of the whole reason the state gets its monopoly on violence and push the needle a little toward whether the executing government needs to be forcibly replaced with one more responsible.

            But that still would in no way justify violence against non-state agents.

          • wysinwygymmv says:

            @Gobbobobble:

            Suppose there is a group on non-state actors A who commit violence against a group on non-state actors B. B tries to press the issue with the state, but the state does nothing about this, so that A has committed violence against B without any consequences. As a result, A continues with violent conduct against B.

            Now, obviously group B is entitled to engage in violence in self-defense, but what about retaliatory violence in this case? B cannot always count on all members being able to effectively defend themselves, but if they can visit retaliatory violence on A so that there is a consequence for A’s violence against B, perhaps that could serve as a deterrent against A-on-B violence.

            Could B ever be justified in such a move in your opinion? Or is the fact that the state is nakedly partisan in allowing group A to commit violence against group B in contravention of the law irrelevant?

          • herbert herberson says:

            I present to the reader: self-(elite-)interested deontology!

          • Jiro says:

            Nazi Germany itself was coordinated meanness.

          • Gobbobobble says:

            Self-defense, of course. Retaliation… eh. Inclined to say no. I’d frankly be more in favor of the aggrieved parties going after the corruption in the justice system, though I’d still prefer it be non-violent if possible (sometimes it isn’t). Escalating a tribal war just makes life worse for Group A, Group B, and Everyone Else.

            This all assumes that we’re going off a definition of violence that involves actual physical harm to persons or property, and not the “speech that hurts my feelings” variety that fringe whackos have been trying to shove the Overton Window towards.

            Also assuming that we’re talking about a country with actual (& popular belief in): rule of law, due process, equal protection under the law, etc. Examples like Rwanda are a different kettle of fish and honestly I haven’t put much thought into how you bootstrap a functioning legal system, so I’m willing to at least entertain the possibility that retaliation could be justified in such cases.

            And as though it even needs saying, Group A is definitely in the wrong. Within the confines of a functioning legal system, Group B retaliating is understandable, but that doesn’t make it morally right.

          • Nyx says:

            > No, because we live in a system with this funny little thing called an (Ostensibly) Independent Judiciary. It would require a trial and there is definitely not enough evidence to convict him of treason (at least that I’m aware of. If he’s selling nuclear secrets to the Russians, sure, fry the bastard. But prove it first.).

            A judiciary that never makes mistakes? Do you really believe that there are no innocent men in prison (or on death row)? Look, I don’t think it’s likely. But there are probably some western countries like Germany that might take a crack at putting him in prison if they could.

            And you didn’t specify “state violence when committed by a state that I like and is committed to civil liberties and an independent judiciary with fair trials.” Just that state violence was “organized” and “ostensibly regulated”, and that made it better than vulgar mob violence. And I would agree that state violence tends to be directed at the right targets and uses the appropriate tactics (in terms of efficacy, precision, accuracy, and so on) more often than street violence: but that doesn’t mean that street violence never targets the right people. Which brings us back to “is Richard Spencer the Right People”.

          • Gobbobobble says:

            A judiciary that never makes mistakes? Do you really believe that there are no innocent men in prison (or on death row)?

            Oh noes… a system that is not perfect… my worldview, it is ruined.

            As long as it is *usually* right, that’s fine. Because there is a whole hell of a lot more opportunity for appeal and redress with a judiciary than with a lynch mob.

            And you didn’t specify “state violence when committed by a state that I like and is committed to civil liberties and an independent judiciary with fair trials.” Just that state violence was “organized” and “ostensibly regulated”, and that made it better than vulgar mob violence.

            You’re taking that out of context. The case where whether you live in a well regulated state matters is Retaliation. One cannot appeal for intervention from the Ministry of Judicially Administered Violence if it does not exist. Random Acts of Violence remain immoral wherever you go.

            And I would agree that state violence tends to be directed at the right targets and uses the appropriate tactics (in terms of efficacy, precision, accuracy, and so on) more often than street violence: but that doesn’t mean that street violence never targets the right people. Which brings us back to “is Richard Spencer the Right People”.

            Richard Spencer happens to live in a country that IS “committed to civil liberties and an independent judiciary with fair trials,” so no, the question is actually “Has Richard Spencer committed a crime?” Or perhaps “Do sufficient numbers of my fellow citizens agree that Richard Spencer has acted in such a way that ought to be a crime?”

            In no way is “Is Richard Spencer the Right People to be targeted for extrajudicial violence” a question any civilized person should ask. We Have Rules.

            People are fallible. Systems, generally less fallible. When a system fails, people have the responsibility to first ask “can the system be improved?” before deciding that it should be torn down completely (sometimes it should be, but you have to seriously consider the question). Because straight-up anarchy will be worse for everyone. You don’t get to defect from the social contract and call it Justice just because it doesn’t always give you what you want.

          • 1soru1 says:

            No doubt you can expand the definition of “works” further to cover this

            In the context of this discussion, we are supposed to be damning some guy for saying violence is bad because it won’t work to reach his goals.

            It’s well known that you can’t do that by simply judging the goals and ignoring the ‘works’; i.e. ‘the End justifies the Means’. But I also I don’t think you can do that by ignoring goals, treating them as some external unrelated to morality. Simply because the effectiveness of tactics varies depending on what your goal is.

            If your goal is ‘become the most notorious murderer in history’, then a tactic of ‘killing women and children’ works. If your goal is ‘a more peaceful and prosperous society’, it presumably doesn’t.

            Personally, I am more suspicious of people (like Spencer, and many western Islamists) who have goals that can’t be achieved by their stated morality than I am of people who genuinely believe their goals can’t be reached by immoral means. The first position is unstable, in that they are likely to change their mind after trying and failing. The second have no reason to try, so no reason to have to update their position.

      • wysinwygymmv says:

        Favors violence against the other tribe: only Freddie

        Link?

  11. maximus says:

    Re: The $900 Navy missile — The engine of the missile costs $900, not the overall project or even the missile itself, which is still incredibly low for a ramjet booster. The saving were primarily because they used a model rocket engine instead of building their own and skipped a few traditional acquisition steps, per the article. A hypersonic prototype was developed in 6 months for an unknown cost, and will likely reach the fleet in 3-4 years.

    As far as trusting myself with regards to surprising news, my initial reaction was something like “Impossible! The parts alone would have to cost well over $900.” Score one for calibration. Still an interesting article that highlights moreso how terrible the normal military acquisition process is.

    • Johannes D says:

      And, of course, I presume the team were paid, and so on…

    • Error says:

      My initial reaction was to misread $900 as “$900 million” and still think that was pretty good.

      I’m not sure what that says about my calibration.

      • maximus says:

        Honestly, still a pretty decent guess. A quick search didn’t find the development cost of the Hellfire missile, but the unit cost today is a bit over $100,000. And that’s about the smallest guided missile in the US inventory. Try to imagine how many they fired to work all the kinks out.

        I don’t know of a military project that’s been done in time or on budget.

        • bean says:

          I can think of a couple (Super Hornet and the Virginia-class both were at least close, the Lewis and Clark-class was so far under budget they were able to order extra ships), but the common denominator is always that they are fairly straightforward developments of existing technology. Pushing out the envelope is going to be expensive.

    • Scott Alexander says:

      Thanks, fixed

    • DBDr says:

      Second comment now.

      I have worked in the military contractor industry in the past; services and equipment.

      Some military contractors are necessary to get the good shit; your Lockheed Martins, your Boeings, your whoever came up with the antenna that fits between the helmet cover and the hard part.

      They guys that run the messhalls, do service and repair, and provide services that at one point were handled by the military until they were privatized?

      Incredibly, insanely, unimaginably inefficient. You have no concept of true privatization until you see a 500 dollar locking washer that clearly came from homedepot.

  12. herbert herberson says:

    People were falling over themselves to accept Watson’s offer to pay for their vacation to Malmo. I’m very skeptical of anything coming from the one person he chose out of that large group.

    • Anonymous Bosch says:

      A sampling of video titles from Mr. Pool’s channel:

      “Is Trump really as bad as they say?”
      “FINDING THE LEAKER IN THE WHITE HOUSE”
      “Trump’s wall might actually work”
      “Hillary’s Health is a SERIOUS Issue”
      “Did Democrats Rig the election?! (Part 2/2)”
      “Clinton Campaign Officially Caught CHEATING”
      “The Oregon Standoff is NOT Terrorism”
      “How many Gamers are Female?”

      There’s one pro-Sanders video in the context of shitting on Hillary / the DNC “establishment,” and the only left-wing issue-based videos are on token social issues like internet privacy and marijuana (i.e., the few issues young alt-righters still overlap with left-liberals).

      Maybe these videos are good or bad, and maybe he’s right or wrong. But pushing him as “left” with no qualifiers seems like a tactic, to imbue any right-wing observations with the extra sheen of admissions against interest.

      Or maybe he used to be left and then moved to the alt-right through various anti-feminism / anti-SJW shibboleths (there’s a video on Ellen Pao being bad!)

      Or maybe he’s still left-wing in a very internal sense but thanks to the alt-right spiking his views he knows his audience wants “Democrats suck” and “Actually, Trump isn’t so bad” (a condition known as Michael Traceyitis).

      In any or all of the above cases, I’d be extra-wary of taking it at face value.

      • herbert herberson says:

        Seems like his leftist credentials are mostly downstream of his involvement in Occupy Wall Street. While Occupy was certainly a left-wing movement (a few scattered Ron Paul people notwithstanding), not everyone who was involved with it stayed that way.

      • He’s a Sanders supporter type. This is what Sanders supporter types are like in my experience. They tend to devote much more energy to attacking the center-left ingroup than the right-wing outgroup. He’s also interested in getting people to distrust the mainstream media (and support independent journalists like him instead), so he’s always looking out for factual errors he can correct, as in the Trump’s wall video, even if it means he ends up arguing on the side of the right. Michael Tracey does that sort of thing too.

        • herbert herberson says:

          That’s true of general Clinton and DLC-type criticism, of poking holes in Russia stuff, and of pointing out DNC collusion during the primaries. But Sanders supporters aren’t worried about finding leakers in Trump’s White House, they’re too busy considering the wall a monstrous waste of money to wonder about whether or not it will “work,” and certainly aren’t Bundy sympathizers.

          Source: am on the left

        • He describes himself as libertarian.

          • lemmycaution415 says:

            Libertarians are not on the left.

          • Matt M says:

            Libertarians are not on the left.

            But a lot of people who describe themselves as libertarians are.

          • wysinwygymmv says:

            But a lot of people who describe themselves as libertarians are.

            Like who? If you name more than two who I’ve ever even heard of I’ll be really surprised.

          • Matt M says:

            I’ll admit I was thinking more of personal friends and acquaintances. I might throw out people like Adam Kokesh and Jeffrey Tucker, who are certainly more left than right leaning on a pretty large number of issues.

          • Sandy says:

            Like who? If you name more than two who I’ve ever even heard of I’ll be really surprised.

            You’ve heard of the Niskanen Center?

          • Glen Raphael says:

            If you name more than two who I’ve ever even heard of I’ll be really surprised.

            Bill Maher has at times described himself as libertarian, though he is obviously of the left and often strongly disagrees with libertarians on particular issues.

            Noam Chomsky has occasionally described himself as libertarian and more consistently calls himself a “libertarian socialist”.

            (FWIW I tend to consider myself both libertarian and “on the left”.)

          • wysinwygymmv says:

            @Glen Raphael:

            Noam Chomsky has occasionally described himself as libertarian and more consistently calls himself a “libertarian socialist”.

            But he uses “libertarian” in the archaic sense of “liberty partisan”. He explicitly rejects the sort of right-wing libertarianism that is now just known as “libertarianism.

            Bill Maher legitimately mixes libertarian and leftist beliefs, so fair enough.

            @everyone else:

            I’ve never heard of those people or groups.

          • Whatever Happened To Anonymous says:

            Like who? If you name more than two who I’ve ever even heard of I’ll be really surprised.

            Scott Alexander?

          • wysinwygymmv says:

            @Whatever Happened to Anonymous:

            Well, I think it’s arguable whether Scott Alexander is either libertarian or on the left, but in the spirit of good sportsmanship I’ll take it.

            So far we have Scott Alexander and Bill Maher. One more and I will be genuinely surprised as I promised.

          • rlms says:

            I think libertarians and left-wingers are two adjacent groups. To the extent that they have an overlap, it is on the periphery of both. So arguing about whether there are people who are both is not particularly useful. People who claim to be both left-wing and libertarian are either non-central examples of both (so neither term tells you much about what they actually believe) or a centralish example of one who for some reason says they are the other as well (despite not being so).

            That said, I think the overlap is small. Most of the examples presented so far are pretty unconvincing: Wikipedia describes Jeffery Tucker as a convert to traditionalist Catholicism, which doesn’t sound very left-wing, and says the Niskanen Center supported Sanders, which doesn’t sound very libertarian.

          • mupetblast says:

            When I was involved in libertarianism way back when, they’d constantly claim that they properly belong on the left (the free marketeers of the French Enlightenment were seated on the left side of parliament), and relatedly, that modern “liberals” stole the term from classical liberals.

            I’m not convinced, seeing as how hundreds of years of developments change the meaning of terms, but I do see the kernel of credibility for thinking of libertarianism as on the left.

            It’s the same libertarian tunnel vision that leads them to think of modern progressives as being basically the same as the progressives of a hundred years ago, or seeing communism and fascism as basically the same because they’re both anti-individualist. (Yes, they are, but that’s not the most salient characteristic of these regimes for most scholars.)

          • The Nybbler says:

            or seeing communism and fascism as basically the same because they’re both anti-individualist. (Yes, they are, but that’s not the most salient characteristic of these regimes for most scholars.)

            Is it the most salient to those living under them?

          • Matt M says:

            traditionalist Catholicism, which doesn’t sound very left-wing

            Have you listened to the pope lately?

          • onyomi says:

            So far we have Scott Alexander and Bill Maher. One more and I will be genuinely surprised as I promised.

            There’s a pretty long list of contributors on this page, which describes itself as “libertarian leftist” and “left market anarchist.”

          • rlms says:

            The Pope is a pretty non-traditionalist Catholic.

      • Or maybe he’s still left-wing in a very internal sense but thanks to the alt-right spiking his views he knows his audience wants “Democrats suck” and “Actually, Trump isn’t so bad” (a condition known as Michael Traceyitis).

        I’d say: this is sort of correct except that his audience is principally other people with Michael Traceyitis, of which a considerable number exist (I know a few IRL), and are still a distinct constituency from the alt-right and think of themselves as left-wingers (although perhaps they’ll end up largely joining it). There were alt-righters watching his videos, but I’d guess they were the ones leaving lots of angry comments about how he was being too sympathetic to the refugees and not portraying Sweden as an unambiguously crime-ridden hellhole.

        • mupetblast says:

          We’ve entered the age of the crowd-moderated (or perhaps mediated?) pundit. Compared to decades past, how you describe yourself is relatively less important than where you appear to be located in the general conversation. This has a lot to do with greater transparency coupled with ideological sophistication; the latter for instance helpful in determining where one is placed by what they DON’T talk about, but could (why focus on combatting Russophobia when you could be detailing Trump’s travesties?).

          Time was when a pundit or public intellectual would be solidly right or left-wing because they told you so, and you’d have little else to go on. But if e.g. Susan Sontag were on Twitter, a self-described identity of hardcore feminist might not fly if we could see she had a ton of followers who utterly adored Camille Paglia. Her reputation as someone safely under the feminist umbrella would be in jeopardy.

          Perhaps this is fodder for a paper dubbed “Political Identity in the Panopticon Age.”

      • Brad says:

        Interesting parallel here to a debate we had a few open posts back about what constitutes a left wing poster. I took the “you will know them by their fruits” position that is analogous to what you seem to be saying about Pool, but there was a lot of pushback.

        • gbdub says:

          “You will know them by their fruits” requires a very fine-tuned fruit detector.

          If your sensitivity is too low, you’re susceptible to concern trolls and people posing as one side just to bolster the credibility of their argument for the other side.

          Having your sensitivity too high though gives you purity spirals, and takes away a valuable means of self correction. Effective internal criticism is a sign of a healthy, mature tribe. If you dismiss too many people saying “hey, I’m sympathetic to this group and have hung out with you for a long time, but this is a real problem…” you’ll miss that you’re losing your constituency.

          That may or may not apply to Pool, I’m not familiar with him.

          This seems to play into the high trust / low trust society thing – in a high trust group, it’s safe to take an internal critic at their word that they ultimately just want what’s best for the group. In low trust world, it’s “shun the non-believer!”

          • Brad says:

            Except that this is a game that lasts over a considerable period of time.

            If someone’s first comment, blog post, op ed, impassioned speech, or whatever is “I’m a leftist, but here’s this thing about leftists that we need to talk about” then yes, you have to decide whether or not you are going to trust them and there are pros and cons to either approach. But if you have hundreds of comments, blog posts, op eds, impassioned speeches, or whatever and 80% of them are “I’m a leftist but here’s this big problem with leftists …” and 0% are “hey don’t you guys just love tax increases on the rich” then I’m not at all worried about skepticism causing purity spirals.

            If you want to claim to be a member of the heavy metal tribe you have to sometimes talk about how awesome heavy metal is. It can’t be all gadflying all the time.

          • Anonymous Bosch says:

            But if you have hundreds of comments, blog posts, op eds, impassioned speeches, or whatever and 80% of them are “I’m a leftist but here’s this big problem with leftists …” and 0% are “hey don’t you guys just love tax increases on the rich” then I’m not at all worried about skepticism causing purity spirals.

            Exactly. I don’t think anything of, say, Chapo Trap House spending a lot of time shitting on the center-left because they speak positively about left-wing issues and when they do talk about the right-wing they talk about it with the same well-practiced disdain, just maybe more curt because a lot of their criticisms are already understood by the audience.

            And I’m neither an alt-righter nor a dirtbag leftist, but I’m very allergic to transparent posturing. There are plenty of people who posture the same way for left-liberals; Bruce Bartlett is an example of playing up the “I’m conservative, I worked for Reagan, but they’re cuh-raaaazy” stuff on every issue all the time such that he’s basically a mainline Democrat.

            When a so-called leftist with a very scant track record of positive Sanders support and a very long track record of Hillary-bashing crosses over into “actually, Trump is right, and if you think he’s wrong, you’re being PC” territory, it sets off my bullshit detector.

          • Cerebral Paul Z. says:

            If someone’s first comment, blog post, op ed, impassioned speech, or whatever is “I’m a leftist, but here’s this thing about leftists that we need to talk about” then yes, you have to decide whether or not you are going to trust them

            What sort of trust is required here? It’s not like you’re sending them money or anything.

          • Salem says:

            Isn’t this the just distinction between identities, ideas and movements?

            It’s obviously possible to be a sincere X-ist in terms of ideas, while disliking X-ism in terms of a movement, for all sorts of X (conservatism, socialism, feminism, environmentalism, Christianity, etc). To give a trivial example, Martin Luther wasn’t trolling Christians by talking about his problems with Christianity as a movement, and he wasn’t a “gadfly.” He was building (what he saw as) a better Christian movement out of one that had got off-track.

            I think the bigger issue is that we have hollowed-out models of political engagement. It’s as if we have a heavy-metal tribe where no-one goes to concerts, but everyone writes fanzines. It’s an unedifying spectacle for the “Modern Metal Is Cool” fanzine writers to accuse the “Modern Metal Is Lame” fanzine writers of not being real metallers, when neither of them goes to the shows. Indeed, if they were going to gigs, or even – whisper it quietly – playing guitar, they might well find that their complaints would evaporate.

          • pdbarnlsey says:

            I’ve always been a bit confused about “concern troll” as it actually applies in practice. The formal definition seems to be something like

            “member of group not-x who argues against a proposed action by group x on the basis that it will be harmful to the interests of group x, while secretly believing that it will actually be helpful to group x”

            That seems like a lot of effort for something that doesn’t sound like a winning strategy.

            In practice are we talking more about sock puppetry where someone comes in and posts “we democrats must band together to stop Hillary for the good of the Democrat party of which I am totally a member”?

          • Nancy Lebovitz says:

            Control trolling may not be a successful strategy, but it’s emotionally tempting.

            You can get in little digs at people while claiming you have their best interests at heart.

            You may actually have their best interests at heart at least in your terms, but you’re still stomping all over their desire for respect.

          • pdbarnlsey says:

            Thanks Nancy, but I’m still not even sure that I’m clean on the definition.

            So, it’s concern trolling if (and only if?) I am someone opposed to the broad goals of, say, the feminist movement, and feminists are planning, I don’t know, to wear pussy hats and I, while believing that pussy hats are likely to advance their goals, say to the feminists I interact with, “I don’t know guys, I think those pussy hats will really harm your movement – they will turn off the moderate republicans”.

            Unless I’m successfully masquerading as a paid-up member of the movement that just seems super unlikely to work. Does it really happen all that much?

            Or is my definition off? I’m genuinely confused/curious, rather than, I guess, concern trolling about concern trolling.

          • Paul Brinkley says:

            @pdbarnsley: Your earlier definition was off, although it might be one implication of the actual definition.

            The term is recent, so its definition isn’t formalized, but it appears to have a consensus meaning – someone who expresses sympathy with group X but concern at a group X position, in order to persuade at least some of group X to reject that position, because the concern troll secretly does not sympathize with group X. Contrived, gratuitous examples: “I’m all for gun rights, but isn’t this Heller decision going a bit too far?” “I normally oppose racial discrimination, but you have to admit, this ethnic group has been responsible for a lot of crime lately.”

            “Concern troll” is an accusation; I know of no case where someone admitted it on the thread they were trolling (except maybe somewhere on 4chan or something). It’s not really provable in an objective way; you can expect group X to strongly agree with the charge, and group not-X to strongly disagree. Although if it’s sufficiently overt, you’ll end up with a soft conviction where said troll simply leaves (and perhaps brags “mission accomplished” to their friends). And in selected cases, accused concern trolls may truly be concerned confederates, although this is also hard to prove. As Brad says, it only works if you have a prior reputation in the forum in question; you have to be Nixon to go to China.

            An “honest” concern troll may secretly believe arguing against a group X position may benefit group X, because the troll believes rejecting that position would be better for everyone, including group X. An “evil” concern troll may believe that position helps group X and is actively trying to undermine them.

            Your later example is pretty square on. Concern trolling can work, say, if the audience isn’t watching out for them, and is easily swayed. A hardened, watchful audience will be largely immune, although they may even see concern trolls where there are none. A rationalist (he says, boastfully) can just evaluate the content of the claim on its merits, and not care whether it’s a troll or not, although there will be some meta-guessing going on based on how earnest the “troll” appears to be, whether the claims are falsifiable, etc.

          • Aapje says:

            IMO, the problem with identifying ‘concern trolling’ is that many people are really bad at identifying the subjective (or even just false) premises on which they build their position, so they assume that their terminal values must result in certain positions.

            Then when someone with different positions claims to have very similar terminal values, they get accused of ‘concern trolling,’ even when the actual cause of the disagreement is different premises.

          • Brad says:

            @Salem

            It’s obviously possible to be a sincere X-ist in terms of ideas, while disliking X-ism in terms of a movement, for all sorts of X (conservatism, socialism, feminism, environmentalism, Christianity, etc). To give a trivial example, Martin Luther wasn’t trolling Christians by talking about his problems with Christianity as a movement, and he wasn’t a “gadfly.” He was building (what he saw as) a better Christian movement out of one that had got off-track.

            But he didn’t, in fact, spend the rest of his life just attacking the Catholic Church and doing nothing else. He wrote a pretty astonishing amount on a great number of subjects. Plenty of these writings were about things that he agreed with the Catholic Church on. That’s what you’d expect because he was a Christian and not just an anti-Catholic Churchian.

            I think the bigger issue is that we have hollowed-out models of political engagement. It’s as if we have a heavy-metal tribe where no-one goes to concerts, but everyone writes fanzines. It’s an unedifying spectacle for the “Modern Metal Is Cool” fanzine writers to accuse the “Modern Metal Is Lame” fanzine writers of not being real metallers, when neither of them goes to the shows. Indeed, if they were going to gigs, or even – whisper it quietly – playing guitar, they might well find that their complaints would evaporate.

            I’m not really sure what you are getting at here. What is supposed to stand in for going to concerts? There’s plenty of policy discussions around, granted not everywhere, but plenty in total. In fact, that’s why the people we are talking about stick out because they never or rarely make concrete left wing policy points but only assert that they are left wing, while doing their supposedly friendly critique thing.

          • Viliam says:

            Seems to me that words “concern trolling” conflate two different things. You know the drill — coin a label for a bad behavior you can convincingly describe, and then apply it strategically to everyone who disagrees with you, regardless of their real motives or actions.

            There may be people saying “don’t do X, it may actually hurt your cause” who actually don’t care at all about your cause; they simply want to stop you from doing X, and appealing to your interests seems like a smart move. Do they actually believe that X could harm your cause? Maybe, maybe not, that is irrelevant here; they just want you to not do X. (Perhaps they believe that doing X would help your cause with probability P, and hurt your cause with probability Q, but the important thing is that you doing X annoys them in both cases.)

            But sometimes people on the same side have similar, but not completely identical values. For example, a person A may say: “I am a loyal member of the Blue tribe, and I hate the Green tribe with a passion, but I am worried that if we launch a nuclear missile at their city, they may launch a nuclear missile at us, and then both cities will turn to wasteland, which is not exactly the glorious Blue future we all imagine.” To which a person B may respond: “No concern trolling allowed here. Enjoy your ban.”

            In this scenario, actually both people A and B may assign a positive value the for victory of Blue tribe over Green tribe, and a negative value for destruction of both city states, but if their values are different, and if they assign different probabilities to Greens also owning nuclear weapons, one of them may sincerely conclude “it’s worth doing” and the other “it’s not worth doing”. It was just more convenient for B to accuse A of secretly not being a loyal Blue, than actually comparing the numbers. Or maybe B actually believed they were surrounded by Green agents pretending to be Blue.

            In other words, seems to me that accusations of “concern trolling” are more about powerful members of the group disagreeing with the minority voicing the concern, than about the actual mechanism that generated the concern. It is their way of saying “your part of this tribe is so powerless these days, that if you won’t shut up, we will simply declare you an outgroup, and nothing of strategic value will be lost”.

          • Douglas Knight says:

            I’ve always understood “concern trolling” to be a form of trolling. The accusation isn’t that they have a secret plan of convincing people of something, but just want to get people to waste their time arguing about something that they had already closed off. (and google agrees with me.)

          • dndnrsn says:

            “Troll” doesn’t really get used very much in its original context – usually when I see it get used nowadays it’s shorthand for “person who is abusive” or even “person who I disagree with”.

          • BBA says:

            This is a general failure mode – if you’ve been dealing with lots of really annoying people dissenting in bad faith, you’ll come to dismiss any dissenter as acting in bad faith, and thus you will brook no dissent at all. This leads to groupthink, yadda yadda yadda, and that’s why Clinton lost.

          • Jiro says:

            To me, Scott’s post about how it’s in the best interests of Trump voters to vote against Trump because voting for Trump helps social justice, came pretty close to that definition of concern trolling.

            Also, that sort of thing, even if it isn’t really concern trolling, can easily be a kind of motivated reasoning. Believing that someone’s support of the other side hurts the other side is a very convenient belief for you to have, after all.

          • AnonYEmous says:

            As usual, disingenuity exists, and there is no good way to root it out. As with virtue signal, so too concern troll. Both phenomena, of course, exist. And there is an inbetween, where you may genuinely believe something because motivated reasoning but it’s probably false.

            Concern trolling is real. I may have even done it before. I try to stay in the in-between, where at least there’s a good chance of taking my advice being a good idea, but it’s a real thing. It sucks if people get mis-accused, but the left has a lot of concern trolls right now – and a lot of things to be very rightly concerned about too, so there’s that.

          • Spookykou says:

            @Jiro

            That is not how I read what Scott was going for there.

            I believe he was speaking particularly to the subset of Trump voters, who were only/primarily voting for Trump in an attempt to hurt Social justice, by arguing that Trump in office might actually help social justice by giving them a real enemy to fight against/legitimizing their grievances.

            It is not clear to me that this prediction was obviously wrong or right at the time, but the reasoning behind it seemed, well, reasonable.

            It is still not clear to me if Scott was wrong.

          • Jiro says:

            but the reasoning behind it seemed, well, reasonable.

            The reasoning behind it was basically “here’s a scenario where voting for Trump helps social justice”. As an argument, that’s pretty terrible; for instance, a blatant omission is that he didn’t argue against, let alone rule out, other scenarios where it hurts social justice. It was basically a just so story.

          • Spookykou says:

            As an argument, that’s pretty terrible; for instance, a blatant omission is that he didn’t argue against, let alone rule out, other scenarios

            He opens with the other scenario.

            Trump is the anti-PC, anti-social-justice candidate. If he wins, he’s going to be the anti-PC, anti-social-justice President.

            Which is also a just-so story.

            This is just-so stories all the way down. My point was that the reasoning behind his just-so story was as good as the reasoning for the just-so story on the other side. If somebody was going to vote for Trump because of the just-so story about how Trump was going to end SJ, then maybe they should consider this other just-so story where that doesn’t happen.

          • AnonYEmous says:

            It is still not clear to me if Scott was wrong.

            Nah, he was pretty obviously wrong, and you see this play out now; the power of social justice comes from their base within the left, and it was adopted more broadly because it seemed like good strategy. And yet a candidate like Trump – who is probably racist, almost certainly sexist, and a privileged white male – lost to an (admittedly white) female in Hillary Clinton. That shows that social justice isn’t a good political strategy for the left – they are currently still emphasizing it, and they may fail to learn their lesson after 4 years, but if they do then it’ll be 8 years, and that will probably seal the deal. Meanwhile, social justice loses its political power, and demonstrates to everyone how few people really believe in social justice rhetoric to begin with.

          • Gobbobobble says:

            @AnonYEmous
            I don’t know, I think that’s ascribing too much meaning to the election. Remember that it shouldn’t change the narrative. As much as I hope you’ll be right about SJWism declining, I think it’s still to early to call.

          • Spookykou says:

            @AnonYEmous

            I am very out of touch, I don’t engage in any social media(well I post some art on my tumblr but I don’t talk to people) and the only news I get is from listening to NPR in the car, so I might have just missed it. I assumed that SJ(Ws) had not suffered any serious blow because the tone of the conversation around SJ had not changed at all on SSC, the only place I ever hear about SJ.

          • AnonYEmous says:

            As to Gobble:

            Remember that it shouldn’t change the narrative.

            Then let’s say rather that the fact that Trump was so close to winning – in fact, so close he won – is very strong proof that social justice is not an effective political strategy. In fact, the specific things that made him so close (and made him win), lower-class whites in usually blue states that voted for Obama, provide even stronger proof. Of course, it’s also worth noting that – even though it shouldn’t change the narrative – most others will perceive it to. And at the end of the day, I’m really just trying to convince others – I already know that social justice is terrible politically and morally.

            As to Spooky: it hasn’t lost its power yet. But eventually it will have to, because it is ineffective. It’s not impossible that I am wrong, mind, but it’s pretty likely that I am right. If they can’t even defeat Trump – or at least, can’t easily defeat him such that luck can’t elect him – even with his sky-high unfavorability ratings, then that’s easy enough for anyone to understand.

          • Nornagest says:

            Remember that it shouldn’t change the narrative.

            Your narrative shouldn’t hinge on whether Trump got 46% or 48% of the vote. But it’s reasonable for him getting >46% to change it, if your narrative would have predicted <20% in early 2016.

            The line between "President" and "credible candidate" is narrow — but lots of people were saying Trump was not a credible candidate.

          • The original Mr. X says:

            @ Jiro:

            The reasoning behind it was basically “here’s a scenario where voting for Trump helps social justice”. As an argument, that’s pretty terrible; for instance, a blatant omission is that he didn’t argue against, let alone rule out, other scenarios where it hurts social justice. It was basically a just so story.

            Plus, a lot of the arguments (e.g., that complaints about patriarchy would be implausible with a female President) would also suggest that electing Barack Obama would have hurt social justice, which pretty obviously wasn’t the case.

          • Jiro says:

            My point was that the reasoning behind his just-so story was as good as the reasoning for the just-so story on the other side.

            I have a much higher prior for “my opponents will try to do X and end up doing X or at least end up not doing much” than I have for “my opponents will try to do X, and actually accomplish ~X”. Pointing out that there isn’t much evidence either way doesn’t really help, since if there’s no evidence, #1 is obviously a more sensible default than #2.

          • Spookykou says:

            @Jiro

            This seems to be making some assumptions that I had not. Namely it seems to imply that Trump actually planned on doing something to actively hinder social justice. My understanding was that both sides of this issue thought Trump would have a passive effect, positive or negative. I know that AnonYEmous has expressed this basic idea ~Trump winning shows that SJ doesn’t work so it will get weaker etc.

            In any case, I was responding to the implication that Scott was not arguing in good faith/was concern trolling, not the legitimacy of the particular argument, which I don’t think is a settled issue. Would you agree that if Scott thought, as I did, that both sides of this issue were passive effects, that he was in fact being fair?

          • Jiro says:

            I can’t read minds. Scott could easily have made the argument honestly. But making it honestly just changes it from concern trolling to motivated reasoning.

            It’s like a pro-lifer telling a pro-choiicer that banning abortions makes it easier for a woman to get an abortion, or a slaveowner telling a slave that obeying his master’s orders is the best way to get revenge on his master.

            If an action, considered on a straightforward level. hurts your opponent’s interests and helps yours, and if you try to tell him that it really hurts his own interests, that is often concern trolling or motivated reasoning.

          • Spookykou says:

            I could agree that for someone with your priors(Trump is going to try and actively shut down social justice, mirrored again in your examples, group A literally makes what group B wants to do illegal) to then try and argue, from a just so story, that the opposite would happen might not be very convincing and could be an example of concern trolling/motivated reasoning.

            My point above was rather that, at least in theory(I also can’t read minds), Scott(and certainly I) did not share your priors. It is possible that I don’t see the Trump-SJ interaction as obvious and as straightforwardly as you do because I am engaged in motivated reasoning. However I would hope you could agree that the long term cultural impact on the zeitgeist of a faction of the left, and it’s interactions with who becomes president of the united states, is not as clear cut an issue as making abortion illegal and access to abortion.

          • Jiro says:

            Trump doesn’t need to actively try to shut down social justice for me to reason like that. He just has to work towards his stated policies; we already know that his stated policies are opposed by social justice, so he’d (on a straightforward level) weaken social justice just by being Trump.

            It’s possible that by some indirect chain of effects his policies will strengthen social justice instead of weakening it, but that isn’t a reasonable default.

          • Spookykou says:

            That is the ‘passive effects’ explanation as I understood it from before, so it seems we have gone in a circle. The only remaining explanation that you have allowed for, is that I am engaged in motivated reasoning, sorry for having wasted your time.

          • Jiro says:

            The point is that if you don’t have much idea what’s going to happen, the default should be that either the straightforward effect, or nothing much at all, will happen. “By doing X he’ll accomplish ~X” must be explicitly justified. If it’s just passive effect versus passive effect, with not much justified on either side, that claim should lose, and in any other context, it would lose.

    • Scott Alexander says:

      I’ve removed the word “leftist” until I can confirm.

  13. random832 says:

    On the mysterious dodecahedrons – is there any consistency to the set of diameters of each hole, or the set of ratios of each pair of opposing holes? This could support the “measuring device” hypothesis.

    EDIT: On further searching, I found A) there is no consistency in the holes and B) there is a hypothesis that they could have been used for knitting features like glove fingers.

  14. The Nybbler says:

    Evidently, the results reported by Clark do not reflect a universal law of social mobility.

    Maybe I’ve just had too much caffeine, but this reads to me as the scientific equivalent of “IN YOUR FACE, CLARK!”

    • Steve Sailer says:

      “I wonder if anyone has looked into whether the places that have been found to have unusually low intergenerational mobility (medieval Venice?) are the ones that have the most assortative mating.”

      In Gregory Clark’s book, India, which has exceptionally assortative mating, has exceptionally low intergenerational income mobility.

  15. baconbacon says:

    I fully expect a link within the next two years along the lines of

    “Navy launches 10 year, $4 billion study to investigate implementation of cost efficiency of programs avoiding bureaucracy”

  16. Cerebral Paul Z. says:

    “Oy, maties.” — (((Long John Silverberg)))

    • Nabil ad Dajjal says:

      Honestly I think just “Silver” would work here.

      Also, Tom Swifties are cancer.

      • Jaskologist says:

        “Andy Warhol produces deep art,” said Tom cancerously.

      • LHN says:

        Long Jon Silver.

        (One enduring mystery for me is that, at least in the circles I travel, “John”-with-an-h is an incontrovertibly unacceptable name for someone who’s Jewish– my wife even had a cousin from a mixed marriage who was John to his dad’s family and Jan to his mom’s to avoid friction– but Paul isn’t especially uncommon.)

  17. homunq says:

    On the Drupal thing… it seems that “if you are firing someone for something you cannot make public, you should say so, instead of stating a clear reason for firing him and then mumbling about secrets when people tell you that your reason is stupid” isn’t really fair. From my superficial reading, the guy said “I was fired for my beliefs”, the person who fired responded “well, kinda” and then later “not exactly but I can’t talk about it”; a clear reason, true or not, was never stated by the firer. I did not look into it deeply so I could easily be wrong here but I still wouldn’t trust Scott’s characterization either.

    • doubleunplussed says:

      The “well kinda” was this:

      It’s not for me to judge the choices anyone makes in their private life or what beliefs they subscribe to. I also don’t take any offense to the role-playing activities or sexual preferences of Larry’s alternative lifestyle.

      What makes this difficult to discuss, is that it is not for me to share any of the confidential information that I’ve received, so I won’t point out the omissions in Larry’s blog post. However, I can tell you that those who have reviewed Larry’s writing, including me, suffered from varying degrees of shock and concern.

      In the end, I fundamentally believe that all people are created equally. This belief has shaped the values that the Drupal project has held since it’s early days. I cannot in good faith support someone who actively promotes a philosophy that is contrary to this. The Gorean philosophy promoted by Larry is based on the principle that women are evolutionarily predisposed to serve men and that the natural order is for men to dominate and lead.

      So whilst it hints at there being more to it, it pretty unambiguously says that it’s about him believing that women ought to be subservient. The charitable explanation is that the “more to it” is that he’s a redpiller or something, and truly thinks women should be subservient in all contexts. The less charitable explanation is that it’s just a gross misunderstanding of BDSM. Even if he is totally sexist in private though, there’s the case to be made that so long as it’s private and he treats his female coworkers with respect, it ought to not affect his employment.

      I’m leaning toward the uncharitable explanation because the post doesn’t say “He’s into this kink stuff and also he’s sexist”, it says “the kink he is into is based on sexism”, which anybody can go out and check, and find to be false.

      • brainiac256 says:

        I didn’t read Larry’s blog but I can tell you from reading other Goreans’ stuff that it can be difficult sometimes to tell the difference between somebody who’s playacting (and aware of it) and somebody who actually believes “male-led female-submissive is the One Twue Way”. So I read this as a case of the Drupal clique misinterpreting the former as the latter (best case scenario).

        • John Schilling says:

          If you’ve got a man and a woman (or several) who all believe Gorean BDSM is the One True Way, it shouldn’t be anyone else’s business, any more than it would be his business that other people are “doing it wrong”. If you’ve got a man who believes it is the One True Way and a woman who doesn’t, either nothing is happening or it is probably the police’s business.

      • BBA says:

        My only exposure to Gor is through MST3K, as a result of which my mental image of Goreans is a bunch of Watney Smiths.

        • Le Maistre Chat says:

          The MSTed Gor movie also featured Jack Palance as a Priest-King. This (right) is what a Priest-King looks like in the books.

          • BBA says:

            Apparently Watney wasn’t even in the book. I guess Norman didn’t have the self-awareness to recognize that most people, upon being dumped into a bizarre fantasy world, would probably just take advantage of the relaxed local mores, not bother questioning anything, and become dupes of the evil overlord du jour. I give the filmmakers props for realism, even if Watney was such an obnoxious shit that I wish Cabot had just killed him in the desert at the beginning.

            Oh, and as Nybbler pointed out in the subreddit, they wasted a chance with a character who spends the first 20% of the movie barking “Cabot! Cabot! Cabot!” by not naming him Ostello.

        • Nornagest says:

          I’ve never read the books myself, but a friend of mine claims to have a copy of Assassins of Gor mounted on the wall as a reminder that anything can get published.

          • Not fair. Norman was quite a good storyteller, and the first few books are pretty good.

            The problem is that he ended up spending too much of each book preaching about his rather odd views instead of leaving them as background and just telling the story.

          • Nancy Lebovitz says:

            I read the first two and a half Gor books or so. I wanted to know whether they were as bad as people said.

            The series started as reasonably good adventure sf, and I’m still kind of wistful that there isn’t something better that includes riding on giant falcons.

            However, it isn’t just that Norman spent too much time preaching about his odd views, it’s that his views were vile.

          • Le Maistre Chat says:

            @Nancy: The series started as reasonably good adventure sf, and I’m still kind of wistful that there isn’t something better that includes riding on giant falcons.

            I’m wistful too, because there was more cool stuff in Tarnsman than that. Like the Spider People, and humans riding dinosaurs. As seen on the Boris Vallejo cover.
            And speaking of the giant falcons.

            As far as his vile views, I used the TVTropes page as a reference for the series before making these comments, as I stopped with #3, and apparently the whole thing with the Priest-Kings suppressing chemistry is part of a critique of industrial society (apparently at some points the obnoxious narrator observes that industrial capitalism has made men and women interchangeable cogs for a machine that’s destroying the ecosystem)… and a critique of capitalism with patriarchal elements is going to pattern-match to fascism.
            As a philosophy professor, I’m sure he got this stuff straight from Nietzsche, and Nietzsche can be pretty vile even if we acknowledge he wasn’t a white supremacist.

          • Deiseach says:

            Yeah, if he’d stuck with characters like Nar and even the Priest-Kings, and just left the whole Pleasure Slave stuff be ‘harem girls’ in the background, they would have been perfectly fine pulp-style SF and enjoyable easy reads for when you just wanted some colour and adventure and excitement and not hard science or tangled moral explorations.

            The first couple of books are okay; I’ve only read one from the 80s (the first one I ever read) and that made me so steamingly mad as a twenty-something year old woman that I couldn’t think of Gor without snarling for a long time. He undercut his own world for the sake of promulgating his particular fantasy/cod-philosophy (I suppose the ego boost of being considered a guru for a set of followers of Gorean mores was irresistible) and the books went badly downhill not alone due to the “what women really want” lecturing but because of the inconsistency and retconning and lack of an overall coherent plot structure and reason for what the Priest-Kings were doing (the alien threat from further out in the solar system wasn’t something I was aware of in the books I did read and I had no idea that was their rationale for keeping Gor in a state of semi-feudal, semi-barbarism with its odd mixture of things like advanced life extension technology and everyone travelling by beast of burden or flying beast).

          • However, it isn’t just that Norman spent too much time preaching about his odd views, it’s that his views were vile.

            Not my reaction. He was portraying a world that doesn’t correspond to the world as I see it, a little closer to the world as people viewed it in classical antiquity, and that was interesting. Being able to see out of the eyes of very different people is one of the things fiction is useful for.

          • Nancy Lebovitz says:

            David Friedman, my reaction to Gor was that the depiction of sex roles was saying that people like me not just should be tortured, but really wanted to be tortured. I don’t know whether you would be so abstract about something that was more about people like you. I don’t think The Iron Dream counts.

            This being said, some fraction of my response was unverbalized fear that Norman’s philosophy would leak into the real world. When it didn’t or at least not to a significant extent, I hoped that I would be less inclined to View With Alarm, but I’m not sure I’ve managed that.

            Le Maistre Chat and Deiseach, isn’t stories built from the good parts of Gor what fan fiction is for? I’m not sure how to search for it, though. My guess is that most Gor ff keeps the non-consensual SMBD.

            Also, I think the desire to go on about one’s ideas is a pretty strong compulsion. Obviously, there are people who vary what they’re saying and update….. and there are people who produce a substantial volume of mere repetition.

        • Cerebral Paul Z. says:

          I got a charge out of reading what an abusive master manipulator Garfield is supposed to be, then scrolling up to the picture of Murray from ‘Flight of the Conchords’.

      • martinw says:

        Not only that, but the first published version of that “well kinda” contained two extra paragraphs which were later removed, as quoted in the article Scott linked:

        Second, I believe someone’s belief system inherently influences their actions, in both explicit and subtle ways, and I’m unwilling to take this risk going forward.

        Third, Larry’s continued representation of the Drupal project could harm the reputation of the project and cause harm to the Drupal ecosystem. Any further participation in a leadership role implies our community is complicit with and/or endorses these views, which we do not.

        So yeah, it’s pretty clear that this was purely about the Gor thing. And that first paragraph is about thoughtcrime in its purest form: it is not enough that your actions are beyond reproach, you are not even allowed to think any impure thoughts.

        • Brad says:

          it is not enough that your actions are beyond reproach, you are not even allowed tothink any impure thoughts.

          Wild exaggeration, as is unfortunately typical on this subject. They aren’t administering lie detector tests or truth serums. His thoughts, pure or impure, were his own until and unless he choose to share them. Then they became communications, not just thoughts.

          • Nabil ad Dajjal says:

            Do you believe in such a thing as a reasonable expectation of privacy?

            Pulp SFi-inspired BDSM necessarily involves at least two people, so yes it’s “communication, not just thoughts.” But it’s not like Garfield had put it up on his LinkedIn profile. This was a case of malicious third parties exposing his private sex life.

            I highly doubt that you don’t do anything online or in the bedroom that you’d prefer your employer didn’t know about. If your standard of privacy is “has never communicated it to another person,” then anyone can freely expose you to your boss without leaving you any room to complain.

          • martinw says:

            Well, that was true for the thoughtcrime in the book that coined the term as well. The evil government didn’t have actual mind-reading technology, so as long as you managed to keep your thoughts wholly inside your own head, you might still be a criminal but you’d be able to get away with it.

            But as soon as you, for example, wrote out your subversive thoughts in a private diary, they’d nab you. Even if you never actually came close to turning any of those thoughts into a forbidden action.

            Back to the present: there’s no indication that Larry ever mistreated anybody in the Drupal project. Nor did he even share his “Gorean” ideas within that community; people only found out about it because he used the same username on the kink forum where he shared those ideas with like-minded people.

            And yet, the idea that his belief system would “inherently influence his actions” was enough to get him banned from a community to which he had made valuable contributions for many years. I don’t consider it an exaggeration to call that thoughtcrime.

          • suntzuanime says:

            “Your thoughts are your own, until they are reflected in any way in your words or actions, including in your private life” is not much of a bone for you to throw freedom of thought, friend.

          • Brad says:

            @Nabil ad Dajjal
            A) I’m reluctant to engage with a sockpuppet, and given the pattern of your replies in this overall links post, I believe it is likely you’ve commented here before despite the unfamiliar nym.

            B) Responding to a post wherein I refute “you are not even allowed to think any impure thoughts” without ever mentioning whether or not you agree it is refuted, but instead switching to “what about a reasonable expectation of privacy” strikes me as bad faith akin to a gish-gallop.

            C) Scott’s links indicate that Dries Buytaert reviewed Larry Garfield’s writings. So your insinuations it is based on former lovers breaching confidences — specifically “necessarily involves at least two people”, “private sex life”, and “in the bedroom” are off base.

          • suntzuanime says:

            His actions [within the community] were beyond reproach, and he was cast out for his impure thoughts [as reflected in his activities in his private life]. If it seems like a wild exaggeration to call this thoughtcrime, you’re probably a totalitarian.

          • Nabil ad Dajjal says:

            Not a sockpuppet as such: I lost my original password a while back but this is Dr Dealgood.

            I was going to mention it in my next top-level comment. Good eye picking out my writing style (?).

            Anyway;

            B) Responding to a post wherein I refute “you are not even allowed to think any impure thoughts” without ever mentioning whether or not you agree it is refuted, but instead switching to “what about a reasonable expectation of privacy” strikes me as bad faith akin to a gish-gallop.

            Imagine that someone says that malaria is a problem because it is decimating Africa. You refute that statement by pointing out that decimation means one in every ten people dying and malaria only kills 0.04% of the African population annually.

            Do you expect anyone to be impressed by your refutation?

            C) Scott’s links indicate that Dries Buytaert reviewed Larry Garfield’s writings. So your insinuations it is based on former lovers breaching confidences — specifically “necessarily involves at least two people”, “private sex life”, and “in the bedroom” are off base.

            I was insinuating that it was due to having been doxxed.

            Garfield never made his Gorean connections public before his firing: that was something exposed by bad actors trying (successfully) to get him kicked out of Drupal.

          • Brad says:

            @Sunzuanime

            “Your thoughts are your own, until they are reflected in any way in your words or actions, including in your private life” is not much of a bone for you to throw freedom of thought, friend.

            I think those of you that advocate that no one ever be allowed to take anything anyone else says seriously by changing how they interact with other people based on what those other people have said ought to acknowledge that it is a radical position and advocate for it outright instead of pretending that you are only advocating for the free speech / free thought principles that everyone in the west has agreed on since the Enlightenment.

          • Brad says:

            @Nabil ad Dajjal

            Good eye picking out my writing style (?).

            I didn’t know who specifically, but I had a strong hunch that you were a regular.

            I was insinuating that it was due to having been doxxed.

            And that’s all well and good. If we were talking about the merits of doxxing, I’d probably agree with your side of things.

            But instead the article and martinw insist on using the term thoughtcrime and you insist on talking about the privacy of the bedroom. Look at my very first sentence: “Wild exaggeration, as is unfortunately typical on this subject.”

            This isn’t a stolen and leaked sex tape. It’s not a stolen diary. It is linking a forum pseudonym to a particular real world person. You can’t just lump up and refer interchangeably to everything you consider bad. Theft is theft, it isn’t murder, it isn’t arson, it isn’t rape.

          • martinw says:

            When I google “thoughtcrime”, the first definition that Google serves me with is “an instance of unorthodox or controversial thinking, considered as a criminal offence or as socially unacceptable”. It sure seems to me that that is what Larry was punished for.

          • Nabil ad Dajjal says:

            I guess I just don’t see the distinction you’re making here.

            Regardless of whether someone steals your diary or doxxes you, the different means they employed nonetheless share the same end. Your privacy was invaded and your personal information was made public.

            To re-purpose your metaphor, theft is theft regardless of whether it takes the form of a pickpocket swiping your debit card or a phishing scam. I don’t see why we should be OK with this just because people scrutinizing the details of his private life used computers to do so.

          • Brad says:

            Forums are a broadcast medium. They are inherently public. The opposite of personal. If his computer had been hacked and his electronic diary leaked then your point of “why is with a computer any different” would be relevant.

            If we were living in small town Yorkshire circa 1900, and I went to York, stood on a soapbox and preached the merits of Labour Party — I could hope it would never get back to my employer, but there would always be the possibility that someone in the crowd was from the same small town and it would get back. Even if I was wearing a wig, I might still be recognized.

            Now you might argue that it was wrong of that person to report back to my employer, and I might even agree with you, but it is an entirely different wrong from stealing my mail, opening it, and giving it to my employer. Or peeking into my bedroom window.

          • martinw says:

            Now you might argue that it was wrong of that person to report back to my employer, and I might even agree with you

            More to the point, I would argue that it was wrong for your employer to fire you because of which political party you liked to proselytize for, outside the office and outside work hours.

            Likewise I am arguing that it was wrong for the Drupal project leader to ban a valued community member for stating, outside of any Drupal-related forum or venue, his opinions about male-female relationships, especially when it wasn’t clear to what extent those opinions were part of a role-playing game he was playing with other consenting adults. (And posting on a members-only forum is not quite the same thing as standing on a soapbox, even if it is relatively easy to become a member.)

            And given that the stated reason for his banning was that his ideas would “inherently influence his actions”, presumably towards women in the Drupal community, even though there appears to be zero evidence that they ever did, I’d say “thoughtcrime” is a fair and non-hyperbolic description.

          • The original Mr. X says:

            @ Brad:

            I think those of you that advocate that no one ever be allowed to take anything anyone else says seriously by changing how they interact with other people based on what those other people have said ought to acknowledge that it is a radical position and advocate for it outright instead of pretending that you are only advocating for the free speech / free thought principles that everyone in the west has agreed on since the Enlightenment.

            And you, in turn, ought to stop hiding behind vague expressions like “changing how they interact with other people”, as if Garfield’s boss had just decided not to invite him to his birthday party or something. What you are actually defending is a person being fired for something he did outside of his work and which had no noticeable effect on his behaviour at work. Own it, or stop trying to defend it.

          • Brad says:

            @martinw

            More to the point, I would argue that it was wrong for your employer to fire you because of which political party you liked to proselytize for, outside the office and outside work hours.

            This is what I was referring to before as the radical notion that no one ought to be allowed to take anything anyone else says seriously.

            I’d say “thoughtcrime” is a fair and non-hyperbolic description.

            Strongly disagree. No one is being punished for their thoughts. And there’s no question here of criminal prosecution. This is hyperbole.

            I know there are a large number of people out there that thinks that words are meaningless things that you can just throw around online “for the lolz” but I disagree. If you — for example — say that people like me should be put in ovens that’s going to have a strong influence on whether I want to work for you, have you working for me, buy things from a company you own, donate to organizations you work for, and so on and so forth. And I’m going to do my darndest to convince other people to likewise have nothing to do with you. I don’t see anything at all wrong with that. On the contrary.

          • The Nybbler says:

            So there are certain thoughts that people unacceptable to be expressed, and these thoughts are _so_ unacceptable that it’s perfectly reasonable for anyone who has been found out to have expressed these thoughts to be fired and/or ostracized.

            That’s downright _libertarian_ of you, Brad, in terms of freedom of association. But I have to wonder, if those thoughts were, perhaps “I enjoy having sex with someone with the same plumbing I have” or “I really feel like the gender I was assigned at birth doesn’t fit with my self-identity” rather than “I enjoy sexual situations where I act as the master and women act as slaves”, would you feel the same way?

          • Brad says:

            No, of course not. It’s those of you that don’t want to get your hands dirty in the icky object level that need to go through contortions to find some terribly implausible meta-level reason why it is okay for Catholic Schools to fire gay teachers but not for March for Dimes to fire anti-gay bigots. As for me, I’m perfectly comfortable dealing at the object level on this one.

            Also, freedom of association has nothing to do with anything. No one is talking about governments here.

          • The original Mr. X says:

            No, of course not. It’s those of you that don’t want to get your hands dirty in the icky object level that need to go through contortions to find some terribly implausible meta-level reason why it is okay for Catholic Schools to fire gay teachers but not for March for Dimes to fire anti-gay bigots.

            I don’t know what March for Dimes is or how they operate, but Catholic schools generally make employees sign a statement promising to uphold Catholic teaching and doctrine. You may consider “This is in direct violation of your contract” to be a “terribly implausible meta-level reason”, but I suspect that will be a minority opinion.

          • Spookykou says:

            Freedom of association here probably means being able to do business, or more importantly, not do business, with people you don’t want to.

          • Gobbobobble says:

            So, people who are sufficiently ideologically divergent deserve to starve, since all right-thinking employers are expected to shun them? (Any divergent employers will be justly boycotted into the ground, of course.) At least felons get a trial before society decides they deserve permanent economic exile.

          • Brad says:

            So, people who are sufficiently ideologically divergent deserve to starve, since all right-thinking employers are expected to shun them?

            “Sufficiently ideologically divergent” — such a bloodless phrase. What’s your alternative to this supposed dystopia? That black people who choose not to shop in a store owned by someone that goes around saying they ought to be lynched be condemned as the *real* bigots?

            It’s an awfully convenient philosophy for a group of people that make a fetish out of contrarianism. But what ever happened to the conservative insistence on personal responsibility?

            What you say has consequences — they can hurt people’s feeling, they can make people dislike you, they can make people angry and so on. And no, saying I’m sorry or I was just kidding doesn’t make everything all better either. I don’t know what kind of upbringing some people had, but you should have learned these lessons a long time ago.

            I don’t owe you a job, I don’t owe you patronage, I don’t owe you anything. If you go around saying offensive shit why in the world shouldn’t I shun you?

            In terms of starving, it’s the conservatives and libertarians that want to get rid of food stamps, not me. Somehow we are supposed to have infinite concern for the people that get fired from high end tech jobs and are so are at risk of “starving” (but not really) but no concern at all for people that are actually, you know, starving.

          • Gobbobobble says:

            I don’t owe you a job, I don’t owe you patronage, I don’t owe you anything. If you go around saying offensive shit why in the world shouldn’t I shun you?

            You’re perfectly free to. What makes you an asshole is when you start crusading into the personal lives of people you disagree with. To quote:

            And I’m going to do my darndest to convince other people to likewise have nothing to do with you. I don’t see anything at all wrong with that. On the contrary.

            Or do you honestly not see the difference between making an individual choice in patronage, and campaigning to get another actor evicted from the marketplace? Based on nothing more than something as bloodless a principle as “saying offensive shit [in private]”?

          • suntzuanime says:

            I can put some blood in my phrases, you horrible Maoist piece of shit, but I try to maintain a certain level of courtesy in this comments section.

          • hlynkacg says:

            But what ever happened to the conservative insistence on personal responsibility?

            I imagine that it was devoured by Molloch, much like the progressive’s hatred of discrimination.

            Surprise! we’re all sinners.

          • The Nybbler says:

            @Brad:

            So it’s perfectly OK, even laudable, for someone to be fired, ostracized, and shunned for discussing their sexual proclivities (anywhere)… unless, of course, those sexual proclivities are on your list of approved and specially-protected proclivities, like homosexuality.

            How wonderfully illiberal of you.

            But when you’ve knocked down all those meta-level principles to get at your hated Gorean, what are you going to do when the alt-right turns round on you and your friends? When they decide something you do is deserving of condemnation and ostracism? Are you going to say what people do in the bedroom is your own business? Because you’ve already knocked down that principle; and forget about privacy, because you’ve already said that they could find out about it without breaking into your stuff, it wasn’t private. You’d be better served by keeping those principles intact.

            (With apologies to Robert Bolt)

          • Cerebral Paul Z. says:

            Because, let’s face it, nothing says “personal responsibility” quite like describing your own decision to harm someone as a consequence of their actions.

          • Brad says:

            @The Nybbler

            So it’s perfectly OK, even laudable, for someone to be fired, ostracized, and shunned for discussing their sexual proclivities (anywhere)… unless, of course, those sexual proclivities are on your list of approved and specially-protected proclivities, like homosexuality.

            I didn’t say that. Why do you insist on knocking down strawmen? Do you have so little confidence in your own position?

          • AnonYEmous says:

            But the original thoughtcrimes, as I recall them, were small signals which could easily be classed as communications. What’s the delineation you’re making? Voluntary vs. involuntary? What if he only wanted to reveal part of what he thought about, what was it, BDSM, without revealing how much it bled into his everyday perspective?

            Edit:

            And now it seems we do know enough specific details, since I read deeper into this thread.

            Look, Brad, I don’t want to assist with swarming you, or anything. But I do want to ask: What is the distinction between the behavior of this Drupal dev, and behavior which constitutes thoughtcrime?

          • vV_Vv says:

            They aren’t administering lie detector tests or truth serums.

            Strawman.

            They used his words as evidence for his thoughts, thoughts that they deemed to be unacceptable because they claim that would inevitably influence his behavior in their community, even if there was no evidence of this happening.

            Therefore it is fair to say that he was fired for thoughtcrime. As others have noted, even in 1984, the book that introduced the word “thoughtcrime”, there was no mind reading technology: words were used as evidence for thoughts.

        • cuke says:

          The cost of the ambiguity in these situations could be avoided through contracts (whether individual employment contracts, union contracts, or work product contracts).

          It seems to me if someone is going to be fired for legal acts (including speech acts) performed outside of work and that are not work-related, then they ought to have a contract with the entity that is paying them that delineates which actions taken outside of work may have bearing on their employment status.

          That way, the “employee” (or independent contractor) can accept or decline based on the terms presented to them, but they don’t have to go around worrying that some stray shit is going to come out of nowhere and bite them on the ass having nothing to do with their performance of the task they’re being paid to do.

          In some fields ethical standards associated with professional licensing provide some of this clarity as well. For instance, mental health workers of all kinds are prohibited from engaging in certain kinds of behaviors outside of work (socializing with patients, for example). In this context, behaviors outside of the work setting can risk one’s license and therefore one’s livelihood. But those standards are pretty clear to the person entering the profession so they can decide whether they’re up for having their non-work behavior regulated in that way.

          If you’re doing your work as part of a particular kind of moral community, the terms of one’s obligation to that moral community ought to be made pretty dang clear. It doesn’t seem like it was in this case.

          I agree with Brad though that our speech acts have consequences and that an internet forum doesn’t come with a presumption of privacy. The piece I object to is that there doesn’t seem to have been any reasonable way this person could have anticipated that the consequences of his speech acts would put at risk his work life since they did not occur in that context. If he’d been sexually harassing co-workers, that would be a different thing.

          I think we should all feel free to think impure thoughts though.

          • cuke says:

            I just noticed that BBA says below that this guy wasn’t fired and that the project was volunteer. I’m not sure it makes a lot of difference in my mind. If Scott banned someone from commenting here because of something unrelated but offensive to Scott that they said elsewhere, how would you all feel about that? It seems clear that Scott gets to eject people from his blog comment space if they say offending things on here, but he’s also gone to some pains to explain what those offensive things might be so that people can make choices about their behavior accordingly.

            People participate in all-volunteer efforts for all kinds of reasons, including some that bear on professional standing and work opportunities and maybe this volunteer project was one of those? So it still seems to me that the gatekeeper of that community ought to spell out in some detail what constitutes ban-worthy behavior, particularly as it relates to behavior outside of the group.

      • suntzuanime says:

        How would you actually go about checking a claim like “the kink he is into is based on sexism”? Sexism is not rigidly defined and the wiggle-room in the definition is regularly used to declare anything the declarer doesn’t like sexist. This is a political question, not anything anybody can go out and check.

        • Deiseach says:

          How would you actually go about checking a claim like “the kink he is into is based on sexism”?

          Very hard to know. Does anyone know how Gorean BDSM in practice stacks up with John Norman’s philosophy as he came to express it, or the attitudes as in the first book of the Gor series, where Tarl Cabot (from Earth but son of a Gorean father) goes from the attitude expressed below to being fully integrated into Gorean society and quite happy to own, rape, and brand slaves of his own? (All the owning, raping, whipping and beating happens to women slaves, by the way; there are male slaves but they’re not used for sexual purposes as the woman are. No homosexuality on Gor! Though there are metric fecktons of homoeroticism:

          The girl I had originally seen had been a slave, and what I had taken to be the jewlery at her throat had been a badge of servitude. Another such badge was a brand concealed by her clothing. The latter marked her as a slave, and the former identified her master. One might change one’s collar, but not one’s brand. I had not seen the girl since the first day. I wondered what had become of her, but I did not inquire. One of the first lessons I was taught on Gor was that concern for a slave was out of place. I decided to wait. I did learn, casually from a Scribe, not Torm, that slaves were not permitted to impart instruction to a free man, since it would place him in their debt, and nothing was owed to a slave. If it was in my power, I resolved to do what I could to abolish what seemed to me a degrading condition. I once talked to my father about the matter, and he merely said that there were many things on Gor worse than the lot of slavery, particularly that of a Tower Slave.

          Hard to tell – is his kink consensual role-playing Master and Slave stuff, or is it 24/7 lifestyle, or does he go full-on Gor mode?

          In a strange way, this is similar to Brendan Eich – if his personal beliefs don’t affect what he does in the workplace or how he treats female colleagues and it’s all legal and consensual, then it’s nobody else’s business. But the sword of ‘firing someone for fear of bad optics’ has two edges, it appears.

          • rlms says:

            Interestingly, wikipedia claims that John Norman’s non-fiction BDSM guide is pretty mild (he suggests that masters should clap loudly as a symbol for whipping, rather than actually whipping their slaves) and he was concerned about psychological harm permanent dom/sub relationships could cause.

          • suntzuanime says:

            A two-edged sword, in metaphor, is one that cuts its wielder, not one that can be used by its wielder to cut two different people. This isn’t hitting the SJWs, this is the SJWs being just as willing to take people down for approving of perversions they don’t approve of as for not approving of perversions they do approve of.

          • eyeballfrog says:

            I must be missing the homoeroticism in that passage.

          • Deiseach says:

            I must be missing the homoeroticism in that passage.

            eyeballfrog, you had to ask 🙂

            Given that canonically tharlarion riders tog themselves out in this kind of gear – well, except for the winged helmet, that would just be silly, no they have broad leather belts fastened tightly round their midsections instead (to go with their canonical special leather tharlarion rider boots) and given some excerpts from the second book as follows, I have to ask (about the first scene) what do you think this sounds like? Especially bearing in mind that the leader is this man as described by Tarl before the whole “tied up to the Frame of Humiliation” part (and really? a name like that for the device? Not the Frame of Horrible Painful Death?) and he’s thinking of him in a way I would expect to read a heroine in a romance novel to be thinking about the hero when they’re still at the UST spitting fire at one another stage:

            I could not forget the figure on the throne, he of the black helmet, and I thought perhaps that he had noticed me and had reacted. It had been, perhaps, my imagination. I sat on the tent carpet, poking at the small fire in the cooking hole. I could hear from a tent nearby the sound of a flute, some soft drums, and the rhythmic jangle of some tiny cymbals.

            Tied up on his back surrounded by a ring of men who all…spit… on him and then the leader…spits… into his hand and wipes it on his chest. I believe I may have encountered references online to a scene something like that in certain Japanese niche publications 🙂

            (1) My wrist and ankles were bound to a hollow, floating frame. The ropes sawed into my flesh as the weight of my body drew on them.

            …I cleared my head as best I could, and into my uncertain field of vision moved a dark object, which became the black helmet of a member of the Caste of Assassins. Slowly, with a stylized movement, the helmet was lifted, and I found myself staring up into a gray, lean, cruel face, a face that might have been made of metal. The eyes were inscrutable, as if they had been made of glass or stone and set artificially in that metallic mask of a countenance.

            “I am Pa-Kur,” said the man.

            It was he, the Master Assassin of Ar, leader of the assembled horde.

            …The stone eyes regarded me, expressionless. “It was [Tarlena’s] wish that you die the death of a villain,” he said, “on the Frame of Humiliation, unworthy to stain our weapons.

            Then, each of the men of Pa-Kur, as is the custom before a frame is surrendered to the waters of the Vosk, spit on my body. Lastly, Pa-Kur spit in his hand and then placed his hand on my chest. “Were it not for the daughter of Marlenus,” said Pa-Kur, his metallic face as placid as the quicksilver behind a mirror, “I would have slain you honorably. That I swear by the black helmet of my caste.”

            “I believe you,” I said, my voice choked, no longer caring if I lived or died.

            I don’t believe I’ve ever before heard the sensation provoked in someone by friendship described as a “flush” but okay, maybe this is just Mr Norman’s prose style (alas):

            (2) I was searching for Kazrak’s tent, which lay in the outer ring near the tharlarion corrals. My calculations had been correct, and in a moment I had slipped inside the domed framework of his tent. I dropped the ring that I wore, with the crest of Cabot, to his sleeping mat.

            For what seemed an interminable hour, I waited in the dark interior of the tent. At last the weary figure of Kazrak, helmet in hand, bent down to enter the tent. I waited, not speaking, in the shadows. He came through the opening, dropped his helmet on the sleeping mat, and began to unsling his sword. Still I would not speak, not while he controlled a weapon; unfortunately, the first thing a Gorean warrior is likely to do to the stranger in his tent is kill him, the second is to find out who he is. I saw the spark of Kazrak’s fire-maker, and I felt the flush of friendship as I saw his features briefly outlined in the glow. He lit the small hanging tent lamp, a wick set in a copper bowl of tharlarion oil, and in its flickering light turned to the sleeping mat. No sooner had he done so than he fell to his knees on the mat and grasped the ring.

            “By the Priest-Kings!” he cried.

            I leaped across the tent and clapped my hands across his mouth. For a moment we struggled fiercely. “Kazrak!” I said. I took my hand from his mouth. He grasped me in his arms and crushed me to his chest, his eyes filled with tears. I shoved him away happily.

            “I looked for you,” he said. “For two days I rode down the banks of the Vosk. I would have cut you free.”

            “That’s heresy,” I laughed.

            “Let it be heresy,” he said. “I would have cut you free.”

            “We are together again,” I said simply.

            “I found the frame,” Kazrak said, “half a pasang from the Vosk, broken. I thought you were dead.”

            The brave man wept, and I felt like weeping, too, for joy, because he was my friend.

            And then we go from this (shackled at the feet of a magnificent barbarian with masterful eyes)

            (3) …Shackled in a kneeling position, my back open and bleeding from the lash, I was thrown before the Ubar. Nine days I had been a prisoner in his camp, subjected to torture and abuse. Yet this was the first time since I had saved his life that I had seen him. I gathered that he had finally seen fit to terminate the suffering of the warrior who had stolen the Home Stone of his city.

            One of the tarnsmen of Marlenus thrust his hand in my hair and forced my lips down to his sandal. I forced my head up and kept my back straight, my eyes granting my captor no satisfaction. I knelt on the granite floor of a shallow cave in one of the Voltai peaks, a sheltered fire on each side of me. Before me, on a rough throne of piled rocks, sat Marlenus, his long hair over his shouders, his great beard reaching almost to his sword belt. He was a gigantic man, larger even than the Older Tarl, and in his eyes, wild and green, I saw the masterful flame which had, in its way, also burned in the eyes of Talena, his daughter. Die though I must at the hands of this magnificent barbarian, I could feel no ill will toward him. If I had had to kill him, I would have done so not with hatred or rancor, but rather with respect.

            to this (bursting through barriers as if they were tissue paper to embrace)

            (4) … It was behind this makeshift rampart, which could be defended against a hundred men, that I saw the haggard but still blazing eyes of Marlenus. I removed my helmet and set it on the steps. In a moment he had burst through the obstruction as if it had been made of kindling wood.

            Wordlessly we embraced.

            And bearing in mind that everyone is walking around in standard Barbarian Battle Garb (i.e. loincloths and not much else apart from weaponry everywhere) while all this embracing and crushing to chests and flushing with friendship and spitting on bodies is going on… well okay it was the 60s (man) and those were still more (relatively) innocent times, but you read these passages today (and the first couple of books are full of them) and the slash goggles practically affix themselves to your face without your conscious will. It’s even more of a stark contrast because in the first couple of books, Tarl Cabot is still struggling with the vestiges of 20th century British gentleman upbringing so when the Damsels in Distress keep hurling themselves at him for him to use for his pleasure according to the modality of Gor, he keeps trying to be chivalrous and refuses them, but with men (or male spiders) it’s all embraces and light stroking and tactile investigations, as well as more saliva or “related type of secretion” (please notice my restraint in not offering further comment on that phrasing). Actually, Nar the Spider is very interesting because not alone is he the one sensible, intelligent, nice character in Gor so far, he’s a “he” where traditionally and culturally spiders tend to be coded as female, and when it’s a male spider exuding bodily secretions, after tactile investigations, in the very near vicinity of Our Hero that’s a whole different kettle of fish:

            When I opened my eyes, I found myself partially adhering to a vast network of broad, elastic strands that formed a structure, perhaps a pasang in width, and through which at numerous points projected the monstrous trees of the swamp forest. I felt the network, or web, tremble, and I struggled to rise, but found myself unable to gain my feet. My flesh adhered to the adhesive substance of the broad strands. Approaching me, stepping daintily for all its bulk, prancing over the strands, came one of the Swamp Spiders of Gor. I fastened my eyes on the blue sky, wanting it to be the last thing I looked upon. I shuddered as the beast paused near me, and I felt the light stroke of its forelegs, felt the tactile investigation of the sensory hairs of its appendages. I looked at it, and it peered down, with its four pairs of pearly eyes.

            The monstrous insect bent near me and I caught sight of the mandibles, like curved knives. I tensed myself for the sudden lateral chopping of those pincerlike jaws. Instead, saliva or some related type of secretion or exudate was being applied to the web in my vicinity, which loosened its adhesive grip. When freed, I was lifted lightly in the mandibles and carried to the edge of the web, where the spider seized a hanging strand and scurried downward, placing me on the ground. He then backed away from me on his eight legs, but never taking the pearly gaze of his several eyes from me.

        • Deiseach says:

          Oh brother, this is bringing me back to 2007 and a lively discussion on a now-defunct blog about the Gor books; the general opinion was that yes, they were decent pulp SF when they began but John Norman’s kink or obsession or whatever it was rendered their quality worse as time went on and his view of women (in Gor at least) degenerated from the idea of both free women and slaves with lives, minds and wills of their own, to the likes of this (from “Renegades of Gor”, 1986, some twenty years on from the first book):

          I had little doubt, accordingly, that the blonde woman kneeling before the fellow with the whip was his free companion, or former free companion. …But now the slave was gone, and there was a chain on her neck. If she were still his free companion, it seemed she would now be kept in the modality of bondage, but perhaps she was now only his former free companion, and had been reduced to actual bondage, now being subject to purchase by anyone. I recalled how she had bent in terror to kiss his feet. There was no doubt that she would now take her relationship to him seriously.

          It is difficult not to do so when one is owned, and subject to the whip. The woman would now discover that her companion, or former companion, a fellow perhaps hitherto taken somewhat too lightly, one perhaps hitherto accorded insufficient attention and respect, one perhaps hitherto neglected and ignored, even despised and scorned, was indeed a man, and one who now would see to it that she served him well, one who would now own and command her, one who would summon forth the woman in her, and claim from her, and receive from her, the total entitlements of the master.

          I mean, sheesh. When they started out they were spicy pulp SF – Barsoom with extra sex and a bit of kink and if they’d stuck to that, it would have been fine; there’s an alien character in one of the early books, a sentient and sapient spider, and it’s a perfectly good character and one you’d like to see explored. If Norman had stuck to “Gods and Fighting Men of Gor” (with scantily clad slave girls and mildly kinky sex), there would have been no problem.

          But certainly from the 80s books onward, it got (if it hadn’t already been) what is termed “problematic” on Tumblr, and I have to agree with the concept here. It’s not just Gorean women who want to be sex slaves, it’s Earth women as well. From the first book we know that Goreans have been raiding Earth for slaves (some men as well as women) but in the 80s books we get all the going into detail about breaking a slave and how to turn one of those silly Earth women who thinks she’s of equal value to a man into what she really is and wants to be, and the Earth men who have been emasculated by the female-dominated society of Earth learn how to be Real Men. It’s “treat ’em mean and keep ’em keen” dialed up so far, the knob has broken off. The Free Women of the earlier books who are the caste equals (in a sense) of the men are downgraded to being frigid, unappealing, nagging nuisances so it’s no wonder their menfolk prefer beautiful, biddable, Real Women slaves instead.

          • Chalid says:

            Out of curiosity, what happens to women in this world when they get old? Or is it just swept under the rug? (I understand that this sort of book isn’t about detailed consistent worldbuilding.)

          • Le Maistre Chat says:

            @Deiseach: I mean, sheesh. When they started out they were spicy pulp SF – Barsoom with extra sex and a bit of kink and if they’d stuck to that, it would have been fine; there’s an alien character in one of the early books, a sentient and sapient spider, and it’s a perfectly good character and one you’d like to see explored. If Norman had stuck to “Gods and Fighting Men of Gor” (with scantily clad slave girls and mildly kinky sex), there would have been no problem.

            Funny thing, Barsoom has slavery, and everyone from slave to princess is a nudist.

            Oh yeah, the first book includes pacifist Spider People. And the Priest-Kings have immortality technology so their leader is 5 million years old, and apparently some time after Book 3, it’s revealed that they’re protecting us from Bear People who live in O’Neill cylinders in the asteroid belt and want to conquer the Solar system.
            Such potential.

            @Chalid: I don’t think so. Gorean humans are kind of elf-like in that they have the technology to stay young for centuries, and I can’t remember if there’s any worldbuilding addressing old people.

          • Deiseach says:

            Funny thing, Barsoom has slavery, and everyone from slave to princess is a nudist.

            They’re also oviparous, which amuses me greatly when you get the standard SF cover of bosomy, voluptuous and scantily clad Deja Thoris 🙂

            If I had to pick between Barsoom and Gor, I’d pick Barsoom. If I’m going to be pushed around by Manly Barbarian Fighting Men, I prefer ’em green four-armed giants than Earth guys with an over-inflated sense of their own studliness.

      • Ozy Frantz says:

        I mean, Goreanism is *explicitly* about how women should be subservient, that’s the thing that the fetish is. Of course, there’s no reason to believe that any particular Gorean is worse at separating fantasy and reality than anyone else.

        • Le Maistre Chat says:

          There are numerous belief systems that say women should be subservient. So screw that, Ozy, I hereby declare that Goreanism is a type of Luddism that says we must abolish chemistry, all the way down to knowledge of gunpowder, because uncontrolled messing with chemicals will lead to environmental collapse and the post-apocalyptic wasteland will be enslaved by bears smart enough to build orbital habitats.
          Praise the Priest-Kings!

          • The Nybbler says:

            I’ve heard the Gor books described as some of the worst science fiction ever published, and that’s before you get to the sex stuff. If what you just wrote has any relation to the actual books, I believe it.

        • random832 says:

          Of course, there’s no reason to believe that any particular Gorean is worse at separating fantasy and reality than anyone else.

          I mean, the thing I’ve noticed is…

          When googling phrases like ‘difference between Goreanism and BDSM’, one finds pages that at least appear to be Goreanist sources, that are at least grounded enough in reality to acknowledge the existence of the real world, other communities, etc… that claim Goreanism eschews concepts like safewords. The overall message seems to be that even if someone has to choose to be a ‘slave’, it’s not a revocable choice (“consensual up to a point” is a phrase one such page uses) – that is, having made that choice there doesn’t seem to be a provision for someone to decide that she’s had enough, that participating in the fetish or having a particular person as her ‘master’ was a mistake.

          If that’s meant to be the reality, rather than just a layer of the fantasy play-acting, that’s not a good look.

          To my understanding (though I have no direct experience), safewords aren’t a fantasy concept, they are a reality concept, a way to dump everyone out of the fantasy scene. Talking about them means either talking about reality or constructing some kind of elaborate multilayered fantasy.

        • vV_Vv says:

          I mean, Goreanism is *explicitly* about how women should be subservient

          Something something Christianity, something something Islam…

          But anyway, as long as somebody’s beliefs don’t cause them to behave in ways that cause others harm or are socially unacceptable, why would you care?

    • The Nybbler says:

      Best I can tell “fired for his beliefs” was the second explanation. First explanation was “fired for his kink”. Buytaert says on his blog post “I did this because it came to my attention that he holds views that are in opposition with the values of the Drupal project.”, so he certainly stated a clear reason. Difference is “fired for his beliefs” is perfectly acceptable in the SJW milieu (and Drupal’s a notoriously SJW project), but “fired for his kink” would be unacceptable.

    • Nabil ad Dajjal says:

      There’s an element of Occam’s razor here.

      Alice has an embarrassing and politically incorrect fetish. It comes to light and she is fired from Bobcorp immediately afterwards. Then, after Bobcorp is criticized for the firing, a Bobcorp flunky vaguely implies that Alice might also have done something else that actually necessitated firing her which just happened to come up at exactly the same time.

      I’d wager (figuratively: I don’t gamble) that it’s a lot more likely that this is SJW prudishness rather than any substantial misconduct on the developer’s part.

      • wysinwygymmv says:

        Not sure why we’re assuming SJWs in general are extraordinarily prudish rather than more likely to be involved in BDSM in the first place. SJW sentiments aren’t exactly hard to come by at fetish clubs.

        • Whatever Happened To Anonymous says:

          I think that, if there’s a meaningful distinction between SJ and SJW, the latter would probably be more prudish than the former.

          • wysinwygymmv says:

            I’m not sure I’ve ever seen “SJ” used to refer to a group of people, so I’m not sure how much sense this comparison makes in context.

            “SJW” is used indexically to indicate people who are prone to making certain kinds of utterances — those utterances tend to have to do with race, gender, and similar facets of identity. In my admittedly limited experience, people who hang out at fetish clubs and fetish chat rooms and fora online tend to say a lot of things about race, gender, and similar facets of identity. That means that they are quite reasonably described as “SJW”.

            If you want to make a more useful, more fair distinction, you should probably just drop the term “SJW” and describe what you mean.

          • Whatever Happened To Anonymous says:

            I’m not sure I’ve ever seen “SJ” used to refer to a group of people, so I’m not sure how much sense this comparison makes in context.

            I think a more clear term would be “SJ aligned”. Someone who’s “SJ aligned” would fit roughly in your definition of SJW, while a SJW would be the kind of person who acts on those utterances and beliefs, usually against people who are “evil” within this mindset.

            I don’t think these people in the kink scenes are particulary prone to participating in “getting racists fired” schemes.

        • Spookykou says:

          I assumed “SJW prudishness” meant prudishness of the SJW variety, as if they are prudish about particular things that other people are not prudish about. Not that they are particularly more prudish in general.

          Hypothetically someone who is homophobic could be particular prudish about homosexual sexual acts while being more lenient towards heterosexual sexual acts, maybe this could be called “homophobic prudishness”?

    • Protagoras says:

      Since this is couched in terms of sexism and so is being presented as SJ overreach, I do feel that as one of the local leftists I should comment, even if I feel I don’t have a huge amount to add. My own position is that if things are as Larry describes, he should certainly not have been fired. But while I am skeptical of the convenient “there are things I can’t say,” it is true that there are things employers shouldn’t say, so as a complete outsider I don’t feel I have enough information to come down unequivocally on Larry’s side.

      • Iain says:

        I am currently almost entirely on vacation in Europe, but I will step in to second this.

      • martinw says:

        Larry claims that if it is about anything other than the Gor thing, then he hasn’t been told either. He could be lying of course, but in that case you’d think Dries could at least confirm or deny that.

      • random832 says:

        it is true that there are things employers shouldn’t say

        What do you think about the position that anything you shouldn’t say, you also shouldn’t fire someone for?

        • Protagoras says:

          That it would require dramatic changes in both employment norms and the law to bring about a state of affairs in which that would make sense. Perhaps those changes would be desirable, but I wasn’t interested in digressing into such larger policy questions here.

      • vV_Vv says:

        But while I am skeptical of the convenient “there are things I can’t say,” it is true that there are things employers shouldn’t say, so as a complete outsider I don’t feel I have enough information to come down unequivocally on Larry’s side.

        In that case the employer could have said something to the effect of: “Larry’s characterization of the reason he was fired is incorrect. While we don’t endorse his beliefs, it is not our job to police them. Larry was fired for issues that we can’t discuss in order to protect the privacy of the people involved.”

        Instead, they made it perfectly clear in their statement that he was primarily fired for his beliefs. The “there are things I can’t say” sounds like a cop out.

    • pseudon says:

      Came here to say: Of course he’s into BDSM. He works on a PHP project.

    • BBA says:

      Just a note on this story – nobody was fired. Mr. Garfield was ejected from a volunteer project in which (to my knowledge) he received no compensation for his contributions. His employer still lists him as an employee. There’s been nothing to suggest that his livelihood is at risk from this decision. This is not Eich.

  18. wohlfe says:

    Apropos of nothing (except maybe insomnia and a crippling caffeine addiction…), I found myself reading speeches of FDR, Churchill, and Hitler. I was struck by how in many of Hilter’s speeches, you could do as little as replace “Jews” with “bankers” or “the 1%” and they would sound exactly like modern anti-capitalists or progressive figures like Elizabeth Warren and Bernie Sanders. I’m not trying to make a serious connection, but it gave me a slight chuckle.

    • herbert herberson says:

      That’s sort of always been the point of antisemitism, no? “Hey, angry peasants, don’t revolt against the feudal aristocracy that practically owns you and almost everything else, go pogrom his creditors with the funny hats and relatively nice houses!”

      • wohlfe says:

        Maybe so, it’s probably easier to redirect people’s anger from systematic political issues to a more specific economic enemy subverting their politics. Especially when that same enemy is the one you’re indebted to, or foreclosing your house.

        • Nyx says:

          Well, Orthodox Jews are conspicuously visible, with their hats and beards and funny rituals and stein-y names. While such practices and rituals can help to prevent cultural drift (part of the reason that Jews have endured for so many millenia compared to most other Bronze Age tribes), they also make them vulnerable.

      • wintermute92 says:

        Yeah, the connection is meaningful, but suggesting Warren and Sanders have ‘returned’ to that sort of dialogue gets it backwards.

        Nazism arose from economic crisis, and worked by metonymy between financial groups and ethnic groups. Antisemitism has hinged on this trick throughout the world – even talk about the Pharisees often appeals to this idea. It’s not an accident that Stalin worked both ideas at once, attacking Jews, ‘kulaks’, and the bourgeoisie in exactly the same language.

        There’s a second-tier point here where we talk about intensity and violent rhetoric. But I think the comparison breaks down here – Sanders isn’t talking about killing the rich. At most that’s an attack on serious violent communism, not American progressivism.

    • Salem says:

      You are by no means the first person to make this link. I actually do think there’s a serious connection. Anti-semitism, communism and modern anti-capitalism come from the same place – a hatred of the other, anti-foreign bias, and anti-market bias. It’s not a coincidence that they all blame “international finance capital”, “rootless cosmopolitans” and “banksters.” The only difference is how they line up the oppressor/oppressed in their narrative.

      • doubleunplussed says:

        Sure, but if you think there is ever such a thing as actual oppression of one group by another, then it is going to look this way too. Surely some claims of oppression are due to actual oppression, and not just bias.

        And it might just be my bias, but I think the claims of the poor that they are oppressed by the rich are the most legitimate, and just because the poor often make bad arguments and single out the wrong people as causing their oppression, doesn’t mean they aren’t oppressed.

      • Spookykou says:

        I just assumed that people could be upset by wealth inequality in and of itself, and that antisemitism leans on some of that otherwise orthogonal animus.

        • Hyzenthlay says:

          I just assumed that people could be upset by wealth inequality in and of itself, and that antisemitism leans on some of that otherwise orthogonal animus.

          I’d say that’s an accurate characterization. People like having a specific demographic to blame for their (often very real) problems. Hating “inequality” or “capitalism” (or “communism” for that matter) isn’t very satisfying, because those are abstract concepts. Even hating “the rich” is kind of tough because wealth is a relative concept, and also wealth fluctuates and isn’t always a highly visible thing.

          On the other hand, hating a specific ethnic group that is perceived as disproportionately rich and privileged…just knowing how people are, it’s easy to see how that became a thing.

          Then again, it’s not uncommon to see patterns of paranoia and othering happening purely around the perceived beliefs of the targeted group. There was McCarthyism. And then there was the whole hysteria over Satanism in the eighties, and now everyone is panicking over fascism/white nationalism, etc.

          And those panics aren’t totally manufactured (well, except maybe for the Satanist one). Some of the people blacklisted were actual communists, and there are some actual fascists/Nazis hanging around. But with all these panics there’s a sense of, “They’re hiding in our midst. Your neighbor might be one of them” and pressure to name names and condemn the accused, lest you yourself be accused, and a joke or an offhand comment can have career-ending implications.

          I’ve probably wandered off-topic, but it’s just kind of fascinating to me how this pattern mutates and keeps appearing in different forms. Sometimes as a response to previous panics.

          • wintermute92 says:

            It also occurs to me that hating “the rich” is frustratingly devoid of a clear cutoff. “The 1%” has been a great rhetorical trick for other-ing the rich, but it keeps breaking down when someone realizes that their family doctor is in the 1% and some corporate manager they hate isn’t.

            “The 0.5%” or “the 0.1%” are somewhat more accurate in terms of singling out the exceedingly rich who absorbed the post-recession recovery, but those measures still conflate “some rich surgeon I like” with “evil corporatists”.

            No one seems to have made this distinction accurately and cleanly. (Probably it can’t be made because this is all a gradient.) Antisemites completely redirect the problem to hit a distinct group, but one that’s often not rich or powerful in the first place. State communists generally redirect the problem to “whoever I hate”, while an-coms generally imply we can solve the problem with structural change and don’t need to identify villains.

            It’s a messy argument when made honestly, so adding ethnic tension adds the sort of clarity that drives people to hate and violence.

          • IrishDude says:

            @wintermute92

            No one seems to have made this distinction accurately and cleanly.

            I think the chart in this tweet does a decent job.

      • NostalgiaForInfinity says:

        Anti-semitism, communism and modern anti-capitalism come from the same place – a hatred of the other, anti-foreign bias, and anti-market bias

        Those last three hardly seem like “the same place”. Even leaving aside the fact that most modern anti-capitalists (on the left) are enthusiastically pro-other and pro-foreign/immigrant.

      • vV_Vv says:

        Anti-semitism, communism and modern anti-capitalism come from the same place – a hatred of the other, anti-foreign bias, and anti-market bias.

        They claim that fair market-based competition is a hypocritical political fiction, and that the oppressors (the Jews/the bourgeoisie/the 1%) collude on a ethno-religious/social class basis and game the system against the common man with anti-competitive policies. “Socialism for the rich, capitalism for the poor” is the common complaint.

        Whether this is factually true can vary depending on the specific case.

        • Spookykou says:

          I would have never thought of the 1% as colluding, I would just assume that Capitalism would naturally generate wealth inequality. Although I am not very prone to conspiratorial thinking/explanations in general so maybe this is the common position.

          • herbert herberson says:

            Doctrinaire Marxism certainly doesn’t think it’s collusion, unless you count the more-or-less natural tendency for competition to force capital to pay the lowest possible wages.

            It’s kind of the whole point–the crimes of capitalists aren’t due to moral failings, they’re intrinsic to the interests of the class. That’s why it seeks to upend the class structure rather than convince capitalists to be nicer.

          • vV_Vv says:

            I would have never thought of the 1% as colluding

            So when they lobby the government in various ways to advance their interests at the expense of the rest of the population what is it if not collusion?

            Sure, they might compete between each other, but as the Bedouins say: “I, against my brothers. I and my brothers against my cousins. I and my brothers and my cousins against the world.”

          • Spookykou says:

            I just assume that is a byproduct of individual actors working for their rational self interest. This can create a ‘system of oppression’ but it seems to me a natural byproduct of humans set loose in a capitalist society. It is not the same as an evil cabal meeting in dark rooms to plot and scheme how best to screw over the masses. Which is the image that comes to mind when I read.

            the oppressors (the Jews/the bourgeoisie/the 1%) collude on a ethno-religious/social class basis and game the system against the common man with anti-competitive policies.

          • Hyzenthlay says:

            It is not the same as an evil cabal meeting in dark rooms to plot and scheme how best to screw over the masses. Which is the image that comes to mind when I read.

            Well, the rooms are probably well-lit, but.

            If you’ve seen The Informant! (based on a true story) it’s about just that: a group of high-level business people conducting a price-fixing conspiracy to improve their profits at the expense of the masses.

            I don’t know how common this sort of thing is, but it does happen.

          • Nornagest says:

            a group of high-level business people conducting a price-fixing conspiracy to improve their profits at the expense of the masses.

            The word for this is “cartel”, and while it’s not exactly rare, it’s not as common as you might think. The basic problem is that the higher prices are fixed at, the more attractive it becomes for an outsider to come in and undercut the cartel, or for a member of the cartel to defect if they find themselves needing short-term profit for some reason. (It’s also illegal in most jurisdictions.)

            For these reasons, you usually see them in very small markets (think “the only two gas stations for fifty miles”) or markets with very high barriers to entry (think “OPEC”).

          • Mark V Anderson says:

            I would have never thought of the 1% as colluding

            So when they lobby the government in various ways to advance their interests at the expense of the rest of the population what is it if not collusion?

            I have never seen the 1% agree on anything, so collusion isn’t the right word. There are several billionaires that lobby for the right, and there are several billionaires that lobby for the left. And then there are others that don’t seem to fit either side, such as Trump, Gates. No collusion at all.

      • wysinwygymmv says:

        I love coming by SSC so that right wingers can tell me what I believe in my heart of hearts.

        • Spookykou says:

          I think it is interesting and helpful to see how other people try and explain my(or just not their) beliefs.

          vV_Vv’s explanation is very interesting to me because it includes some assumptions(about economic mobility) that I would consider right wing , in an explanation of a left wing concept.

          • wysinwygymmv says:

            Good point, I should probably take it as more of an anthropological exercise than a meeting of the minds. I’ve already sort of determined that people here are not nearly as open-minded and ready to change their beliefs as they like to say or imply.

        • suntzuanime says:

          Isn’t calling vV_Vv a right winger itself a claim about the beliefs of someone else? You can’t really avoid it honestly.

    • The original Mr. X says:

      If you go through some of Juvenal’s poems and replace “Greeks” with “Jews”, it also sounds a lot like Hitler. If Hitler was an ancient Roman writing in dactylic hexameter, that is.

    • suntzuanime says:

      Yeah, progressives are real worried about banker blood polluting the pure German race.

    • reytes says:

      I am incredibly skeptical of this claim. I would greatly appreciate examples of speeches by Elizabeth Warren or Bernie Sanders which sound like Hitler’s speeches, or speeches by Hitler where you can replace “Jews” with “bankers” or “the 1%” and have it sound like something that Bernie Sanders or Elizabeth Warren would actually say, or generally which speeches or statements which you read and had in mind.

    • pipsterate says:

      “Der Antisemitismus ist der Sozialismus der dummen Kerle.”

    • MartMart says:

      It may be that there is a limited number of ways to effectively motivate large crowds.

  19. youzicha says:

    China Lake, the research center that did the ramjet, is most famous for developing the Sidewinder missile in the 1950s. This was another dramatic success story, where private companies like Hughes were developing very complicated and expensive missiles that didn’t work at all (the components were not reliable enough), and China Lake took an extremely lean approach, both in terms of management structure and in terms of design. There is a a book about it.

    It seems the same lab was beating cost disease all the way back in the 50s. 🙂

  20. onyomi says:

    Re. BDSM, why do nerds love BDSM (no, I am not good enough at math to be that type of nerd)? I have heard, for example, that it is extremely popular at MIT, where (last I heard; my info may be out of date), it was called “Klingon.”

    • Protagoras says:

      Well, here’s one wild ass guess at a possible factor. Social power games are a big part of sex. Nerds don’t like the normal social power games, because they’re bad at them, but excluding them makes the sex less sexy. BDSM provides an alternate set of social power games with clearer rules, so they can get the fun of playing with power while staying more within their skill set (they’re better with clearer rules and their lack of experience with the more mainstream power games isn’t as much of a hindrance in the alternate games).

    • leoboiko says:

      We can only speculate. We don’t even know if nerds really love BDSM more than non-nerds, but let’s assume that’s true:

      – BDSM seems to have a therapeutic effect for victims of violence and abuse; and many nerds have had experiences with violence and abuse. By ritualizing and roleplaying situations of power differential, of domination and submission, you make these situations “yours”; they’re now safe, controllable. Nerds may often feel socially vulnerable or oppressed, and playing at power (on both sides, incidentally) acts as a relief. It’s a bit like exposure therapy, except humans have this weird feature where you can flip a binary flag and turn “scary, taboo, revolting” straight into “goddamn this is hot”.

      – BDSM is a subculture, not unlike any other kind of fandom, which its own lingo, fashion style etc. (In fact, the guy from the article wasn’t just a BDSM enthusiast, but a Gorean; Goreans are like live-action roleplayers of an old sci-fi story featuring male sexual domination). If you’re already invested into dressing like a heavy metal fan, why not add a leather speedo and try something new? You’re already willing to risk social ostracism for that sweet feeling of belonging, of structure.

      – Many nerds tend to political/ethical commitment. In the political spectrum, many leftists/gender-theorists/etc. are heavy into BDSM, for reasons of sexual liberation, progressivism etc. Many conservatives/alt-righters/libertarian/sociobiology believers etc. are heavy into BDSM, because they think it proves that women want to submit to a strong hand and that’s the natural order of things (submissive men, when they’re acknowledged at all, are explained away as ‘betas’). All of these things appear to me to cluster together: nerdish interests, social outcast status, bullying/abuse, political commitment, sexual fetishes. I think many of them are contagious: I know I had no interest in BDSM play until I was introduced to it by my girlfriend, and now it’s very much a thing for me. The dynamics seem similar to that of a nerd introducing his fellow nerd to a new fandom, or hobby, or attractive political principle (“markets are mathematically more efficient, everything should be markets!” “workers should own the means of production!”).

      I’m rambling, sorry. TL;DR—BDSM is a memeplex, and fandom already has channels for the spreading of this kind of meme.

      – This bit is definitely overanalysis, but here goes nothing. When playing an RPG, you act the role of the fictional character, and you keep a clear distinction between the character and yourself. But “yourself” is also a character, or rather, a social role: the role of a “RPG player/DM in a gaming session”, which its own scripted behaviors, code of conduct etc. There are three levels of identity, three roles: the character, the player, and the philosophical subject observing it all (and taking decisions on how to act).

      The philosophical subject knows that there are rules about what you can do as a character, and another set as a player. These rules are previously negotiated, with the goal of making play sessions satisfying for everyone (“we can make jokes but don’t derail the session”, etc.) At first sight, the DM seems to hold all power; but this is just at the bottom-most, fictional level. At the player level, the DM is actually beholden to their players; if they don’t conduct sessions in a mutually satisfying way, the players will just walk out. The DM only has their powers because the players want to play, and therefore empower them; that is, power flows from the players to the DM. This is a direct parallel of how, in BDSM, power flows from subs to doms. BDSM also involves exactly the same multi-layered roleplaying; during a play session you roleplay sub or dom (or Gorean slave or Daddy or w/e); but between sessions you perform “BDSM practitioner”, which may involve things like typing out contracts, renegotiating limits, setting safewords, finding people in clubs/Fetlife and so on.

      And yet, even though it’s all so symbolic and make-belief and, well, “fake”, a good RPG session still feels real at the character level. If someone steal your magic treasure, you really feel angry. If your paladin gets a natural 20 in a tight spot, you really feel relieved as if you’ve just escaped a great danger. In the same way, even though at the upper level power flows from the sub, during a play session the sub really feels dominated, and the dom really feels dominant. Imagination is powerful. Life is larger than mere reality.

      This cognitive isomorphism means that RPG players are perfectly equipped to understand and enjoy the somewhat complex architecture of consensual BDSM. They both even have Dungeon Masters…

      References: I’m not sure but I think I took the three-layer model from Gary Alan Fine; I’m behind the paywall now so I can’t check.

      • Le Maistre Chat says:

        This cognitive isomorphism means that RPG players are perfectly equipped to understand and enjoy the somewhat complex architecture of consensual BDSM. They both even have Dungeon Masters…

        *clap, clap* Comment of the week.

    • andrewflicker says:

      High Openness? *shrugs*

    • Nancy Lebovitz says:

      Have a boring theory. Assume that a lot of people like BDSM. Assume that nerds want to be more reasonable than conventional society. Observe that there’s a norm (an eroding norm, but I think it’s still there) against BDSM.

      From which it follows that if a nerd might like BDSM, they’re more likely to engage in it.

      • onyomi says:

        Also possible that nerds, prone to overanalyzing and not known for social subtlety, are equally likely to engage in it but more likely to talk about it.

        *Edit Tayfie says basically the same thing below.

    • Nabil ad Dajjal says:

      Do nerds love BDSM?

      Maybe biochemists aren’t nerdy enough to count but I know of exactly one other scientist who’s into it. And she’s really really far from being an archetypal nerd. People here aren’t very hesitant to talk about their sex lives but if anything it sounds more vanilla than the general public.

      I wouldn’t exactly call nerds prudish but they / we don’t seem particularly adventurous at any rate.

    • tayfie says:

      I don’t think nerds like it more than average, but they are less worried about people finding out. The non-nerds may like it just as much, but are more socially conscious and therefore keep it hidden better.

    • skef says:

      MIT undergrads … so much time on their hands.

  21. bean says:

    Re the electric airplane, no, it won’t be silent. I don’t have hard numbers right now, but a fair bit of the noise is aerodynamic in origin, and that won’t be helped by the electric motors. You can still hear quadcopters, after all.
    This whole thing looks like a soap bubble. In context, I work for a major manufacturer of airliners. At least 90% of my job is finding ways to placate the bureaucracy gods so that we can publish instructions on how to make the planes safer. (This is worse than average, but not that much worse.) Certification is a really big deal, and one that a research team from NASA is totally unprepared to deal with. A couple of points. First, I suspect easyJet is taking a page from RyanAir’s book, and is involved for cheap publicity. Second, why are they starting at 150 seats? Start smaller, in a market that will be a lot cheaper to enter, and where you can actually make typical performance figures. Build something with 50 seats. Saying “30% of narrowbody flights are under 300 miles” is true, but that doesn’t mean an airline will be able to simply replace 30% of their fleet with your planes. Scheduling all of your short-haul flights on one fleet and the long-haul ones on another makes their life harder. Third, remember what happened last time someone put advanced batteries on an airliner.

    • Eltargrim says:

      All of this is vapour anyway. Batteries don’t presently have the energy density for economically efficient passenger flight at any range. Barring some incredible breakthrough (and I work in the same building as people trying very hard to make one), I would be surprised if we ever get battery-powered commercial flight as a realistic option.

    • I think you’re entirely right about the regulatory issues but electric planes do have an advantage that can make them less noisy. Electric motors are relatively simple which makes it feasible to have a large number of them scattered over the plane which apparently can increase efficiency quite a bit, enough that the lower power storage density of batteries versus fuel becomes merely a serious issue rather than a reason that electric planes are entirely infeasible. And the energy savings mostly come from not losing as much energy as noise.

      • bean says:

        I don’t really buy that. Modern propellers aren’t that inefficient. Yes, by doing what you’re proposing could get you better efficiency, but not enough to overwhelm the disadvantages of batteries as power source.
        That said, I’m working form fairly hazy memories, so if you have numbers, I could be convinced.

        • I’m working from an article I read in The Economist a while ago. Link.

          • bean says:

            Interesting. I hadn’t thought of using electric props to move airflow around like that, and it could help some. But not enough to make something like the OP feasible without really big improvements in batteries. This is definitely tech which we’ll see first in light airplanes.

    • bean says:

      I’ve looked into the noise thing more, and it appears that on approach, a modern airliner has airframe noise which is approximately comparable to the engine noise. On takeoff, engine noise obviously dominates. But that’s for jets. I haven’t been able to run down numbers on props yet (this stuff is proving a lot harder than I expected to track down), but this doesn’t leave me optimistic that an electric plane would get more than a 10 dB reduction in noise. Which is significant, but still a long way from silent.

      • hlynkacg says:

        I don’t have the numbers in front of me but I would note the standard technique for making a prop quieter is to increase it’s diameter while reducing it’s RPM.

        • bean says:

          I’m aware of that, but all of my references on noise and silencing have to do with sonar, not airplanes. The problem is that while there are lots of papers on modeling and simulation, I haven’t been able to find ‘a technical introduction to aircraft noise’, at least not online. There are lots of interesting-looking books in the catalog, but none of them are online, and it’s not worth ordering them.
          Edit:
          Found something. Generalizing badly from a graph I found, it looks like the jet itself is usually about half of the propulsion contribution to the noise, the rest coming from the fan. Slightly more on takeoff, significantly less during approach. As such, getting rid of the jet (and assuming that the prop is broadly equivalent to the fan) will save at most 3 dB. That’s not nothing, but it’s also not all that much.

    • hlynkacg says:

      Like Bean I work in the aerospace industry and while I wouldn’t go so far as to call it a fraud, my impression reading the link is that this is a marketing gimmick rather than a serious engineering proposal. That ducted fan/wing design is going to be a lot more draggy than a conventional prop at mid to high sub-sonic speeds (which it will need to be able to reach if it wants to compete with existing narrow body airliners like the B737 and A-320) and one of their own slides seems to highlight a serious engineering hurdle that I don’t see mentioned anywhere in their press release or on their website.

      Pushing the aircraft forward is only part of the engine’s job. The other part is powering everything else. Without an engine driven alternator or other source of power all of your auxiliaries will have to be run off the battery as well, and some of those auxiliaries (such as cabin pressurization and AC) are quite power hungry in their own right. Are they planning to have everyone wear oxygen masks for the duration of the flight?

      • cassander says:

        I’m not sure it does, depending on your definition of mid to high subsonic. The P.180 Avanti can be very competitive with jets 100+ miles an hour faster than it on short trips because relatively little those short trips happens at maximum speed. On a three hour trip you’ve got at least an hour of boarding, taxiing, climbing, descending, etc, so losing 20 minutes for much less cost is a pretty good trade.

        • hlynkacg says:

          I’d define “mid to high subsonic” as mach 0.5 and up, which is actually pretty close to the Avianti’s listed max-range cruising speed of 320 kts at 30’000 ft.

          Even approaching (never mind matching) the Avianti’s performance in an all-electric aircraft at this stage would be a massive breakthrough.

  22. digitrev says:

    I think the obvious answer for the dodecahedron is that the Romans clearly played a lot of barbarians in their role-playing campaigns.

  23. John Schilling says:

    You neglected to mention that the lawyer whose pants burst into flames in court, was arguing an arson case. And basing his defense of a (subsequently convicted) client on a theory involving spontaneous combustion.

    The Roman Catholic Church would be a lot more fun if, when they set out to investigate alleged miracles, they looked into this sort of thing rather than yet another cancer remission story.

    • Randy M says:

      I assume he was trying to prove the validity of his defense?
      “Judge, obviously spontaneous combustion happens all the time. Why it could even happen right now.”

      • Jordan D. says:

        He swears he wasn’t, actually:

        “Shortly after beginning my argument, I noticed that my pocket began to feel hot. When I checked my pocket, I noticed that the heat was coming from a small e-cigarette battery I had in my pocket. I noticed the heat was intensifying and left the courtroom as quickly as possible – straight into the bathroom. I was able to toss the battery in water after it singed my pocket open.”

        …but that might just be because he lost the case anyway, and judges can get real particular if you stage pyrotechnic displays in the courtroom without their say-so. We’ll probably never know.

        Sauce

  24. MartMart says:

    “Some of Trump’s crueler policies might make sense in the context of trying to scare people out of illegal-immigrating.”
    Cruelty is almost always terrifying, if one has any chance of ending up on the receiving end. Am I wrong to think that “might make sense” sounds rather supportive in this context?

    • doubleunplussed says:

      Australia has a policy of locking up refugees who arrive unannounced by boat, and generally treating them really awfully in detention centres. The rationale is ostensibly to deter people from coming by boat, because it’s super dangerous and they die at sea a lot.

      The policy came in in 2001 under a centre-right government, was removed by a centre-left government in 2007, and then hastily reinstated and exists to this day (the centre-right party is in government again).

      Prior to being abolished, the number of people arriving by sea had decreased to practically zero. After being abolished the number of arrivals seemed to increase without limit, and people starting drowning at sea again. A Labor MP, Tony Burke, gave an impassioned speech at the Labor conference about wanting to prevent children drowning at sea, and that the evidence was in that the policy worked as a deterrent, and the party voted to reinstate the policy.

      It’s cruel as hell to the people in detention centres, but it does appear to work. It feels awful to say it, but it might be the right utilitarian call to make. People aren’t drowning at sea anymore.

      (and in case it seems like people who would risk drowning might be running from something even worse, this is hotly contested – I’m of the opinion that the people coming by boat are people who have other options but who have the money to pay people smugglers to take them, and are unaware of how dangerous it is – the smugglers obviously have a motive to conceal this).

      Just an anecdote about cruelty possible being useful. There are responses of course, that cruelty sets a bad precedent and you should avoid it even when it’s useful because it’s a slippery slope to people accepting cruelty in less clear contexts. Another response it that the people who most support the policy are racist and support it for racist reasons. This is true, and the centre-right party certainly exploits this for support, but doesn’t mean the policy is a bad one.

      • MartMart says:

        There is little question that cruelty can be effective, but given what I’ve read here about chickens, I didn’t think that the idea that cruelty should be opposed because it is cruel would be particularly controversial.

        The sheer numbers with regards to illegal immigration in the US are, to me, what makes these solutions unacceptable even if they are based on sound logic.

        The illegal immigration population in the US is (supposedly) around 12m. That’s roughly 4% of the population, or 1 in 25 people that we, as a country, plan to forcefully eject from their homes. In the past (as I understand it) when a person was arrested for being in the country illegally, they were typically granted bail prior to having to appear in court. Since there are few immigration courts, it would often take over a year before a person appeared before a judge. One of the first changes made by the administration is that bail would no longer be granted, which means that the plan involved imprisoning 12 million people for at least a year (likely longer, since the increase in volume will result in greater delays). In comparison, the current prison population in the US is a touch over 2 million.

        So the idea of imprisoning 1 in 25 people, to swell the prison population to 8x the current one seems so unimaginably cruel as to be worth opposing, regardless of its effectiveness with regard to reducing unauthorized immigration. Well, to me at least.

      • Machina ex Deus says:

        Cue Nick Lowe.

    • Scott Alexander says:

      I’ve changed it to “interpretable in light of” to emphasize that I’m not saying it’s good, just that it explains Trump’s actions.

  25. gwern says:

    Remember how everyone was talking about how Trump must have inspired an anti-Semitic crime wave among his supporters? And remember how some of the incidents were traced to an anti-Trump socialist working at a leftist magazine? Well, the rest of them seem to be the fault of an Israeli Jew who may have a personality-altering brain tumor. The Atlantic has a pretty good postmortem of the whole affair.

    This is an interesting one because it illustrates a version of Littlewood’s Law of Miracles: in a world with 7 billion people, one which is increasingly networked and mobile and wealthy at that, some extremely odd things can happen routinely. Anti-Semitic attacks are pretty rare in the USA, so it doesn’t require a common cause to account for such rare effects.

    If someone said, ‘I don’t really believe these anti-semitic hoaxes are real in the sense of a bunch of anti-semites have been emboldened by Trump’s election, I think there’s something else going on, like maybe an employee made them up to drum up donations’, you would probably think that was a desperate attempt to avoid the obvious; if they had said, ‘I don’t believe them, maybe they’re actually fake because some schizophrenic or crazy Jew with a brain cancer and a flair for VoIP pranks did them all themselves’ you would definitely think they were desperately coming up with excuses and denying the obvious.

    Yet, there you have it! It is apparently a real thing, that a (self-hating?) Jew halfway across the world in Israel decided to spend all his spare time hoaxing over the Internet dozens of Jewish institutions with hate-crimes in the US post-Trump-election in part because he is an anti-social criminal driven by a brain tumor causing a severe personality disorder. It sounds absurdly implausible and made up – yet, among 7 billion people, there turns out to be at least one evil brain-tumor phreaker Jew.

    As time passes, it becomes increasingly hard to believe rare events at face value, and one has to simply ‘defy the data’. Think about scientific papers. Because of the massive exponential expansion of the academic-industrial complex worldwide, there’s something like 1 million papers published each year; assuming fairly normal research practices of testing out a few configurations on a few subsets and using a few covariates, each paper has somewhere like 1000 p-tests; thus, it is entirely possible to legitimately see a p=1 in 1 billion or p=0.000000005 when the null is true, and if you consider just the most recent set of papers from the past decade or so, you could see p=0.0000000005. Throw in the low but non-zero base rate of fraud, questionable research practices, endemic publication bias, etc, and there’s a point at which no matter how many studies there are on a particular effect, you still don’t have particularly strong belief in it because the data may simply be measuring ever more precisely the level of publication bias in that field rather than the effect you interpret it as (Duhem-Quine, but for biases).

    • It strikes me that if one sincerely believes that Trump is basically Hitler then it’s vitally important that Jewish Americans flee to Israel as soon as possible, and the obvious way to try to cause that to happen is to phone in bomb threats. That would be my first guess as to motivation but I suppose we’ll find out eventually?

      • Nabil ad Dajjal says:

        Or he’s just nuts.

        Remember the delusional Ithaca townie a while back who was convinced that he had shot and killed Donald Trump instead of some random UPS driver? He’s still holding to that even after he was presented with incontrovertible evidence that President Trump was still alive and that the guy he shot wasn’t him.

        I doubt that the Israeli guy had any more of a master plan than this guy did.

        • mupetblast says:

          “Or he’s just nuts.”

          This seems like a wildly insufficient explanation. Why would being just generally nuts manifest in THIS WAY? Or any other particular way, getting away from this specific instance…

          • Spookykou says:

            I think Nabil is not offering up ‘he’s just nuts.’ as a holistic explanation for the behavior, I view it more as an appeal to the outside view. It is very tempting to try and explain the various factors that could have lead to this behavior but it is really unlikely that we actually have good enough information for these predictions to be terribly accurate so leaving it at, the person was a-typical, they do strange things, is ‘safer’.

      • Douglas Knight says:

        Trump is a pretty poor excuse for making bomb threats to New Zealand JCCs or in 2014.

    • Glen Raphael says:

      @gwern:

      If someone said, ‘I don’t really believe these anti-semitic hoaxes are real in the sense of a bunch of anti-semites have been emboldened by Trump’s election, I think there’s something else going on[…]’

      I said essentially that exact thing at the time. The idea that the bomb threats or headstone-topplings were caused by antisemites emboldened by Trump’s election was silly to begin with. The theory underlying that media narrative isn’t merely overly convenient to Trump attackers, it’s also really weird. Near as I can tell the theory behind this phase of the “wave of hate crimes” narrative is based on approximately this set of assumptions:

      (1) There is a hidden cabal of Trump-favoring Secret Antisemite Racists in the US. (We’ll call them TSAR for short).
      (2) The TSAR wants to attack Jews and embolden non-Jews, but when the president ISN’T Trump they are afraid to do so. So they lie in wait, cowering in the shadows, until their moment arises.
      (3) Suddenly Trump wins the primary and then the main election. Even though Trump has never really said anything against Jews (nor has Breitbart) and both Trump and Brietbart are extremely pro-Israel and Trump has Jews in his family and as advisors and Breitbart was founded by a Jew and has Jewish writers and hasn’t really said anything antisemitic, some penumbra of the emanations of Trump’s rhetoric manages to reassure the TSAR that the President is on their side so now it is finally okay to attack Jews. Their moment has arrived!
      (4) Therefore the TSAR decides to BOLDLY STRIKE…by…robocalling fake bomb threats to JCCS around the world. And toppling headstones!
      (5) Even though the TSAR are SO EMBOLDENED BY TRUMP as to do things that normally would be socially unacceptable, they are NOT ENOUGH emboldened as to CLAIM CREDIT for any of these things they do. The world is expected to guess who toppled the gravestones and called in the threats and guess what this is supposed to mean…and somehow this will accomplish the ends of the TSAR.

      Seriously, does this make any kind of sense? At all? When gravestones have been found toppled in the past it’s usually weather or bored teenagers or (in at least one case) a single weird family that simply enjoyed doing it. Absent some kind of MESSAGE clearly establishing that this ISN’T the work of teens, weather, some weird family or a cemetery-related business dispute, how does toppling gravestones serve the interests of the TSAR conspiracy? And how does doing it NOW serve that interest substantially more than would doing so prior to Trump’s election?

      Similarly with the bomb threats – bomb threats can draw positive attention to a specific cause (“Free Mumia!” “Save the gay whales!”) only if the people doing it CALL A NEWSPAPER to say WHY they are doing it, and it only has real force if there are any REAL BOMBS. Surely a real TSAR worth worrying about would be able to muster at least ONE real bomb somewhere, and would have at least ONE specific message to get out.

      So just thinking through the logic, it’s obvious the gravestones weren’t toppled by the TSAR and the bomb threats weren’t called in by the TSAR.

      As I understand it, only one set of headstones has been resolved – the 42 toppled headstones in Brooklyn were due to weather and neglect. The cause of 100 fallen headstones in Philly isn’t yet known. So I hereby offer a wager: if the police ever determine what toppled the other headstones it won’t be the work of the TSAR. That is, it won’t be the work of white, pro-trump not-jewish antisemites trying to attack Jews in the most passive way conceivable by…quietly knocking over headstones in a few places and then not telling anybody why they did it and never doing it any more.

      I don’t know what did knock over those headstones, but I win the bet if it is anything BUT that, as determined by some neutral third party. Any takers?

      • Spookykou says:

        I think the media narrative is a lot simpler than that.

        We had social norms that if the media called you bad you apologies and insist you are not bad, because our society 100% does not tolerate badness.

        Trump does not do that, breaking the social norm.

        This informs people with bad leanings or beliefs or temperaments or what have you, that our society does tolerate some amount of being bad, thus they are emboldened to go be bad.

        It might not be what actually happened, but it hardly seems like a totally outlandish chain of reasoning.

        • Glen Raphael says:

          @Spookykou

          Even your simpler narrative is silly. The logic crucially depends on the premise that your “People with Bad Leanings” (PLBs) uniquely care what the president says. Their leanings would have to be ridiculously finely calibrated to explain the behavior seen. They have to be bad enough that they are naturally inclined to knock over gravestones…but not bad enough as to claim credit for it. They’re only bad enough to do the few pathetic anonymous attacks they do and only bad enough to do those if they think “society” tolerates it.

          PLBs are apparently extremely sensitive to public opinion. But not just any public opinion. They’re not extra-sensitive to what the media says or what the Pope says or what their teachers and friends and neighbors say or what pretty much everybody on the planet other than the president says. Their impression of what “society” thinks is based solely on what the president gets away with saying. If the president can successfully say mean things, then and only then is it okay for them to do bad things. Have I got that right?

          Is there evidence that any actual American human beings look to the president – especially this president – as that sort of moral compass, as their guide to right and wrong behavior?

          (This feels a bit like a religious argument, analogous to the God Of The Gaps. So long as we don’t know how something works, we assume it must be God; so long as we can’t know who committed anonymous vandalism, we assume it must be racist Trump fans.)

          • Spookykou says:

            uniquely care what the president says.

            That is not really implicit in what I said. It can just be any high status important person, rocking the social norms I would think.

            Their leanings would have to be ridiculously finely calibrated to explain the behavior seen.

            As I understood it, and I could be wrong, the narrative was born from the ‘increased hate crime reporting’ from the something something law firm?

            What I heard at the time, was a bunch of relatively minor transgressions that perfectly fit the mold of ‘socially unacceptable behavior’ like drawing swastikas on things, or publicly insulting someone in a head scarf, etc.

            The idea that a high status person disrupting the social norms of political correctness could drive people to believe it is ok to engage in politically incorrect behavior, again, doesn’t seem that outlandish to me.

            Edit: I can agree to the God of the Gaps interpretation, I guess I just don’t see the God of the Gaps as a really obviously failure of reasoning it feels more like really motivated but still plausible reasoning.

          • Randy M says:

            As I understood it, and I could be wrong, the narrative was born from the ‘increased hate crime reporting’ from the something something law firm?

            I assume you are thinking of the southern poverty law center. Their list was based on self-reports and was one sided, not particularly strong evidence, imo.

          • Spookykou says:

            Agreed Randy, but I thought that was the start of the “Trump emboldens hate crimes” narrative?

          • The original Mr. X says:

            PLBs are apparently extremely sensitive to public opinion. But not just any public opinion. They’re not extra-sensitive to what the media says or what the Pope says or what their teachers and friends and neighbors say or what pretty much everybody on the planet other than the president says. Their impression of what “society” thinks is based solely on what the president gets away with saying.

            To steelman this position a bit, it’s not just that the President, personally, can get away with saying bad things, it’s that enough people either liked or were indifferent to his saying bad things to elect him in the first place. I don’t think it’s too implausible to imagine someone thinking “Oh, it looks like we PBLs are much stronger and more numerous than I thought, I guess I can be more bullish about my PBL-ness now.”

          • Glen Raphael says:

            @The original Mr. X:

            “Oh, it looks like we PBLs are much stronger and more numerous than I thought, I guess I can be more bullish about my PBL-ness now.”

            Except that the crimes attributed to PLBs are the opposite of “bullish”. As a way to attack a group, sneaking into one of their graveyards when nobody is around to knock down stones seems like the most secretive, cowardly, ineffective attack imaginable. It’s disrespecting people who are already dead – you can’t get more passive than that! It’s the kind of attack one might do under a perception of being wildly outnumbered, not under a perception of “lots of people agree with me”.

            @Spookykou:

            I guess I just don’t see the God of the Gaps as a really obviously failure of reasoning it feels more like really motivated but still plausible reasoning.

            The trouble with God of the Gaps is that if God really existed, he’d probably be doing at least a few things that are actually detectible and can be traced back to him. If each “miracle” we take a close look at turns out not to be God, it starts to seem silly to attribute any that we haven’t so closely examined to Him. (Unless we start with a pre-existing religious certainty God exists and are only casting about for reasons to support that view.)

            Similarly if Trump-flavored PBLs were real, they’d be detectable in non-anonymous events. The more often we look into their alleged “hate crimes” and the events turn out to be hoaxes and misunderstandings, the less credence we should give the view that any such that we haven’t solidly attributed are due to this particular flavor of PBL. (Unless we start with a pre-existing religious certainty that these PBLs exist and are only casting about for reasons to support that view.)

          • Deiseach says:

            They’re not extra-sensitive to what the media says or what the Pope says or what their teachers and friends and neighbors say or what pretty much everybody on the planet other than the president says.

            To go off on a somewhat related tangent, this is part of what drives me bonkers about “The Roman Catholic Church’s opposition to condoms is causing millions of deaths by AIDS in Africa”. As in this website, which mentions “In addition, in 2009, the Pope Benedict, on a trip through Africa, banned the use of condoms in general. In 2013 the catholic church renewed banning of condoms in catholic schools” as how terribad religious repression is the problem, after first mentioning many significant cultural practices and reasons for the rate of HIV infections such as “The ABC method stands for “Abstinence, Be faithful, and Condom use”. It seeks to promote a different cultural view regarding safer sexual behavior, with an emphasis on fidelity, fewer sexual partners, and a later age of sexual debut” as well as the problem of “cultural stigma …In addition to stigma, there are several other factors medical professionals site as being detrimental to HIV treatment such as male promiscuity and polygamy in some places.”

            So, not alone is each and every member of every nation on the continent of Africa a devout Catholic who obeys the pope, they are selectively devout about their obedience: a married long-distance trucker who is having sex with a prostitute is okay with committing adultery, okay with fornication, but draws the line at using a condom because “Oh no, the church says that’s a sin”? Really?

          • Nancy Lebovitz says:

            “As a way to attack a group, sneaking into one of their graveyards when nobody is around to knock down stones seems like the most secretive, cowardly, ineffective attack imaginable. It’s disrespecting people who are already dead – you can’t get more passive than that!”

            Vandalizing a graveyard is also a way of attacking those who care about the dead, whether the dead where there’s an individual connection or the dead who are members of the group.

            I’ll grant you that vandalizing an unguarded graveyard isn’t risky. It’s not like hitting someone who might hit back or handling explosives.

            I have a notion that dead bodies are viewed as part of a community. That’s why desecrating graveyards or (much more so) descecrating corpses has a large emotional effect.

          • Glen Raphael says:

            @Deiseach:

            To go off on a somewhat related tangent, this is part of what drives me bonkers about “The Roman Catholic Church’s opposition to condoms is causing millions of deaths by AIDS in Africa”

            Huh. I think you’re right – what we’re both perceiving and objecting to is a general argument pattern that looks like this:

            (1) A Bad Thing has happened!
            (2) I have a Villain I like to demonize!
            (3) Therefore, my Villain caused the Bad Thing!

            This argument “proves” that my outgroup (anybody who hates Villain less than me) are bad people who should feel bad for enabling Bad Thing, whereas my ingroup (people who hate Villain about as much as me) are good and virtuous people who should feel good for resisting Bad Thing. So long as there is some however-tenuous connection between the identified Villain and the Bad Thing, the logic works well enough to serve its purpose – it produces that conclusion. Even if there are 50 other less-salient factors that might equally well produce Bad Thing, and even if we’d expect this causal chain we’ve just identified to produce additional measurable outcomes which…never show up.

            Here are a few examples…
            (trump, trump voters :: “hate crimes”)
            (pope, catholics :: AIDS, teen pregnancy)
            (global warming :: any bad weather event)
            (immigrants :: unemployment)
            (illegal drugs :: crime)
            (foreigners :: terrorism)
            (foreign competition :: any economic downturn)

            We could call this process the Casablanca Method. There’s been a crime; rather than investigate the circumstances to determine a cause, we round up the usual suspects and pick one of our existing list of Villains to pin it on.

        • Deiseach says:

          We had social norms that if the media called you bad you apologies and insist you are not bad, because our society 100% does not tolerate badness.

          And who or what makes the media the arbiter of righteousness? The media is just as capable and culpable of going off on witch-hunts as anything or anyone else, see the Satanic Ritual Abuse panic of the 80s and how the media leaped at the chance for sensationalism, without stopping to consider if they were creating a false narrative of villains and victims.

          Besides, I think it’s a bad principle in general that “once you have been accused, your only response should be to apologise”. If you have done something wrong, certainly! If it can be shown you have done something wrong, even if you deny or refuse to accept it, fair enough! But haven’t we seen with the campus rape allegations the rotten fruit resulting from “always believe the accusation and if the accused tries to defend themselves they are in the wrong and only proving their guilt”?

          Again, I think Trump is a blustering vulgarian, but I don’t see anything more here than the media consoling itself that it’s all his fault for not bowing before them that persons unknown are going out allegedly doing bad things.

          As for the Southern Poverty Law Centre, their watchlist of “hate crimes” in the aftermath of the Trump election showed that the majority of complaints and alleged crimes took place in schools – and we’re not talking universities, we’re talking primary to secondary schools. Things like a bunch of kids at a lunchroom table chanting “build the wall” were reported in tones of sorrow and dread by a teacher about how this was down to Trump’s baleful influence (and not a bunch of 14 year olds trying to be edgelords). SPLC rounded all these up into “thousands of hate crimes since Trump’s victory!” and of course the same media happily trotted this out when required.

          • Matt M says:

            Eh, I think there’s definitely a sense that, if accused of something bad, one is expected to adopt a humble and conciliatory tone, even if protesting their innocence, to show that they “take the charges seriously” and that what they are accused of, is indeed, properly serious, etc.

            If you’re a famous or high status person accused of rape, you’re definitely expected to throw in a bunch of quotes about how horrible rape is and how you’re donating a bunch of money to women’s groups in between your assertions of being innocent. You certainly aren’t allowed to say, “I didn’t do it – the accuser is a lying whore, and we all know that false accusations are super common, I’m the real victim here!”

            And that’s basically how Trump has reacted to everything he has been accused of. That’s the norm he is violating. He isn’t properly supplicating himself before his accusers. He isn’t conceding that the things he is being accused of would be serious even if they were true. He is attacking right back at his accusers with the same (or greater) levels of hyperbole and vitriol. The media aren’t used to being treated this way. They have no idea how to properly respond to it.

          • Deiseach says:

            If you’re a famous or high status person accused of rape, you’re definitely expected to throw in a bunch of quotes about how horrible rape is and how you’re donating a bunch of money to women’s groups in between your assertions of being innocent.

            And all that expectation does is promote hypocrisy and insincerity; even a guilty person of high status or fame will have a canny lawyer who says “Right, first thing you do is write a huge cheque for the local rape crisis centre then we get a statement written up for you to read out and stick to the script whatever the hell you do”.

            I don’t think people believe “Oh, Smith Smithson gave a donation and said this is a terrible thing, plainly he didn’t do what he’s accused of!” after all this, I think most people have swung around to “Of course he did it and he’s now trying to buy his way out of trouble” which is just as bad because it hangs the albatross of condemnation around the necks of really innocent people.

            The media getting enraged when someone fails to properly supplicate their accusers, especially when it’s the media doing the accusing, is bad because that helps taint public perceptions. If a journalist or news anchor is going on the attitude “Okay, Smithson is refusing to talk to me or give me an exclusive interview, he’s guilty as sin” and they slant their coverage to reflect that, then they are biasing people.

            But this is the world we live in now and I don’t think you can put the genie of “trial by public opinion” back in the bottle.

          • Jaskologist says:

            “Trial by public opinion” was never bottled to begin with.

          • Robert Liguori says:

            “Trial by public opinion” was never bottled to begin with.

            Agreed. I think there could be a hypothetical detente, in which the media is careful to stipulate that false accusations cause measurable harm and that there should be no rush to judgement, and accused individuals were careful to avoid denigrating the very idea of charges against someone obviously-right-thinking like themselves, and we got increased respect and consideration all around.

            I just don’t think this was ever the state of affairs ever.

      • wysinwygymmv says:

        (1) There is a hidden cabal of Trump-favoring Secret Antisemite Racists in the US. (We’ll call them TSAR for short).

        They’re not hidden. They’re not secret. They’re on reddit. You can go talk to them right now.

      • herbert herberson says:

        I just hope that all the right-wingers who, correctly, see how silly the “emboldenment” argument is when applied to American racists apply that learned lesson to the comparably idiotic debate around terrorism.

        • Randy M says:

          Can you complete the analogy please? What is the action we claim will embolden terrorists? Some things I may agree with, some I may not find analogous and disappoint your hopes.

          For example, electing Obama, probably didn’t embolden terrorists, (although they may have felt the need to sound out the new guy).
          However, if Islamic groups decree that drawing Mohammed is an insult worthy of death, and Western publications thereafter censor all such drawings, declaring their intent is to avoid disrespecting Islam, then I think Islamic groups will quite reasonably be emboldened by it, seeing an ability to affect change with threatened violence.

        • Glen Raphael says:

          @ herbert herberson:
          If you mean the notion that internal dissent should be minimized (and “appearing tough” maximized) because doing otherwise emboldens the enemy, I agree: much of that stuff has a similar structure and is similarly dumb.

          (though I don’t consider myself a right-winger, you might)

        • Conrad Honcho says:

          Isn’t it the left who thinks terrorists are the ones emboldened by things we do, like speak mean words about them? The general right-wing consensus is that Islam as a collective is bent on world domination, so they’re going to attack you no matter what you do. It’s a subjective worldview that everything happens because of the will of Allah, and the will of Allah is that Islam conquers the world. So if they bomb us and then we bomb them, the imams say “ah ha, see they defy the will of Allah! Slaughter the infidels!” and if we hug it out and sing Imagine instead they say “ah ha, Allah has blinded them to their enemies, slaughter the infidels!”

          What is it that right wingers think emboldens terrorists, besides our existence?

          • Spookykou says:

            I have seen examples of left-wing people thinking that some actions we take might embolden/encourage terrorism, primarily wars in the middle east, and torture. I am not familiar with how they feel about saying mean things.

          • Sandy says:

            I am not familiar with how they feel about saying mean things.

            All sorts of liberals and lefties went off about horrible “punching down on marginalized people” was after the Charlie Hebdo shootings.

          • HeelBearCub says:

            Not embolden, motivate.

            Like, if you are in a bar and start talking shit in a loud voice about that no good Such-and-Such University fan over there, you are motivating them to get violent.

            If you do this while in a Such-and-Such-U bar, they may be emboldened.

          • Spookykou says:

            @Sandy

            I don’t really get on social media, but I thought the main stream media was pretty in favor of free speech after the Charlie Hebdo shootings.

            @HBC

            I agree and think that is an important distinction, embolden is a particular kind of call to action that does not seem appropriate here.

          • Loquat says:

            @Spookykou

            I was hanging around a popular SJ-aligned site at the time, and there was no shortage of people on there making comments like “I don’t excuse the killings, but Charlie Hebdo was really racist” and posting links to images (sans context and sans translation, naturally) of cartoons CH published which might look at first glance like racist stereotypes.

          • Conrad Honcho says:

            I have seen examples of left-wing people thinking that some actions we take might embolden/encourage terrorism

            Sure, but unless I misunderstand herbert herberson he was suggesting right wingers are the ones who think left-wing actions embolden terrorists. Some might occasionally say left-wing weakness (singing Imagine after Paris) might embolden terrorists, but that’s the weak man argument. The strong right-wing argument in acting against Islam/terrorists is that it doesn’t really matter what we do, they collectively have a terminal goal of world domination, and will attack us whether we ban Muslim immigration or shower them with hugs.

          • dndnrsn says:

            @HeelBearCub, Spookykou

            It seems like there’s 4 separate things (names are just the first thing I thought of and may sound dumb)

            A-Inherent motivation (townies think Such-and-Such U students are sissies who deserve a beating): group A has a motivation to act against group B without any action on the part of group B.

            B-Backlash motivation (bunch of townies overhear some students talking shit about them – they decide to give them a beating): group B gives group A a reason to act (of course, this might be inherent motivation in disguise – the townies “overhearing” it might be lying, it might be a rumour that blows up, etc)

            C-Own-side emboldening (town sheriff nudges and winks to indicate he wouldn’t much care if some university kids got beaten up): group A has some indicator acting will be successful, unpunished, etc.

            D-“Opposite-side”emboldening (students talking shit get beaten up and admin, worrying about upsetting the local community, make an apology to the townies for the comments – the townies now know they can get away with future beatings): your opponent gives some indication that acting against them will succeed, will not be met with retaliation, etc.

            A given situation will be explained with different combinations by different people.

          • DrBeat says:

            Only left-wingers think that things we do embolden terrorists?

            Sometimes I feel like I’m the only person who remembers the George W. Bush administration. “Don’t question our actions, you’re emboldening the terrorists” was thrown around a lot.

          • herbert herberson says:

            re: DrBeat

            Yeah, I’m genuinely surprised this provoked so much confusion. So, history lesson for the 25 and under set: it used to be a routine talking point that failure to take disproportionate decisive action in response to terrorist attacks, or to fail to posture project strength, would promote terrorist attacks by emboldening terrorists.

            Perhaps the discourse shifting to the point where saying that “the general right-wing consensus is that Islam as a collective is bent on world domination, so they’re going to attack you no matter what you do” doesn’t come off as an obviously fake strawman designed to make right-wingers look genocidal has rendered my various assumptions around the word anachronistic.

        • Machina ex Deus says:

          I just hope that all the $OUTGROUP-wingers who…

          Has any good comment here at SSC ever started that way?

          • herbert herberson says:

            What, I don’t get any credit for affirming $OUTGROUP-ARGUMENT after the ellipses?

    • Deiseach says:

      Look, this is more of Trump’s “cursed with luck” thing. He gets attacked for not being quick enough off the mark to respond with the expected ritual condemnation* and that is interpreted as dog-whistle anti-Semitism, so he says (not unreasonably) that you don’t know it’s the gang of Red MAGA Hat wearers doing this, and everybody clutches their pearls about how very dare he deny that his supporters are all anti-Semite bigots who want to kill Jews and lynch African-Americans and murder brown people in general, and even worse he is blaming the victims by a tortured interpretation of what he said that was taken to mean “He’s saying the Jews themselves are doing this!”

      And then (a) a magazine which was all self-righteousness about how dare Trump deny these are his own racist sexist anti-Semitic homophobic followers doing this get egg on their face when it turns out some of the bomb threats were indeed hoaxes and not alone hoaxes, one of their ex-journalists is responsible (b) some of the toppled gravestones were due to what happens gravestones after years of exposure to weather and soil settling (c) more of the bomb threats were due to an Israeli-American youth who probably has mental issues and possibly his father was involved as well.

      Until we get more news on the Israeli-American boy and his dad, we have no idea what was going on there; maybe Dad thought American Jews were all assimilated and practically goyim and needed to be shaken up or something?

      *I don’t mean to say the threats and fear weren’t serious, I do mean that – as with the aftermath of Princess Diana’s death, when the Queen was on the receiving end of public disapproval for not being visibly in mourning enough to satisfy the emotional circus – there is now an expectation that whatever happens, from a natural disaster up, in whatever part of the world, that politicians and leading figures will come out at once with a speech of condemnation and support and apology and mourning. Not even “let’s wait till we know what happened”, no, within five seconds of the news you’ve got a TV camera in your face and “what is your statement on this dreadful tragedy and what are you going to do about it?”

  26. Some Troll's Serious Discussion Alt says:

    No HWNDU link to pair with the link on polarization? Saying we won’t be divided is apparently a devise enough statement that you can’t do it even after fleeing the country.

  27. Jack says:

    There was a spike in recorded hate crimes following Brexit. Explanations include: a) some kind of random that we all noticed because of the attached narrative (ie what SSC implied for the anti-semitic threats); b) hate crimes increased because people were “emboldened”, or perhaps more likely the salience of race increased; and, c) hate crimes were the same but reporting and policing increased. The c) explanation could be a story of people being extra careful out of the fear that hate crimes would increase and a(n apparently ineffectual) desire to make racialized people feel safer. Anyway, I’d like to know more about what it is. The fact that some randos were responsible for the post-Trump anti-semitic threats doesn’t tell me much about the possibility of this as a more general phenomenon.

    • Conrad Honcho says:

      Also, d) people fake hate crimes for sympathy or to disparage their political opponents, which seems to be the case here. Same thing with the black church that was burned down in Mississippi with “Vote for Trump” scrawled across it (burned down by a black member of the church) and…to be honest just about every “hate crime” of the form “anonymous person scrawls hate message in the middle of the night” I can think of in recent times.

      Just ask “whose propaganda purposes does this serve?” When the church was burned down every liberal on my Facebook feed was sharing the story with “look how awful Trump people are!” Not a single conservative was sharing it around saying “haha, yeah, this is great!” Why would they? It does not, at all, serve their purposes of gaining political support from blacks or non-racist white people.

      • tmk says:

        > Just ask “whose propaganda purposes does this serve?”

        If you only apply this to things you don’t like it will become a very effective shield against anything that challenges your beliefs.

      • quarint says:

        Just ask “whose propaganda purposes does this serve?”

        And that’s how you get to the end of Truth. Because no one can be right, ever, otherwise it just serves their propaganda.
        Yeah, I’d rather simply ask “is this true ?”.

  28. grendelkhan says:

    I said a few months ago that Trump was considering choosing some exciting candidates for the FDA who might be true libertarians and really change things. Needless to say, Trump did not do that.

    This is uncomfortably isomorphic to the face-eating leopard thing. Our host may not have voted for this sort of thing, but plenty of people did. Aren’t rationalists supposed to be better at decision-making? And yet plenty of us wound up doing the same ‘ignore the things he says that we don’t like, latch onto those we do, be very surprised when it turns out he was lying about different stuff than we thought’ dance. It’s very disappointing.

    • Scott Alexander says:

      I don’t think anyone voted for Trump because they expected him to choose interesting FDA directors. It was a random rumor that he was considering some people nobody had expected, probably because of Peter Thiel.

      • grendelkhan says:

        I think some people, at least, voted for Trump because they expected him to do positively disruptive things, which they interpreted according to their own hopes and dreams. I hate to come up empty with cites, but I remember seeing a weird analogy between coal miners excited that he’d bring back their jobs and /pol/ folks excited that he’d get us to Mars or whatever.

        I remember seeing people excitedly guessing that he’d have a great replacement for the ACA which would reduce costs and expand coverage because of his wonderful negotiating abilities, that he’d put in Congressional term limits because of the drain-the-swamp thing, that he’d get a massive infrastructure bill through Congress, that he’d clear the lobbyists and bankers out of the Executive Branch, and so on, and so on. Maybe this was post-election grasping-at-straws, but people should know better at this point.

        • Jordan D. says:

          In fairness, though, the more intellectual Trump supporters were always mostly focused on the positive economic effects of deregulation. Unfortunately, the political will for deregulation of the FDA-overseen fields is much less than the political will for deregulation of, say, EPA-overseen projects, but there still seemed like a real chance that the right person at the right time could talk Trump into supporting an FDA-abolitionist in the same way that he did for the EPA and Education.

          I’m very much the opposite of a Trump supporter, but he did seem rather more likely to do something good as far as the FDA went than the other candidates- the Federal Government is a large enough entity that even a terrible president would have to have a few policies that are better than a good president. But it looks like it is not to be.

        • wintermute92 says:

          In person, I know several people who supported Trump and are now upset on basically these grounds. Though, none of them are rationalists – I think you might be describing a real thing but assigning it to a group that did very little of it? (I did see some rationalists enthusing about this stuff after the election, in a “maybe it’s not 100% terrible” sense. I think that can probably be excused since they didn’t ignore the bad.)

          As for the people I have seen do it, there was definitely a lot of cherry-picking. Since Trump didn’t give a shit about consistency, he could say diametrically opposed things and trust people to choose the one they liked. But I also know some people who made this decision consciously – they liked Trump on a basis of “He’s promised X, Y, and Z. X and Y suck, but I think he’s most likely to follow through on Z and Z is good.” So those people are annoyed that he’s focusing somewhere they don’t like, but they didn’t blindly overlook the other points. They just guessed wrong, which is not blameless but is different.

    • herbert herberson says:

      I dunno. You could count on the leopards eating people’s faces, because whatever the name on the ticket said about how your face was gorgeous, the greatest face, he still surrounded himself with a lot of very hungry leopards. In this case, Scott based his hopes on the fact that there was a very high-profile Trump supporter seeking the exact things he wanted, and it wasn’t obviously contrary to the interests of the leopards. I don’t think that limited optimism was unreasonable, and it’s sad to see it thwarted.

  29. HeelBearCub says:

    I said a few months ago that Trump was considering choosing some exciting candidates for the FDA who might be true libertarians and really change things. Needless to say, Trump did not do that. Big Pharma is thrilled; I hope people think long and hard about the significance of an industry deeply relieved that they are not going to be deregulated. Watch for Gottleib to use vaguely libertarian rhetoric while continuing the crony capitalist system, drug prices to continue to rise, and liberals to declare this proves that libertarianism always fails.

    This paragraph makes me angry at Scott.

    It’s basically a bunch of boo-lights strung together.

    • Virbie says:

      I mostly agree with you, but to be slightly more charitable: the reason it didn’t strike me as such is because he has written loong posts on the subject of what exactly he thinks is wrong with the state of drug regulation. The reason I know this is that I’ve read that post, but then I’ve read every post for the last 3 years (plus a healthy dose of the archives). Is it super-unreasonable for him to assume that someone reading his links post has all that context (or at least target it to those who do)? I dunno.

    • Deiseach says:

      Well, Scott is disappointed and he’s entitled to his feelings. I’m not so sure this is An Unmitigated Disaster, as I’ve said before it’s the “honey and vinegar” principle. If the big pharma companies are feeling relieved because whew, this guy is sensible, no crazy swivel-eyed mad reformers running around with axes here! then there’s a better chance he could get reforms through since they’re not all on the defensive and “whatever he says, we say no!” as with a Truly True Libertarian pick.

      Granted, the guy may want and be perfectly happy with the status quo, but we won’t know one way or the other until he gets his backside in the boss’ chair and starts signing off on things.

    • FacelessCraven says:

      As the days go by, I keep a sort of tally in my head of how things are going in politics, aimed at deciding whether voting for Trump was a good idea or not. That paragraph was one of the notable ticks in the “bad idea”n column in the last few months.

    • wintermute92 says:

      I hope people think long and hard about the significance of an industry deeply relieved that they are not going to be deregulated.

      I think this at least is a genuinely interesting point? It’s negative, sure, but it’s intriguing that many industries fervently fight for deregulation while a handful fight against it. We all know the basic principle here – companies like protectionist government intervention, but not restrictive intervention – but seeing which way companies push is a decent guide to the state of the industry. Pharma’s terror of deregulation even when it could expand short term profits hints at how uniquely beneficial their regulations are.

      • HeelBearCub says:

        @wintermute92:
        I’d submit that that this is far too simplistic.

        Certainly regulatory capture is a thing. I won’t (I hope) ever make the argument that we can ignore regulatory capture as a driver of regulation.

        But it has never been the only reason, nor even the primary reason why regulations exist, even government regulations. Coordination problems are hard to solve from within an industry. It takes outside pressure to do this.

        Implicit in Scott’s “argument” here is that the mere fact that Pharma would like the FDA to continue to actually regulate the industry is that the only reason they would possibly do so is because it protects them from competition and increases their profits. That there is little of value in the regulations.

        Whereas what I see as the most likely reason is that they would prefer not to have to compete against companies that don’t give a shit whether their products actually work.

  30. Douglas Knight says:

    Why will the federal actions matter to IRBs? As the article said, it was all creeping overreach in the first place.

  31. Le Maistre Chat says:

    Scott, Gor fandom is not “mainstream” BDSM. While BDSM is predominantly female submissive, it’s also overwhelmingly Blue, pro-LGBT and has various shibboleths to avoid any thought that women want to be dominated.
    Tarnsman of Gor was published in 1966 as one of many attempts to capitalize on the renewed popularity of Edgar Rice Burroughs’ planetary romances. It’s a fun story where the protagonist, mild-mannered northeastern college professor Tarl Cabot (it may be relevant that the author’s day job is a philosophy professor in New York) gets abducted by a UFO, where he meets his father who went missing years ago and travels to a planet 180 degrees from Earth in the same orbit. It’s a world with advanced medicine and construction but Fantasy Gun Control and cultural practices the author ripped off from antiquity, like slavery.
    This was around the time straight BDSM took off, and word got to the author that readers were latching onto the slave subplots. By the third book (which reveals the aliens who rule the planet), he’d introduced the part everyone hates but Goreans: long-winded asides about sex roles by the narrator. Their cult following allowed the series to survive the neo-Burroughs fad and groups of men and women LARPing Goreans started showing up at both BDSM and SCA events, where they were turned into an outgroup.

    (Yes, Chat just lectured y’all about stuff that happened long before she was born.)

    • Anonymous Bosch says:

      has various shibboleths to avoid any thought that women want to be dominated

      Wait what?

      • Le Maistre Chat says:

        At a typical dungeon party, there will be 0-1 men getting dominated, all the rest being women. You are not allowed to comment on this: no reasoning from the specifics “Jill wants to be dominated, and Anne, and Mary…” to “women want to be dominated.”
        It’s just basic politeness, as anti-sexism is part of the party-goers’s identity. Goreans make themselves an outgroup by being sexist.

        • Anonymous Bosch says:

          At a typical dungeon party, there will be 0-1 men getting dominated, all the rest being women. You are not allowed to comment on this: no reasoning from the specifics “Jill wants to be dominated, and Anne, and Mary…” to “women want to be dominated.”

          It’s just basic politeness, as anti-sexism is part of the party-goers’s identity.

          Oh, I guess that seems fair enough.

        • tmk says:

          Or it is just a consequence of individualism. Declaring that “women in BDSM like to be dominated” doesn’t give you anything useful, and would excluded the women who don’t and men who do. As BDSM is already breaking social norms, it makes sense that they value the right to break norms.

          • Aapje says:

            Declaring that “women in BDSM like to be dominated” doesn’t give you anything useful

            It seems valuable for a man who wants to be dominated to realize this and not blame his difficulty in finding sexual partners on himself. Similarly, it seems healthy for a woman who is into dominating to realize that her ease of finding sexual partners is not (merely) due to her desirability, but also because she is offering a rare commodity.

            In general, knowing (rough) base rates is useful in decision making.

          • AnonYEmous says:

            It gives you an immensely useful heuristic, though.

            I mean, you’re literally advocating not noticing obvious facts because it might offend people who…wait for it…fall out of the norm. Because apparently, even for people who break norms, breaking gender norms on the basis of individualism is too much, and instead gender norms must never be referred to and thought of as illegitimate. Even as they stare you in the face.

          • Besserwisser says:

            This reminds me of one of the reviews of this book mentioned above with gender issues and all that. The gist of it was, if we have biological tendencies, we can deal with that. We can’t deal with something we won’t even acknowledge.

          • tcheasdfjkl says:

            I find it annoying that this subthread assumes the only two options are “women want to be dominated” and “you’re not allowed to notice patterns”.

            I want to notice patterns, and I also don’t want anyone to say “women want to be dominated” because that is a gross and dangerous distortion of the actual pattern.

            “Being a sub is much more common among women than men, vice versa for being a dom” is good information.

            “Women want to be dominated and men don’t” is, in I think the most reasonable interpretation, a rather stronger statement than that. It’s almost “all women want to be dominated, and no men do”, or at least “most women want to be dominated”. This is not really something you can conclude by seeing that among those who want to be dominated, most are women.

        • Ratte says:

          One has to wonder at the health of a subculture whose standards of behavior are built around not noticing things, especially when that subculture is so open and honest about other things that run counter to the image they’re trying to present (e.g. the sub is the one actually in charge, being a dom is hard work, doing it as a way of life isn’t healthy).

          Looking at it from the perspective of the gay subculture – which is quite blunt about the lopsided ratio of tops vs. bottoms, for example – these taboos seem silly and counterproductive. Women, generally, have preferences. Are men not allowed to notice that?

          • Le Maistre Chat says:

            I think it would be a healthier subculture if men (and women!) were allowed to notice that without getting ostracized.

          • vV_Vv says:

            One has to wonder at the health of a subculture

            Oh, the BDSM culture is not healthy? Who could have thought? /s

            Women, generally, have preferences. Are men not allowed to notice that?

            Try and say in public that women have tend to have a preference for wealthy and dominant men and gauge the reaction.

            As for the BDSM culture in particular, I don’t know, maybe they got the SJ mind infection early, maybe because they tend to attract the kind of people (e.g. cluster B personality disorder types) that tend to be SJWs, or maybe because the SJWs were able to successfully infiltrate them by playing their “inclusivity” card and the BDSM culture was a natural and soft target.

          • Nyx says:

            > Looking at it from the perspective of the gay subculture – which is quite blunt about the lopsided ratio of tops vs. bottoms, for example – these taboos seem silly and counterproductive.

            That’s because there’s no political implication; the gay community is all men. If there were lots of bottoms and few tops, that wouldn’t really mean anything other than “it’s quite hard to get your preferred position as a bottom and quite easy as a top”. Whereas it’s not that hard to draw an inference from “women like to be submissive in the bedroom” to “women like to be submissive in places other than the bedroom”.

      • Skivverus says:

        Steelmanning attempt: insert “inherently” after “women”, and/or “non-universal, but in big enough subsections to feel universal” before “shibboleths”.
        Or at least, that’s sufficient qualification to reconcile the statement with my own experience: have definitely seen such “shibboleths”, frequently using the word “Twue”, but (either more recently, or as I’ve just seen more) they are not uncontested.

    • martinw says:

      I read on the Slashdot thread about this case that a large majority of the Gor community is female, is that correct?

      Amusingly, that would imply that if the Drupal project were to ban all the Goreans from its midst, it would, statistically, cause its ratio of female to male developers to go down.

      • Le Maistre Chat says:

        I can’t testify to “large majority”, but 50/50 or women-majority. In my experience, there’s also an outer circle of women with a sexism/rape/slavery kink who pick up the books as porn rather than SF and then stop because the narrator is so annoying.

    • Nabil ad Dajjal says:

      Scott, Gor fandom is not “mainstream” BDSM. While BDSM is predominantly female submissive, it’s also overwhelmingly Blue, pro-LGBT and has various shibboleths to avoid any thought that women want to be dominated.

      This seems like a misleading definition of mainstream to me.

      The BDSM Community™ is heavily activist. But you don’t need a license to tie someone up and most people into bondage don’t go to dungeon parties.

      For example, it’s safe to say that Shades of Grey readers outnumber the organized kink scene by at least a hundred-to-one. If only one in fifty of those women tries out spanking and likes it, they’ve become the “mainstream” of BDSM by a comfortable margin. And remember that this happens roughly once a generation: Christian Grey and the Sheik could be brothers. The overwhelming majority of people actually engaging in bondage will consist of enthusiastic “amateurs” who couldn’t care less about gender politics.

      • Le Maistre Chat says:

        For example, it’s safe to say that Shades of Grey readers outnumber the organized kink scene by at least a hundred-to-one. If only one in fifty of those women tries out spanking and likes it, they’ve become the “mainstream” of BDSM by a comfortable margin. And remember that this happens roughly once a generation: Christian Grey and the Sheik could be brothers.

        That’s a good point: disorganized BDSM is probably larger than the organized kind at any time, and perhaps an order of magnitude so when inspired by such a romance novel.

      • Machina ex Deus says:

        But you don’t need a license to tie someone up…

        Yet — Growth Mindset!

  32. xXxanonxXx says:

    Random832 beat me to it, but I remember the Roman dodecahedrons coming up in conversation years back and the consensus was they were for knitting gloves. A guy 3D printed them and demonstrates here

    • Le Maistre Chat says:

      And the Roman d20 was probably used for divination.

      • xXxanonxXx says:

        I’m sticking with the theory that gladiators used them in games where they pretended to be 20th century nerds.

        • random832 says:

          They’re printed with letters of the alphabet – This article simply says they “don’t appear to be either Arabic or Roman numerals”, but I don’t know if it hadn’t been figured out yet (other examples are clearer) or the article is going for sensationalism, but on other examples they’re much more clearly greek letters, in alphabetical order clockwise around the die (i.e. with ΑΒΓΔΕ positioned at the “top”, the middle ten and bottom five are subsequent letters until Υ). Which is also not a set of letters that lines up with using the Greek numeral system. So my theory is that the game of Scattergories is far older than anyone knows.

          EDIT: Removed my interpretation of this particular image, because I can’t account for the presence of a letter that seems to be Ρ in the middle tier where Π would be according to the layout described above. Unfortunately there don’t seem to be any rotatable 3D views of this specific example.

  33. suntzuanime says:

    There’s something kind of crazy about starting from nothing in 2009, getting valued at $600 million in 2015, and shutting down in 2017, but I guess that’s business. Or something.

    Nothing crazy about it. Startup valuations are basically a real-life game of Deal or No Deal, where you have a briefcase with an unknown amount of money in it and you iteratively get more evidence about the contents of the briefcase, which causes your valuation of the briefcase to change without changing the actual value. Basically in 2015, you had a briefcase that might have been the $1 briefcase, the $25 briefcase, or the $1,000,000 briefcase. Then between then and 2017, it was revealed not to be the $1,000,000 briefcase.

  34. bean says:

    I found the source article on the ramjet thing, and it provides some interesting information. Most notably, this was a prototype solid-fuel ramjet. (The USN does not like liquid fuel on missiles at all.) It was not an operational missile. It was a testbed.
    While there are important lessons about bureaucracy to be learned from this, the claim that they could have a missile to the fleet in 3-4 years is pure nonsense. A technology development testbed is of interest, but it’s not an operational missile. That commercial model rocket booster is not tested to withstand a shipboard fire. There’s no guidance, and goodness knows what sort of problems that will turn up. Even more importantly, what would this hypothetical missile do? We don’t really need a long-range ASM right now, and Tomahawk is perfectly adequate for land attack.
    For that matter, there are already solid-fuel ramjets in active military service. I’ve heard that a similar system might be in development for the next-generation AMRAAM. So it’s possible that this is where they’re going with this. But I expect it to take more than 3-4 years.

  35. Christopher Hazell says:

    Those reviews of Testosterone Rex themselves strike me as being not so great themselves. Greg Cochran’s is particularly not careful:

    “Teenage boys routinely beat professional female athletes, as when the Newcastle Jets U-15 team recently defeated Australia’s national women’s soccer team 7-0.

    There are psychological differences as well. Boys prefer rough-and-tumble play, girls prefer ‘intimate theatrical play’. “

    So, uh, what exactly accounts for Australia’s national women’s soccer team? Perhaps they’re all really men. No, wait, that wouldn’t explain why the Jets beat them. Maybe they play a more “intimate, theatrical” version of soccer? I know! Maybe every single member of the team has congenital adrenal hyperplasia.

    Or perhaps Cochran, (And quite possibly Fine as well) don’t actually have a good grasp on what they’re arguing about?

    Like, one thing that is common in all three reviews is that none of them explain why our current social set-ups exist, even when they don’t, to my ignorant eyes, seem to actually support the kind of reproductive strategies that are supposed to drive evolution.

    I mean, it seems to me that if the optimum strategy for men is to mate with many women, and the optimum strategy for women is to mate with one strong man who will support her in pregnancy, then we should tend towards polygamy, yeah? You math whizzes and scientists can tell me if I’m off, but that would be an arrangement which would seem to satisfy the needs of the male who wants to spread his genetic material, and the choosy woman who wants one very fit male.

    So what explains monogamy, which would seem to favor women’s reproductive needs over those of men?

    Cochran brings up this example:

    Here’s the practical example: suppose some dude has a wife and a girlfriend next door. Suppose he has intercourse 50 times with each of them over a year – both are probably going to have a kid, while with just the wife , he would have had one. 2 > 1. Am I getting too abstract here?

    Well, no, you aren’t getting abstract enough: Why doesn’t the dude just have two wives? Suppose the mistress is unmarried: Social stigma, shame, or general hound-doggishness might prevent the man from acknowledging his offspring. What is the evolutionary benefit to being the girlfriend next door, rather than the second wife?

    Or, here’s Cochran again:

    Fine goes on to criticize the ‘man-the-promiscuous-horny-hunter/woman-the-choosy-chaste-gatherer. It can’t be the case that men want sex more than women – why, if that were true, prostitution would exist.

    I mean, that’s a sick burn, except… Isn’t prostitution a terrible reproductive strategy? Like, for both parties? The woman is not being choosy at all, and in fact doesn’t want to get pregnant, and if she does her career will take a hit and she knows the man won’t support the offspring. The man knows the encounter probably won’t produce a successful offspring, and probably won’t support the offspring if it does produce one. Both parties will take conscious action to ensure that the encounter doesn’t lead to a viable offspring.

    So, evolutionarily, where did it come from? Maybe that one is social conditioning? So how do we know that differences in male and female play aren’t just as much products of social conditioning as prostitution?

    I think it would be insane to say that there aren’t sex differences in humans, but that’s not the interesting or open question. The open questions are things like “Which sex differences are attributable to what factors? Which ones are environmental? How many members of a given sex exhibit a stereotypical trait (e.g. when you say “boys prefer rough and tumble play” does that mean 55% of boys gravitate towards that kind of play for 60% of their playtime, or that 90% of boys gravitate towards that kind of play for 90% of their play time?)?

    • Nabil ad Dajjal says:

      I mean, it seems to me that if the optimum strategy for men is to mate with many women, and the optimum strategy for women is to mate with one strong man who will support her in pregnancy, then we should tend towards polygamy, yeah? You math whizzes and scientists can tell me if I’m off, but that would be an arrangement which would seem to satisfy the needs of the male who wants to spread his genetic material, and the choosy woman who wants one very fit male.

      So what explains monogamy, which would seem to favor women’s reproductive needs over those of men?

      This is actually a very interesting question! I would like to know myself.

      A while back I* linked to this paper which suggested that widespread polygyny might explain why we see much less diversity on the X chromosome than one would expect. From the evidence it seems that many fewer men than women reproduced in our evolutionary history, and polygyny seems to have at least contributed to that.

      But in Western culture, polygyny isn’t the norm. How did that happen?

      *I’m Dr Dealgood, I just made a new account.

      • Creutzer says:

        You have a type there, I think, you mean Y chromosomes.

        I found two proposals, not specific to humans, in a quick search. I’m neither an expert nor mathematically sophisticated, but I somehow doubt that these are on the right track for humans. Monogamy in humans looks to me very much like a social cooperation technology. (Between unattractive and attractive men, as something like “you guarantee us a woman, we’ll put in work for the tribe” or “you don’t take all the women for yourself, we won’t plot to kill you”. I can’t quite see the cooperation formulation from the female side. Women get a better guarantee of resources + a chance at good genes via adultery, maybe that’s basically women cooperating with unattractive men?) And precisely because it’s a social cooperation technology, you expect to retain a certain rate of defection, hence the usual picture of monogamy + adultery.

        • The original Mr. X says:

          I can’t quite see the cooperation formulation from the female side. Women get a better guarantee of resources + a chance at good genes via adultery, maybe that’s basically women cooperating with unattractive men?

          Monogamy means that your mate’s resources all pass to your own offspring, not to those of one of his other wives. In polygamous societies, the usual stereotype seems to have been that the various wives would spend all their time scheming against each other to promote their children at the expense of the other wives’; monogamy removes the need for this.

        • Deiseach says:

          I can’t quite see the cooperation formulation from the female side.

          *gently rolls eyes* You young’uns!

          Read the synopsis of the play Sive from 1959 about “hmm, whyever would young attractive women marry older unattractive men?” Co-operation wasn’t necessarily a huge part of it in many cases:

          The story is centred on a young eighteen-year-old girl, Sive, who is illegitimate. She lives with her uncle Mike, his wife Mena, and Nanna who is Mike’s mother. A local matchmaker Thomasheen Sean Rua decides that Sive should marry an old man called Seán Dota. Seán is rich but old and haggard. Thomasheen convinces Mena to organise the marriage of Sive to Seán Dota. She and her husband will receive a sum of two hundred pounds as soon as Sive marries Seán Dota. Mike is originally unwilling to have Sive married to a man so much older than her, but Mena convinces him otherwise. …Sive is distraught but is forced to do the will of her uncle and his wife.

          Happens with men too, to be fair; in one of the Irish language text books where people from the 1920s are telling their life stories, there is the story of a young man in love with a young woman whom he wants to marry, but his father makes a match for him with another woman. On the wedding day, at the reception afterwards, the man who is not participating in the fun is chivvied by his friends and relations to join in, so he sings a ballad about a similar situation of a lost love and a forced marriage (which must really have been a great start to married life for his wife).

          (And there are oodles of songs and ballads in English and other languages about exactly this: we were in love but my cruel parents disapproved of him and forced me to marry a rich old man and we both died/we were in love and he promised he’d marry me but I am poor if beautiful and his mother made him marry a rich if ugly girl and everyone dies/I really fucked this one up didn’t I when I married the rich ugly girl instead of you my true love).

          My own mother told me a story about a cousin of hers who was married off by her widowed mother, who went into town to the cattle mart, met a man there buying cows, and told him “I’ve a nice little heifer at home”. The man was older (I don’t know how much) but hey, he had land and money so he was a respectable match.

          And of course my mother’s father, who Married For Love and was disinherited by his mother who wanted him to marry another woman she considered more suitable 🙂

          “Marriage for romantic love and of free choice” is really a modern and indeed mostly Western conception.

    • martinw says:

      So, uh, what exactly accounts for Australia’s national women’s soccer team?

      If I said “men are taller than women” would you consider it a devastating takedown if someone came up with 11 examples, out of a population of 23 million, of women who are taller than the average man?

      I mean, that’s a sick burn, except… Isn’t prostitution a terrible reproductive strategy?

      Yes. Mother Nature figured that actually making men desire to have lots of children was too much work, so she came up with a sloppy hack: she made men desire to have lots of sex, and she figured that was basically the same thing. Which actually worked fairly well until not very long ago, when we figured out how to create reliable contraception.

      • Christopher Hazell says:

        If I said “men are taller than women” would you consider it a devastating takedown if someone came up with 11 examples, out of a population of 23 million, of women who are taller than the average man?

        Well… Yeah. In the very specific sense that “Men are taller than women” doesn’t actually really convey anything very specific. What do the distribution curves look like and how closely do they overlap?

        When you say “Men are taller than women” that could mean all kinds of things; from “The tallest woman in the world is shorter than the shortest man in the world” to “The tallest woman in the world is half an inch shorter than the tallest man in the world and the average woman is half an inch shorter than the average man”.

        If we’re designing something like, say, a desk or countertop that we expect to be used by both men and women, then the difference matters.

        A lot of times people talk about designing curricula around the differences between boys’ and girls’ learning styles. If we’re doing that, it really does matter if 55% of boys thrive in the boys style classes or if 95% of them do.

        Yes. Mother Nature figured that actually making men desire to have lots of children, was too much work, so she came up with a sloppy hack: she made men desire to have lots of sex, and she figured that was basically the same thing. Which actually worked fairly well until not very long ago, when we figured out how to create reliable contraception.

        None of which explains why the women participate. But also… it undercuts the whole entire evolutionary premise: Now you’re saying that our sexual arrangements aren’t based on maximizing reproductive success, but on maximizing something else.

        For example, you might say that the men are trying to maximize the amount of sex they have and the women the amount of money, but now your whole explanation has become completely unmoored from reproduction, and we have the right to ask:

        Is the man’s desire to maximize sex just as much a feature of social forces as the woman’s desire to maximize money? What makes us think it is or isn’t?

        • The original Mr. X says:

          None of which explains why the women participate. But also… it undercuts the whole entire evolutionary premise: Now you’re saying that our sexual arrangements aren’t based on maximizing reproductive success, but on maximizing something else.

          It’s maximising something else as a means to maximising reproductive success.

          ETA: By way of analogy, think of a school system that wants to determine how much its pupils are learning to help educate them better, so it sets standardised tests and collects data about how well everybody does on them. Now, the system isn’t directly testing learning, and it’s quite conceivable that a strategy for improving students’ tests scores might not increase the amount they actually learn; nevertheless, it would be fallacious to infer that the entire system is really about maximising test scores, not learning.

          Is the man’s desire to maximize sex just as much a feature of social forces as the woman’s desire to maximize money? What makes us think it is or isn’t?

          Most animals seem to want to have sex quite a lot; is that due to social forces? Or did humans somehow lose this biological drive found in the entire rest of the animal kingdom, before coming up with a load of social pressures which just so happen to exactly mimic the biological drive they’d just gotten rid of?

          • JayT says:

            None of which explains why the women participate. But also… it undercuts the whole entire evolutionary premise: Now you’re saying that our sexual arrangements aren’t based on maximizing reproductive success, but on maximizing something else.

            Women that are prostitutes are a very, very small small part of the population, and as such I don’t think you can make very good generalities based off of them. Add in the fact that prostitutes are usually quite different from the median woman in other categories (eg, income, opportunities), it makes them even more of an outlier.

            However, on the other side of the coin, the number of men that have paid for sex is a fairly significant portion of the population, and is not confined to specific types of people.

          • Gobbobobble says:

            the number of men that have paid for sex is a fairly significant portion of the population

            That’s rather surprising. Might just be typical mind fallacy on my part? Still, I’d be curious what the actual numbers are, if you’ve got em.

          • Spookykou says:

            It was my understand, although I have no source and might be totally off base, that it used to be much more common for men to frequent prostitutes but with sexual liberation for women, the need has declined.

          • JayT says:

            I don’t want to look up the numbers while I’m at work, but off the top of my head I believe the rate in the US is something like 20%-25%. I believe it is much higher in Asian countries.

            I believe that Spookykou is correct to say that the numbers have fallen dramatically. It used to be a thing that a father would take his son to a brothel to “become a man”, but I don’t think that is common anymore. Also, fewer men serve in the military nowadays, and that is a common time for men to visit prostitutes.

          • Gobbobobble says:

            Huh, TIL. Thanks guys.

          • Mark V Anderson says:

            According to this, 15-20% of US men have used a prostitute at least once. I saw some other sites that said 14%. Other countries have much different percents.

        • Besserwisser says:

          The thing with different learning styles is that the “boy’s” ones we get both boys and girls doing better as with the “girl’s” ones where girls do slightly worse but boys do massively worse. It’s not even a question how many boys do better here anymore.

          While risking to sound like the libertarians here, you don’t seem to understand how markets work. The entire reason a much bigger group of women than men can offer sex for a profit means most women tend not to offer sex to an equal degree as men do. Money is a compensation for an act most of them would otherwise not do. Why people accept money for work is almost a philosophical question but you don’t get to decide genetic origins of prostitution are nonsense because we have no genes specifically for money.

        • martinw says:

          If you want to debate how large the difference between men and women is in various areas, and how much of that difference is biological, then sure, all kinds of interesting discussions could be had about that. But that’s quite different from your original claim that Cochran “[doesn’t] actually have a good grasp on what [he’s] arguing about” because the existence of female soccer players proves that there is no difference between boys’ and girls’ playing styles.

        • AnonYEmous says:

          None of which explains why the women participate.

          Orgasms. This, by the way, is how the body makes people want to have sex.

          But also… it undercuts the whole entire evolutionary premise: Now you’re saying that our sexual arrangements aren’t based on maximizing reproductive success, but on maximizing something else.

          That’s because natural selection doesn’t consciously do things. Organisms which successfully pass on their genes, have children with those genes, who then have children with those genes if they successfully reproduce. Eventually, genes that don’t reproduce are weeded out and those that do are an ever-larger percentage of the population.

          In other words, men and women are optimized to have kids and make sure those kids survive to have kids, but not necessarily in a direct way. For example:

          you might say that the men are trying to maximize the amount of sex they have and the women the amount of money

          Men are trying to maximize the amount of sex they have, balanced with the survival of their children, which must come with the knowledge that it is, in fact, their child.

          Women are trying to maximize the fitness of the genes of the men they mate with (so are men, actually), and must balance that with ability / desire to provide for the child and protect the mother.

          And speaking personally, I don’t think my orgasms are socially constructed. Even if everything else is, that already puts a galactic-sized hole in your argument.

        • spN44p8 says:

          Well… Yeah. In the very specific sense that “Men are taller than women” doesn’t actually really convey anything very specific. What do the distribution curves look like and how closely do they overlap?

          That really isn’t a refutation of the premise. You’re just saying they aren’t being as specific as you would like in that one sentence.

          None of which explains why the women participate.

          Most women don’t participate, and those who do generally aren’t the ones with the most options in life. Most girls don’t dream of becoming prostitutes. There was a large increase in prostitution in Germany after WW1 because of women who were widowed by the war, but there were few other career opportunities available to women at that time in Germany. It is sometimes the only option to survive.

          But also… it undercuts the whole entire evolutionary premise: Now you’re saying that our sexual arrangements aren’t based on maximizing reproductive success, but on maximizing something else.

          Of course evolution is not omniscient. It works on behavior mainly by changing the brain. If you observe sea turtles eating plastic bags, it doesn’t mean that eating plastic bags maximizes their evolutionary fitness. “Eat the white floaty things” was a heuristic that worked well historically for maximizing evolutionary fitness, so sea turtles today have an instinct to do that.

          Having sex and acquiring resources were heuristically good things to maximize evolutionary fitness in the past, so we have instincts to do them. There were different degrees of pressure on men and women for favoring different heuristics, so if evolution could find a way to make women and men behave differently, it would do that.

      • Yrke says:

        IDK, it looks like men who are optimizing for children are doing way better reproductively in the modern context than those who are optimizing for sex.

        Consider
        -the most successful guy on Tinder in NYC
        -the median monogamous Haredi male in NYC

        Who do you think would have the most kids? I’d bet the Haredi guy, and it wouldn’t even be close. And he isn’t even optimizing for children in the way a sperm donor would, and is taking a sex break around his partner’s cycle.

        If you look at the GSS, highly promiscuous guys have never actually had the most kids within the last century or so.

        The pinnacle of modern optimizing for kids is multi-partner, but not necessarily sexually multi-partner, since it involves becoming a sperm donor. You can do that and be a virgin.

        • The Nybbler says:

          The guy in Tinder is probably using birth control. How about promiscuous lower-class criminals?

          • Yrke says:

            The guy on Tinder is probably using birth control because having children interferes with him optimizing sex – i.e. his strategy of optimizing sex is reproductively self-sabotaging.

            A promiscuous lower class criminal could well be optimizing for reproduction as well as sex. Fast life strategy guys tend to be ready for kids earlier even as they invest less. On an absolute level, these guys aren’t really having a lot of kids either though. The average guy in prison has less than two kids.

            (There’s a cultural perspective that views (say) having a child each with 3 different baby mommas is having a lot of kids, but demographically really it’s not. Reproductive optimizers have a lot more than that)

    • xXxanonxXx says:

      I mean, that’s a sick burn, except… Isn’t prostitution a terrible reproductive strategy? Like, for both parties?

      Isn’t that a bit like asking if a reed warbler feeding a cuckoo is a sound reproductive strategy, and if not than how can we say evolution gave it the instinct to do so in the first place? The evolved behavior doesn’t need to be a sound strategy in all situations. On the contrary, sub optimal strategies for a specific environment give rise to evolutionary pressures that allow the whole process to work in the first place.

      And now I want someone to write a scifi story about evolution’s arms race against a species that’s smart enough to get all the sex it wants without having to reproduce. Except I think Zach Weiner basically already got there.

      • Deiseach says:

        And now I want someone to write a scifi story about evolution’s arms race against a species that’s smart enough to get all the sex it wants without having to reproduce.

        If everybody is happily banging everybody else but nobody is having babies, didn’t P.D. James get there with what happens when there aren’t any more kids in “Children of Men”? Even cuckoos reproduce, they just shove the child-raising off onto other birds. Sterile cuckoos (whether naturally or voluntarily) mean no more cuckoos at all.

        I suppose it’s workable if the species only gets around the “sex without babies” once it’s already been well established and there are enough members of it that it’s not going to die out in one generation, but still – somebody has to have the kids, or you need to invent artificial wombs and have reproduction all done in the mode of “Brave New World”, or it’ll be a short party even if it was marvellous fun.

        • xXxanonxXx says:

          I had screwball comedy in mind over realistic dystopia. To make it work you’d have to anthropomorphize evolution and give her fantastically heightened abilities. It’s then a struggle between an increasingly selfish and hedonistic population and the sentient force of nature for whom it’s really important humans continue to propagate, not just have a lot of fun and then die. Humans invent condoms. Evolution gives men semen that acts as a solvent on latex. Humans start pulling out. Evolution arranges it so women’s eggs are ejected during the deed and settle into their skin like a Suriname toad. This… this got a lot grosser a lot faster than I had anticipated. I’ll stop now.

          • Nornagest says:

            It seems a lot cleaner for evolution just to give everyone a weird pregnancy fetish, as per that SMBC comic.

    • The original Mr. X says:

      So what explains monogamy, which would seem to favor women’s reproductive needs over those of men?

      Marriage has never just been about who you get to have sex with, it’s also been about the man providing for his woman/children. Most people through history haven’t had the resources to provide for multiple families at once (which is why, in societies where polygamy exists, having lots of wives is generally a sign of status — it proves you’re rich enough to support all these extra people), so they just got married to one.

      Plus, even if that weren’t the case, widespread polygamy would be bad for society as a whole, because there isn’t a high enough male:female ratio for everybody to have multiple wives, so you’d end up creating an underclass of young males with no realistic prospects for marriage, something which rarely ends well for the people who’ve got to share a society with them.

      I think it would be insane to say that there aren’t sex differences in humans, but that’s not the interesting or open question.

      I think you’ll find it is in fact an open question in large parts of the modern intelligentsia.

      • Besserwisser says:

        Plus, even if that weren’t the case, widespread polygamy would be bad for society as a whole, because there isn’t a high enough male:female ratio for everybody to have multiple wives, so you’d end up creating an underclass of young males with no realistic prospects for marriage, something which rarely ends well for the people who’ve got to share a society with them.

        That’s effectively the case with a lot of our closest relatives. Infanticide is relatively rare in humans but almost ubiquitous in a lot of species which do practice polygamy. Which is an effective deterrent to that kind of reproductive strategy, resulting in other species, among them humans, evolving specifically to avoid that. There are other signs of that, like the relative large penii and testicles of men in proportion to body size, and no, I’m not just saying that to awe you at my magnificient reproductive organs. It’s just a sign that human males evolved to have competition in other ways, such as removing sperm from previous partners of the current female and simply having more sperm than the competition.

        • Deiseach says:

          widespread polygamy would be bad for society as a whole, because there isn’t a high enough male:female ratio for everybody to have multiple wives

          Ack. Look, I’m not in favour of polygamy, but if you’re going to introduce a modern, improved, better version of it then get with the times and consider polyandry as well as polygyny! Some cultures already have a version of it and surely all you smart kids can work the bugs out, instead of jumping to the conclusion that “polygamy always and invariably = one guy plus harem of women”?

          There are rich women in the world as well, you know, who could happily have a couple of cute young fellas to do the heavy lifting around the house and look decorative while they were at it.

          • The original Mr. X says:

            Men are generally less keen on sharing sexual partners than women are, and women aren’t keen to start with. The potential for jealousy strikes me as too great for any system of polyandry to work out over the long run.

          • Randy M says:

            evolutionarily, what does anyone get out of the many men/one women scenario? Assuming no off-spring outside of the arrangement, the men now have 1/n as many off-spring and the women have no additional. Is it just a way of making the numbers work out, ie, using up the extras after the alphas claim their harems? This assumes a sort of bi-modal sexual selection strategy based upon how likely one expects it will be to acquire the most/the best mates. Interesting proposal.

          • Skivverus says:

            @Mr. X

            Maybe so in aggregate, but personal experience suggests the opposite. Source, so we can avoid anecdote wars?

          • Nancy Lebovitz says:

            https://en.wikipedia.org/wiki/Polyandry

            “Fraternal polyandry was traditionally practiced among Tibetans in Nepal, parts of China and part of northern India, in which two or more brothers are married to the same wife, with the wife having equal “sexual access” to them.[6] It is associated with partible paternity, the cultural belief that a child can have more than one father.[5]

            “Polyandry is believed to be more likely in societies with scarce environmental resources. It is believed to limit human population growth and enhance child survival.[6][7] It is a rare form of marriage that exists not only among peasant families but also among the elite families.[8] For example, polyandry in the Himalayan mountains is related to the scarcity of land. The marriage of all brothers in a family to the same wife allows family land to remain intact and undivided. If every brother married separately and had children, family land would be split into unsustainable small plots. In contrast, very poor persons not owning land were less likely to practice polyandry in Buddhist Ladakh and Zanskar.[6] In Europe, the splitting up of land was prevented through the social practice of impartible inheritance. For example, disinheriting most siblings where many of whom then became celibate monks and priests.[9]

            “Polyandrous mating systems are also a common phenomenon in the animal kingdom.”

            So much for theory.

          • Gobbobobble says:

            The marriage of all brothers in a family to the same wife allows family land to remain intact and undivided. If every brother married separately and had children, family land would be split into unsustainable small plots.

            Well now I want a Crusader Kings mod for this

          • John Schilling says:

            Well now I want a Crusader Kings mod for this

            I vaguely recall reading that they have the code and hooks for matriarchal equivalents for all the traditionally patriarchal marriage and inheritance customs in the game, but that the AI does such a poor job of strategizing for them that nobody much bothers.

    • Anon. says:

      The absolute vast majority of cultures are polygamous. Even those that are monogamous in theory work polygamously in practice, it’s just not institutionalized. Hence girlfriend, not wife.

      • hlynkacg says:

        Citation needed.

        It doesn’t matter whether she’s your girlfriend or your wife, most cultures still take a rather dim view of infidelity.

        • The original Mr. X says:

          I presume Anon’s counting serial monogamy as a form of polygamy.

        • Spookykou says:

          Really tangential but this got me thinking about how many distinct cultures existed pre-writing, and if they make up a majority of all human cultures, how relevant are any given features that they share to our modern cultures?

        • Of 1231 societies with data for “Marital Composition: Monogmy and Polygamy” in Murdock’s Ethnographic Atlas, only 186 are strictly monogamous. But 453 have only “occasional” polygyny, and that results in a (slight) absolute majority of 639 societies with at most “occasional” polygyny. There are also 87 societies where polygyny is common but generally co-wives are sisters, which one might not consider quite full polygyny. So only 501 societies are “polygynous without caveats”. That’s still a fairly large number—you could say to an approximation that half of human societies are monogamous, and half of them are polygamous, with monogamy having the slight edge.

          (The number of polyandrous societies is insignificant: there are only 4 in the sample.)

    • Wrong Species says:

      It’s not the 19th century anymore. Marriage as an institution is breaking down. The reason we still have it is because of conservatism(not as in political but behavioral).

    • AnonYEmous says:

      So, uh, what exactly accounts for Australia’s national women’s soccer team? Perhaps they’re all really men.

      I really like this blog because you usually don’t see arguments like this one. The answer to your question is: outliers.

      Like, one thing that is common in all three reviews is that none of them explain why our current social set-ups exist, even when they don’t, to my ignorant eyes, seem to actually support the kind of reproductive strategies that are supposed to drive evolution.

      Well, I’ll try to educate you out of your ignorance, even if you invoked it facetiously.

      I mean, it seems to me that if the optimum strategy for men is to mate with many women, and the optimum strategy for women is to mate with one strong man who will support her in pregnancy, then we should tend towards polygamy, yeah?

      Would certainly explain all the polygamy we’ve had throughout the centuries and even into the present day.

      More reasonably, polygamy often leads to men who can’t support all of their women in pregnancy (and the subsequent children), because you have to have a lot of resources to be able to do that. It’s also often destabilizing to the society at large, which is why many religions and cultures found their way out of it. But it’s not like polygamy never existed, or doesn’t exist currently.

      In other words:

      So what explains monogamy, which would seem to favor women’s reproductive needs over those of men?

      The intermeshed desires of women and men, biology, and a bit of game theory.

      Women want either resources or great genes. If men are investing resources, they need to be relatively sure that they’re investing the resources in their kid and not the neighbor’s kid passed of as theirs, meaning monogamy. If men aren’t investing resources in their kid, the kid’s chance of survival drops pretty steeply, especially in the before-time when good jobs weren’t available to women and safety was a serious concern. This dichotomy could be avoided with polygamy, sleeping around, et cetera, but you’d need to have great genes or great wealth for that, meaning it’s not a strategy most can pursue.

      Why doesn’t the dude just have two wives?

      Wife number one understands that a second wife means her and her children will receive less resources, so she refuses. wife two understands that her and her children will receive less resources, so she refuses too. In the end, he decides on one.

      I mean, that’s a sick burn, except… Isn’t prostitution a terrible reproductive strategy? Like, for both parties?

      It’s a great way to make money for the woman. As for the man…this is sort of a complex explanation, but natural selection can just as easily optimize for wanting to have kids, or enjoying sex which leads you to having kids. In fact, the second is probably a lot easier, especially since it comes to us from the animals we evolved from. As humans, we can circumvent the limits of our biology and enjoy ourselves in ways nature didn’t intend.

      So, evolutionarily, where did it come from? Maybe that one is social conditioning? So how do we know that differences in male and female play aren’t just as much products of social conditioning as prostitution?

      Well, monkey experiments have provided the strongest proof of that nature we will ever get, by trapping baby monkeys in cages until they mature more. Unless mad scientists do the same to human babies, all we’ll ever have a bunch of really strong circumstantial evidence across all cultures and no real evidence to the contrary, but obviously not hard scientific evidence.

      Oh yeah, monkey experiments also showed that excess cash produces monkey prostitutes (mostly female I believe). Why do you think prostitution is a function of social conditioning?

      I think it would be insane to say that there aren’t sex differences in humans, but that’s not the interesting or open question.

      My answer to the question is: we’ll never know, and that’s why large-scale social activism shaming people for believing certain things and trying to alter those percentages shouldn’t be allowed. I recognize you weren’t so much saying that, like at all, but that’s just my position on the subject – we’ll never, ever know, again unless mad scientists lock human babies in cages, and I sincerely hope that doesn’t happen.

      • Randy M says:

        but natural selection can just as easily optimize for wanting to have kids, or enjoying sex which leads you to having kids.

        For sexually reproducing animals without an ability to understand cause-and-effect a large remove, even if the species evolved to care for and desire children, it is still more effective to enjoy the act of making them itself, since all the wanting in the world won’t help if you don’t understand how to fulfill the desire. Given that we’ve evolved from mammals with less foresight, strong desires for the reproductive act itself make sense.

        In other words, prostitution is a byproduct of the reproduction incentive.

      • fossilizedtreeresin says:

        We’ll probably never know for sure, but people raise their kids in different ways, and doesn’t that partially create different social environments for scientists to explore? Surely, scientists can tail a bunch of conservative and progressive parents, and see whose kids like more rough play/end up in more STEM jobs/ etc etc.

        When it comes down to it, my problem with research about difference between men and women is that it doesn’t seem to be a good predictor to social behaviors, but rather to retroactively explain, and whatever doesn’t fit the mold just get another not so convincing explanation, instead of an admission of “hmm, doesn’t fit the mold, maybe we should study it a bit more?”

        I’ll give an example: Women are supposed to be better at team work and communication. What art form is incredibly dependents on communication and team work? Film making, but in 2016, only 7 percent of the top 250 film directors were women*. Well, directing is hard work, maybe women prefer jobs with fewer hours, so they could raise a family? But 24 percent of top 250 producers were women, and producers work similar hours to directors, so shouldn’t we see similar numbers? Well, to be a good director you need to be bold and aggressive (there goes the importance of team work), so men, who are more prone the risk taking and aggression, do better at directing. But studio heads actually don’t like risk, or directors that are hard to work with! Even if men take more risks then women and end up with better movies, shouldn’t we see more women getting a shot at directing a movie for the first time?

        And so on and so on. I gave film making as an example because as a film student that’s the field I know the most about and it seems like a pretty good example, but I’m having trouble giving other fields as an example, and I can’t tell if it’s because film is an anomaly or if it’s because I don’t enough about other fields. Anybody else have a similar example?

        So I think at least part of the push-back against people believing in biological driven difference between the sexes is that the science itself seems insufficient, and not a good predictor (at least it does to me). It’s one thing to believe that boys in elementary school do better than girls in math in X percent, and it’s another, bigger thing to believe that because of that there are so few female mathematicians. I think it’s possible to settle the dispute between nature vs nurture by testing the accuracy of judgment by higher ups in male dominated fields. If we go to the physics department and show all the professors the same paper, but half the time have the name john on the cover and the other half jane, and if the paper is evaluated the same regardless, it’s a victory to the “women are just not as good at physics” crowd. If it is evaluated differently, it’s a win for the “there aren’t as many women in physics because sexism” crowd. From what I know, women are mostly undervalued: Funnier jokes are more misremembered as made by men, if a woman speaks for 30 percent of the time and a man 70 percent it’s viewed like they talked a similar amount, same with crowds and so on. But I get most of my research from feminists sources, if anybody got good studies that contradict that I’d like to read them.

        Another thing the biological-difference-are-a-big-deal side can do is try to predict future job trends. Maybe team up with a bunch of futurologists, try to predict what kind of new jobs will pop up in the future, and if someone can guess correctly what fields will be male dominated or female dominated that’s a big point to them. Extra points if it goes beyond “high salary fields would be men dominated, low salary jobs would be female dominated”.

        * I don’t have data on all the films made in USA, so the top 250 will do even though it probably don’t represent the full picture. For what’s it worth, according to my memory the numbers are pretty similar in indie movies, but I can’t find the article.

        • Aapje says:

          Women are supposed to be better at team work and communication.

          This is actually an extremely far reaching claim, which is expressed in a very unscientific way. What does ‘better at team work’ and ‘better at communication’ mean? Team work and communication are not behaviors, they are outcomes that result when people apply a bunch of different behaviors to a specific scenario. One set of behaviors can result in great team work in situation 1 and bad team work in situation 2.

          If we take the claim at face value, then it makes very little sense if you look at how men are clearly capable of very good team work and communication in male dominated environments like the military.

          The (somewhat decent) research that I’ve seen actually doesn’t claim that women are better at team work and communication, but claims that men and women work together and communicate differently. The logical expectation is then that the male style works better in some cases and the female style in other cases.

          So I think at least part of the push-back against people believing in biological driven difference between the sexes is that the science itself seems insufficient, and not a good predictor (at least it does to me).

          You are looking for predictions at a level of complexity where so many variables play a role that no conclusions about biological driven differences can be reached. Proper science is about eliminating variables as much as possible. This is very hard for gendered human behavior, since we are all ‘nurtured’ from a very young age.

          The strongest evidence for biological driven differences is research that found that babies already have gendered toy preferences and that monkeys have the same gendered toy preferences.

          If we go to the physics department and show all the professors the same paper, but half the time have the name john on the cover and the other half jane, and if the paper is evaluated the same regardless, it’s a victory to the “women are just not as good at physics” crowd.

          A tiny victory at best, because you have then only eliminated one possible cause that is not due to nature.

          I’m not aware of an experiment like that, but this paper found no differences in peer review scores or acceptance for papers with female authors. In general, research tends to find that different choices that men and women make explain most of the gender differences in outcomes. But none of this actually tells us how much of this is caused by nature and how much by nurture.

          My position is that the evidence suggests that both nature and nurture play a role.

          • fossilizedtreeresin says:

            You are looking for predictions at a level of complexity where so many variables play a role that no conclusions about biological driven differences can be reached.

            I mean, that’s kind of my point. I don’t think that those researches, even when they are well done, can predict real life, so I don’t think they should be used to explain real life, or at least not yet. If science can’t isolate it’s subject from society (and it can’t), then it’s not yet accurate enough to provide answers to complicated, real life questions. I don’t have a problem with scientist that do honest work and research the difference between women and men, I have a problem with people that say “WELL this explain EVERYTHING”. I don’t even have a problem with people that think at women and men are somewhat naturally different (I think that’s probably true on some level), but I have a problem with people saying “men and women are biologically different” as a conversation ender. Especially when they don’t follow up on how those biological differences work! Is it a hormone thing? Is it a brain thing? Is it a men-tend-to-be-bigger-and-we-are-still-dumb-animels-so-that-affect-even-non-violent-arguments thing?

            If proper science is about removing variables, we should try to remove the variable of “are men and women get treated the same?” before we try to explain different behaviors, in my opinion. If there is a big enough body of work, I think it can be done with reasonable certainty.

            As far as I can understand, most feminists have a problem with well paying male dominated fields because they assume women are being wrongfully kept away from them, because up until a few decades, women HAVE being kept out of well paying jobs. Prove that everybody is judged by their merit, and you will have a lot less feminists calling for affirmative action and so on.

            Repeat the jane/john paper test in academia, and high school, and elementary school, and actual companies hiring physics, and with parents, and if you always get that male and female work is evaluated equally, I think we can be pretty sure that it’s not society standing in the way of women physicists so it’s probably nature.

            And yes, it is a very long process, but it seems a lot more doable to me than trying to prove that it’s nature: you’ll need to find what it is (a gene? a hormone? an area in the brain?) and then prove that society didn’t influence the thing’s growth. Making a battery of tests that measure bias seems a lot easier to me (I’m thinking about something like describing A LOT of social situations, and switching up the gender described, and seeing if people rate the same behavior in different ways, and then for each field having a specific test for work related skills).

            In the end of all, I think most people want concrete answer: is the market place fair? Do we need to change something? If less women end up working more hours or in a manger position, is it because they don’t want to, aren’t good enough, or were wrongfully judged?

          • Aapje says:

            If proper science is about removing variables, we should try to remove the variable of “are men and women get treated the same?” before we try to explain different behaviors, in my opinion.

            I agree and strongly favor trying to get rid of cases where we can show that men and women are treated differently, but…

            As far as I can understand, most feminists have a problem with well paying male dominated fields because they assume women are being wrongfully kept away from them

            The bold bit is exactly why I don’t think that current mainstream feminism is doing this. If you start assuming causes, without any solid evidence (and actually, in the case of the wage gap, solid counter evidence that shows that most of the wage gap is choice-related), then you are just indulging in your biases.

            IMO, all activism has a strong tendency to buy into a victim-perpetrator narrative, as it is very enticing for humans to claim that all problems are the cause of the other and to make them responsible for solving the problems.

            My opinion is that most of feminism has developed a culture that is built around this victim-perpetrator narrative (generally called the patriarchy). A good question to ask a feminist in this regard is: Was it female privilege that women weren’t drafted for Vietnam? My experience is that while many feminists are willing to admit that gender norms harm men, they are not willing to use symmetric language, where for example, benefits to each gender are called ‘privilege’ (which is a poor word choice in itself, but that wouldn’t be so bad if it was used symmetrically).

            IMHO, people cannot be egalitarian if they classify harm or benefits for one gender differently from harm or benefits to another gender. Then they are just rationalizing treating one gender differently from the other.

            If less women end up working more hours or in a manager position, is it because they don’t want to, aren’t good enough, or were wrongfully judged?

            You are failing to ask the opposite question: if men end up working more hours or in a manager position, it is because they want to, or it is because they are pushed into it by society (in the same way that women are pushed into choices)?

            Social norms are applied to both genders and merely asking how they affect the behavior of one gender is exactly the kind of bias that I’m talking about, where women are assumed to be victimized and men are assumed to get their way. This is not a good way to look at reality.

            When the analysis is built on biased assumptions, the outcome will mimic those assumptions. It won’t mimic reality, though.

            PS. I can give many examples of feminists explicitly advocating for treating men and women differently.

          • Hyzenthlay says:

            Was it female privilege that women weren’t drafted for Vietnam?

            When I bring up this, or the fact that men are more likely to be victims of violent crime, etc., the usual response is, “Ah, but those crimes are being committed by other men.” Or, “But the Vietnam War was also started by men.”

            Or, a response I saw to a statement that men didn’t have many good birth control methods to choose from: “Well, men are running the world so if they wanted more birth control methods they would invent them, so clearly they don’t want them.” Apparently some people can’t quite wrap their heads around the idea that the individual members of a gender are not fungible and that men/women are not a hivemind; the fact that some men somewhere might want something doesn’t mean that other men will automatically move to provide it. Society is not “ruled by men,” it is ruled by a small group of people who are mostly male, and that’s a pretty significant difference. Men on the bottom don’t necessarily get a trickle-down benefit.

            But that ties into my main beef with the privilege rhetoric: it’s inherently collectivist. Through that lens, the world is not composed of individuals doing things to other individuals, but groups doing things to other groups. Human beings are seen mainly as appendages or representations of their group.

          • Aapje says:

            @Hyzenthlay

            When I bring up this, or the fact that men are more likely to be victims of violent crime, etc., the usual response is, “Ah, but those crimes are being committed by other men.” Or, “But the Vietnam War was also started by men.”

            Exactly and that shows that they believe in a victim-perpetrator narrative based on gender or they would consider it fairly irrelevant if the gender norms result in men harming men, women harming women, men harming women or women harming men. All that is necessary for gender norms to deserve attention is that people are socialized based on gender into different behavior.

            If people only care about situations where women are victims, they are not advocates for egalitarianism, but for women’s interests. Which is fine as long as they are honest about it, but that is often not the case.

            A huge flaw in most feminist theory is that it simultaneously argues that men in power do what benefits men*, but also that benevolent sexism exists (which is men sacrificing for women). However, the logical conclusion of the latter is that men in power may seek to benefit women. This undermines the entire concept of patriarchy as a system designed to benefit men, which is generally argued merely based on the gender of the people in leadership functions.

            * For some reason is it often assumed that these men act to benefit all other men, rather than the men and women from their own class or such. This seems weird to me, since these men are usually married to women, so it makes a lot more sense for them to seek benefits for those women, than Bob McDishwasher.

          • hlynkacg says:

            Through that lens, the world is not composed of individuals doing things to other individuals, but groups doing things to other groups. Human beings are seen mainly as appendages or representations of their group.

            Further evidence that we are living in a post irony world. The vast majority of anti-fa are orders of magnitude more fascist than the “fascists” they’re lining up to oppose.

          • fossilizedtreeresin says:

            @Aapje

            You bring up a lot of good points.

            When I wrote “assume”, I meant “assume that things might not be as they seem, and keep investigating” and not “assume everything is wrong we must act now GO GO GO” but you make a fair point anyway, because a lot (most?) feminist groups do lean a lot more heavily towards activism then research then I would like, but I’m not sure how I can change it, or what different group I can support. In the end of the day, feminism is the reason I can right now pursue my dream, and there don’t seem to be any kind of other groups doing good work or work that will make my life easier, so I’m sticking with feminism until I find something better. If a gender-race-sexuality natural group that tried to eliminate all bias will rise up, I will join them in a heartbeat (I don’t consider rationalism a group yet because there don’t seem to be any real world action, but correct me if I’m wrong).

            Considering the draft: as an Israeli I got to experience something like that first hand, and I’m pretty damn happy I didn’t have to serve in a combat position or an extra year. So in a certain regard I got a huge benefit or privilege or however you want to call it, but as a whole, my gender might have took a hit. If I was the kind of person that was suited to combat service in the army it would have been a lot harder for me to get into a combat position or climb up my way to higher ranks. And I do think that only-male draft have to do with bias against women in combat positions, because most are closed to women, women that want to get in have to battle it out in court, and the IDF could have just as easily said “fine, you can serve in Golani, but lift 60 kilos and do 200 push ups first”, and when things got bad enough women did serve and die for Israel – in the independence war every sixth soldier that died was a women.

            So: I think feminists are right that men only being drafted benefits the exceptional man (the one that will become a general) and hurt the exceptional woman, but hurts the common man and benefits the common woman.

            You are failing to ask the opposite question: if men end up working more hours or in a manager position, it is because they want to, or it is because they are pushed into it by society (in the same way that women are pushed into choices)?

            Well, yeah, because we were talking specifically about women, but I do think that men are also influenced by social pressure. Feminism doesn’t focus on it, but it doesn’t ignore it either, ie the glass escalator and Michael Kimmle books, that are mostly about how patriarchy effects men.

            And even though it is possible to push a man into manger position against his will (take this promotion or we’ll fire you) it seems less likely and the man will have more ways to resist a raise he doesn’t want, so I’ll bet it happens less.

            My crude understanding of human and feminism history is something like this: ambitious men had all the power, and screwed unambitious men and ambitious women in the process (the general is happy that he is a general, the unambitious man was sad that only men go to war, the ambitious woman was sad that she couldn’t go to war and become a general, and the unambitious woman was glad she could stay home with the kids). Along came the feminists, that were mostly ambitious women (I assume that you need to be ambitious to organize a social revolution) and wanted some of that power too, and in the end they got some of it, but they are still promoting the fight for power for women, not necessarily what’s best for ALL women.

            That’s why so much is being done and written about biological differences, I think: ambitious women take it as a threat – as people saying that THEY can’t rise high, and because it might get in the way of practices that are comfortable to them: It’s just easier and more pleasant to work with enough women around, they’re a certain buffer against sexual harassment and misogyny and other things. I described it all in a very mwhahaha kind of way, but I think it’s probably unconscious (like, “I couldn’t imagine not wanting a promotion, but people always told me women just didn’t want to work long hours! They’re making things up again!”).

            Anyway, I think part of the problem is a lack of an alternative: there is no rational, calm group working against bias and for better, more equal treatment for everybody, and I don’t think there will be one soon. So you’re stuck with no action and not enough data to calm you that no action is needed, or with possibly bad action. I get why you choose no action, but I can also get why some women choose possibly bad action.

          • Aapje says:

            @fossilizedtreeresin

            And even though it is possible to push a man into manager position against his will (take this promotion or we’ll fire you) it seems less likely and the man will have more ways to resist a raise he doesn’t want, so I’ll bet it happens less.

            The issue is that for both men and women, the evidence strongly suggests that most of the difference is incentives. For example, in the US the earnings gap really starts opening up when people get children, where couples tend to make the decision that the woman works less and the man becomes primary breadwinner. Who lacks free will in that scenario?

            My crude understanding of human and feminism history is something like this: ambitious men had all the power, and screwed unambitious men and ambitious women in the process. […] Along came the feminists, that were mostly ambitious women (I assume that you need to be ambitious to organize a social revolution) and wanted some of that power too, and in the end they got some of it, but they are still promoting the fight for power for women, not necessarily what’s best for ALL women.

            Yeah, that is pretty much the feminist narrative, but it suffers from a lack of historical perspective.

            Before the agricultural & industrial revolution, about 80% of the population were farmers. IMO, the lack of social mobility at the time was not merely due to those in power holding the commoners down, but a consequence of poor farmer productivity which necessitated that most of the population become a farmer, even if they had capabilities beyond that.

            The agricultural revolution made many farmers superfluous, so they moved to the cities, which then provided the workforce for industrialization. This in turn really hurt most men, who were treated badly in the mines & factories, but benefited a small upper class. This is why Marxist ideas arose during this period and not before.

            At this time, feminists were mainly upper class women who wanted the same benefits as the upper class men. However, lower class women had little desire to work in the mines & factories and thus didn’t see much benefit in feminism. Due to the successes of education for all & social democracy, more and more men got decent work conditions and such, so more and more women became jealous.

            The great benefit of my story over the feminist narrative is that it doesn’t treat historic people as idiots, but sees their choices as relatively logical responses to circumstances. If you read Blackstone, who pretty much wrote the book on British patriarchy, you can see that he is very concerned about treating everyone fairly. He is not gleefully arguing for oppressing women when he wrote down the rules for coverture and there is a lot of focus on ensuring the welfare of women.

            That’s why so much is being done and written about biological differences, I think: ambitious women take it as a threat

            The problem with seeing something as a threat is that you can start denying reality. It is beyond obvious that some biological differences exist (womb vs no womb, for example). The only question is the extent.

            Anyway, I think part of the problem is a lack of an alternative: there is no rational, calm group working against bias and for better, more equal treatment for everybody, and I don’t think there will be one soon.

            Well, a substantial subset of feminists very aggressively fight against alternatives, with almost no opposition by the rest.

            Domestic violence researcher Strauss got so frustrated over how people kept working against the truth for ideological reasons that he wrote this paper. When that level of oppression exists due to feminist influence, it is hard to see how a substantial rational, calm group can exist, as such a group will be discriminated against for research positions, funding, opportunities to spread their findings, etc, etc.

            Either feminism need to change to stop fighting against inconvenient truths or the power of feminists need to be weakened so alternatives get a chance. As things are going I think that more and more people are getting wise. The male gender norm requires stoicism, but the great advantage of the Internet is that it enables men to talk anonymously, so they are opening up about issues, such as domestic violence and rape. We’re seeing small victories like the CDC finally registering male rape by female perpetrators (although they refuse to call it rape, but at least we have the numbers now). Furthermore, there is now a situation where men are doing worse in the same area where women historically did worse (education). This makes it much harder to deny that there is an issue than for situations where men always did worse, but where we see that as normal.

            I hope that a radical shift in perception happens. When we start addressing men’s issues properly and stop using a biased narrative, this will also allow us to help women better.

          • AnonYEmous says:

            If I was the kind of person that was suited to combat service in the army it would have been a lot harder for me to get into a combat position or climb up my way to higher ranks.

            Hey, Caracal is making gains, or at least was slated to last time I heard of it. But more seriously, most of the guys I served with would’ve taken your trade in a heartbeat, and you’re definitely not considering the aspect of physicality – most women can’t do the things you are describing and even those that do are much more likely to get injured, among other things (sexual assault for example). At last recall, the most fit female soldiers were apparently about as good as the bottom-tier male soldiers. Why make everything gender neutral and incur all those problems, just for the sake of a few bottom-tier soldiers? Especially if you’ve already got a draft on, and doubly so if very few women want to be combat soldiers anyhow.

            (I mean, seriously, female soldiers in the Golani? Those fuckers are crazy, I’m not letting that happen and neither should anyone else. Though at least you might get some Golanis who aren’t likely to blow the heads off anyone who insults them, heh.)

        • AnonYEmous says:

          Well, there was one tech study where interviews were conducted over the phone (or Skype I believe) and voices were randomized, so men would have either a computerized male voice or a computerized female voice, and women same thing.

          Result: women who got male voices did slightly worse, men who got female voices did slightly better. Sure seems like just “being a female” gives you a boost.

          • Deiseach says:

            You have to wonder, though: computerised voices always sound a little fake, and would a “woman’s voice run through a vocoder” sound that extra bit off-putting to the interviewers?

          • rlms says:

            Here‘s the study you were probably talking about. Neither of the differences in performance were statistically significant.

            @Deiseach
            They tried to control for that by modulating some of the voices without changing the pitch (so the implied gender stayed the same).

          • Ratte says:

            @Deiseach Given that there appears to be a widespread preference for computerized female voices over male ones in everything from Star Trek to Alexa, I don’t think there’s much evidence for a disadvantage.

          • Deiseach says:

            Ratte, if there is a preference for female computerised voices and they were making the women sound like men with the computerised voices, that would perhaps explain the difference: male-sounding computerised voices sound worse somehow than female-sounding computerised voices, maybe because of the very fact that we’ve been given expectations that computerised voices will be female, and so male-sounding computerised voices for the women interviewees made them do slightly worse.

          • AnonYEmous says:

            RLMS:

            Yeah, but it looks like it was statistically insignificant due to a lack of data. I personally believe the trend will continue. If it does, that’s some powerful evidence.

        • Unsung says:

          There’s a preponderance of evidence that suggests innate biological differences between men and women that, like all almost everything having to do with population genetics, is statistical in nature. The reviews reference a lot of them. But a few more things to chew on: testimonies of transsexual people undergoing hormone therapy during their transition reflect significant changes in psychology.

          https://en.wikipedia.org/wiki/Hormone_replacement_therapy_(female-to-male)#Psychological_changes

          (As food for thought, ask yourself that if socialization is so strong, why do trans people exist in the first place in spite of supposedly overwhelming socializing forces?)

          I don’t think anyone here would argue that there isn’t a healthy amount of variance around personality traits in male and female populations, nor do I think anyone here would say we need to “bin” people into gender stereotypes based on valid statistical differences; an individual instantiation of a statistical phenomenon should not be held at gunpoint to match the statistics. Roughly translated into a practical philosophy is that we should respect an individual’s right to pursue whatever path/career/life they wish regardless of their demographics and, yes, minimize institutional barriers that are unfairly excluding people from access/opportunity to their “pursuit of happiness”, if I may borrow that colloquialism.

          The questions then become what, if any, institutional barriers are there in various fields? What is the nature of said barriers? What are the practical responses to removing them? It’s my position that the issue of sexism in most arenas is dead and dying, and reports of bias in fields, e.g. STEM, are overblown (of course sexist assholes exist, but the claim is that sexism barring women from entry is potent and widespread; that I disagree with).

          Much of the arguments for sexism claim that the lack of equality in outcome is due to some sort of implicit bias. Now, when it comes to implicit bias I’m not denying that it may exist, but it’s potency and link to real-world behavior is on shaky ground as the main (only?) tool used to validate it as a psychological concept has come under fire as part of the “reproducibility crisis” in social psychology

          http://nymag.com/scienceofus/2017/01/psychologys-racism-measuring-tool-isnt-up-to-the-job.html

          I never liked it to begin with as it’s rhetorically been used like some sort of original sin to justify reverse discrimination of a sort in discussions like this (e.g. how could you be impartial or have anything valid to say? You’re a white male riddled with icky implicit bias! I exaggerate, but you get it.)

          So beyond implicit bias, why should an inequality of outcome be considered an indicator that the system isn’t fair to begin with when it’s reasonable to expect differing often dichotomous temperaments and lifestyle choices between the sexes? Do we have a metric that allows us to determine the ideal balance of men/women in chemistry for example? Should we use quotas. I’d say of course not.

          http://www.pnas.org/content/112/17/5360.abstract

          There’s a constant drumbeat coming from the academy, the humanities and social sciences side, about the pervasiveness of implicit bias and discrimination along with an implication that it should be smoked out and expunged from society. But I find it increasingly hard to trust them when a.) many of their cherished concepts and pieces of evidence are not standing up to scientific scrutiny (see the IAT above and for example https://en.wikipedia.org/wiki/Stereotype_threat#Failures_to_replicate_and_publication_bias), and b.) it seems more and more that that those fields are rather biased.

          Why do a.) and b.) matter? Because if in position b.) and you have a tendency to look for evidence to support your conclusions about the world (don’t we all) in a position where you purport to be trying to determine the capital T truth about the way the world works, you’re likely to find yourself in position a.) when people finally step up to hold your ideas to the same critical standard we can for things like molecular/cell biology. Furthermore, dictating with certainty theories that serve your political agenda as the capital T truth on these complex social issues, even with the best of intentions, are likely to be wrong/flawed and result in unforeseen consequences when you translate them into policy.

          Bringing it full circle, I’m skeptical of implementing/advocating widespread political/cultural agendas when we have incomplete knowledge of the reality/truth about how the human mind and biological societies/cultures operate and when the supposed arbiters of truth in these areas seem to have a vested interest in having it fit a certain narrative (both of Fine’s books seem dedicated to avoiding knowledge of sex differences and setting up strawmen and non-sequitirs to obfuscate it, as if it were too dangerous to know).

          Relating specifically to women, people who claim western society is somehow holding women back are so opposed to even discussing to what degree, are so appalled at the suggestion that differing lifestyle choices and biological differences may contribute to contrasts between men and women, and are so invested in socialization and social construction that I can’t trust their claims that sexism is rampant or even a major problem in western society.

          • fossilizedtreeresin says:

            I believe that people are biased when it comes to gender because people are biased in general, and every previous generation thought that the way they were living is natural (women are just natural home makers, black people are just naturally not as smart, etc). Could it be that we truly got to the point where we see the world clearly? Maybe, but can’t you see why I’m a bit skeptical? The social changes allowing women to pursue whatever career they want are so recent, I’d just like to be extra sure we really did reach a point of equality.

            And having a field male dominated, even if it happens neutrally, can have bad consequences for women. I’ll give an example from my university: I have to work with other people to make movies, so I always try to make good connections for the future. Out of the four men I seriously worked with, two hit and me and wouldn’t let it drop even after I was clear I wasn’t interested. It wasn’t the end of the world, but it was a waste of resources for me: I would never have spent so much time working with them if I knew it wouldn’t lead to a good working relationship in the real world. About 30-50% of the students are female in my school, so I could just work with them more and mostly avoid this problem, but I have no idea what I would have done if 90% of the people studying with me were male. And no one did anything illegal! What they did is not sexual harassment, just kind of annoying. I don’t know what’s the solution to such situations, because I don’t think we should make people study or work at something they don’t like or forbid people from socializing and dating, but this could be a real problem for women, and that’s why I want to be really sure women just don’t like STEM before we leave the STEM departments alone.

            But I agree with you we shouldn’t take actions until we’re sure they’re founded, and if it were up to me I would trade affirmative action with more structured hiring and promotion practices (like, I don’t know, the boss keeps in writing what every employ did every day, and when it’s promotion time someone from HR goes over it to make sure good candidates are not overlooked).

            The research you linked to is interesting, but I think it’s only the first step: what I would like to know is if it carries over to all fields, like does equal female and male work is rated the same? Does the same social behaviors is rated the same? How much does sexual harassment (or regular but negative behavior like my example) hinder women? Could it be that the people hiring prefer women with the same achievements as man because it takes more effort from women to get to the same point as men, or are they just straight up biased towards women? I don’t know, it takes more research (I think both of us agree on that).

            I don’t think that social science will ever hold up to the same standards like cell biology, the same way economy doesn’t hold up to the standards of cell biology, but I do think we need to bring up the level of those researches before we reach an answer. Just because there aren’t enough people doing the work that’s needed, it doesn’t mean a problem doesn’t exist. I study in a super liberal place, at a super liberal field, with a lot of women, in a field that will allow me to be more independent later if I want, and I still run into a bit of problem when it comes to men. It leads me to think that there probably are some issues that women run into in heavily male dominated fields, where there isn’t an emphasis on feminist theory, and you’re more tied to your professor/boss/huge grants you could never come up with yourself. Whether those issues stop 5% of the women that want to work in that field or 50%, I don’t know.

            Ps. I don’t think your argument about transgender individuals and socialization holds up. People are also socialized to be able to read and seek other people’s emotions, yet autistic people exist, and I didn’t think biology had NO effect on women and men. I mostly want to know what is the extant of this effect. Does difference in biology explain 90% of the existing differences between men and women? 50%? 20%? The same way you aren’t satisfied with “But socialization matters!” I’m not satisfied with “But biology matters!”

          • reasoned argumentation says:

            I’ll give an example from my university: I have to work with other people to make movies, so I always try to make good connections for the future. Out of the four men I seriously worked with, two hit and me and wouldn’t let it drop even after I was clear I wasn’t interested. It wasn’t the end of the world, but it was a waste of resources for me: I would never have spent so much time working with them if I knew it wouldn’t lead to a good working relationship in the real world.

            Fish don’t notice water.

            How about – he’d never have worked with you in the first place if he wasn’t sexually interested in you. All the opportunities you get because men naturally notice women and try to have them around – those are invisible but you do notice and object to getting hit on.

          • fossilizedtreeresin says:

            How about – he’d never have worked with you in the first place if he wasn’t sexually interested in you.

            That’s the point, I don’t want him to work with me if he only wants sex. I would rather work with someone that wanted to build a professional relationship, even if it meant getting less help right now, because it would be more beneficial in the long term.

            All those kind of “opportunities” are second rate for me, and I would rather work harder/do more favors for other people to get them on board for my films rather then work with people that only want sex, but I can’t actually make that choice because they don’t tell me what their intentions are before I put time and effort into them.

            And as a side note: telling a woman “have you considered all the nice perks you get from people wanting to have sex with you?” comes off as insensitive, because it’s actually not that great. Men doing you favors because they want to sleep with you is as flattering as someone inviting you to their party because they want you to write their term paper for them. It’s a hollow and fake kind of attention, that disappears the moment you tell them no, and if I could flip a switch that made no man that I work with be attracted to me, I would.

          • AnonYEmous says:

            All those kind of “opportunities” are second rate for me, and I would rather work harder/do more favors for other people to get them on board for my films rather then work with people that only want sex, but I can’t actually make that choice because they don’t tell me what their intentions are before I put time and effort into them.

            So what would you say to people, male or female, who would rather have free opportunities due to sex appeal that they can then exploit?

          • fossilizedtreeresin says:

            @AnonYEmous

            I would say they are crummy people, but that they can do whatever they want as long as they are not breaking any laws, and that they should announce their intentions so no one confuses me with them.

            But, let’s be real, if female film students wanted to trade sex for services they would work as prostitutes on the side and get enough money to hire real professionals to work on their films. The kind of favors you can get are mostly for your peers, who are still learning the trade and not at the highest quality, and therefore a lot of their value is in future cooperation.

          • Nancy Lebovitz says:

            One thing I haven’t seen tested for is whether people are significantly distracted by being harassed (whether ambiguously or clearly) by sorts of thing people like them may well have a history of being harassed for.

            If the experiments which show that even mild exclusion throws people off balance hold up (have they held up?), then I would think that real prejudiced behavior matters a lot.

          • Hyzenthlay says:

            Men doing you favors because they want to sleep with you is as flattering as someone inviting you to their party because they want you to write their term paper for them.

            This quote aptly sums up a common gender-related disconnect.

            Obviously I’m generalizing here, but for a lot of (straight) men, sexual interest from a woman feels validating or flattering (“she’s interested in me”) and for a lot of women (straight or otherwise) sexual interest from men can feel vaguely uncomfortable or even demeaning (“he’s just looking for sex–he’s not interested in me.”) Which is not to say that women don’t want sex, but they’re more likely to see their sexual desirability as something separate from their identity and so don’t find it validating in the same way. For a man, the equivalent would probably be feeling that a woman was only interested in his money.

            It wouldn’t be hard to come up with either an evo psych or a social reason for why this is. But either way, it leads to men and women feeling envious of each other’s situations…men feeling like women get all the perks and attention, women envying the freedom men have to go anywhere without getting catcalled.

            Though I’m sure there are plenty of counterexamples, too…women who are flattered by sexual interest, men who wish they got less of it.

          • AnonYEmous says:

            I would say they are crummy people, but that they can do whatever they want as long as they are not breaking any laws, and that they should announce their intentions so no one confuses me with them.

            But you want a world in which men who behaved this way towards them would be stigmatized, meaning you would win and they would lose.

            But, let’s be real, if female film students wanted to trade sex for services they would work as prostitutes on the side and get enough money to hire real professionals to work on their films.

            Isn’t it possible to be willing to trade female attention and the promise of more, without being willing to prostitute yourself? This doesn’t really add up as the kind of interchangeable option that factors with opportunity cost – you lose a lot being a prostitute, including but not limited to time.

            The kind of favors you can get are mostly for your peers, who are still learning the trade and not at the highest quality, and therefore a lot of their value is in future cooperation.

            And you don’t think they may cooperate with you in the future if they enjoy being around you and feel they may get into your pants at some point? I mean, just saying.

            Point is : some people benefit from this. You don’t. Your changes would hurt them and help you, so it’s not a unilaterally good change. Feel free to try and make it anyhow, but it’s at least somewhat grey.

          • and if I could flip a switch that made no man that I work with be attracted to me, I would.

            I wonder to what extent you can. Presumably some things you do signal general interest or disinterest more or less strongly. Never having been a woman, I don’t know how effective tactics along such lines are.

          • fossilizedtreeresin says:

            @AnonYEmous

            This doesn’t really add up as the kind of interchangeable option that factors with opportunity cost – you lose a lot being a prostitute, including but not limited to time.

            Ok, fair point. So a more accurate equivalent will be a female student making a raffle or something, and putting in the name of every guy that helped her in exchange for maybe sex. I do wonder how much money you can make for a sex raffle (I think a lot of guys will put in 5$ no problem for a chance of sex, but what do I know?), but my point was female students are not getting a high quality service.

            And you don’t think they may cooperate with you in the future if they enjoy being around you and feel they may get into your pants at some point?

            Again, those are short terms deals – in my experience nobody hangs around you years waiting to fuck you. They make they’re point pretty quickly and don’t let it drop.

            some people benefit from this. You don’t. Your changes would hurt them and help you, so it’s not a unilaterally good change. Feel free to try and make it anyhow, but it’s at least somewhat grey.

            I think it’s a bit ridiculous to say that a change towards a more honest system, and one that is closer to what was promised is not a unilaterally good change (not mentioning the question of what most women prefer. I think most women at my school don’t like the tradeoff of maybe sex for services, because female directed movies at my school tend to have a greater female stuff from what I see, but I don’t have any data). This is a university: its declared purpose is to make you better at your craft and introduce you to people you can work with. Men sneakily trying to hit on me via getting on my project are getting in the way of my purpose, the purpose that is supposed to be shared by everyone in this school: getting better at your craft and making connections. The fact that some people might find it beneficial is irrelevant, because it goes against the declared purpose of the school, and is getting in the way of women and men who want to work together. If they want to go against what was agreed, they can announce their intentions. If declaring their intentions ruins the game or opens them up to scorn, perhaps they should rethink their actions. Or make a secret handshake, I don’t care as long as they leave me alone.

        • liquidpotato says:

          I will provide more details on the film making part, as I work in the industry.

          The majority of production assistants, production coordinators and the like are women. They provide communication links between the artists on the ground and the supervisors, keep track of schedule and budget, help liaise with clients and the like. In addition, HR-related matters like training and such often are majority women.

          There are plenty of women in artist roles as well, although there’s also a trend into which departments they tend to go to, it seems. It’s less pronounced now, but back 10 years, it’s very clear.

          As a disclaimer, this is my observation over the years. I do not propose any hypothesis to explain that behaviour.

          • fossilizedtreeresin says:

            Thank you for your answer. Have you noticed any difference in the amount of women working on set in regard to what’s the film budget/how many shooting days are? What would be the natural promotion for production assistants and production coordinators, and do you find women in the same amount on those jobs?

            In Israel, my home country, there were 12 female directors and 23 male directors that put out movies in 2016, and I wonder how much that have to do with the smaller budgets and shorter shooting schedules of Israeli movies, because I wouldn’t assume Israel is any more progressive when it comes to women (but who knows?).

    • Douglas Knight says:

      Sure, condemn the fact that they bothered to write reviews. But it’s unfair, a kind of double-counting, to condemn the contents of those reviews as if they chose the topics. Yes, they should have ignored the book, but if they write about it, they shouldn’t ignore that it denies the existence of prostitution.

    • Art Vandelay says:

      I believe that when it comes to polygamy the problem is not normally, as some have suggested, being able to support more than one family because in such societies the wives and older children will themselves contribute to the provisioning of the household. I think the bigger problem is having hordes of sexually frustrated young men with no particular attachments or commitments who rather resent the old dudes for hogging all the women.

    • vV_Vv says:

      So, uh, what exactly accounts for Australia’s national women’s soccer team?

      The Australia’s national women’s soccer team is most likely made of very unusually masculine women, both physically and behaviorally. And yet, they are not as physically strong and fast as a high-level amateur team of teenage males. This is evidence of the vast difference, at least in terms of physical performance, between men and women.

      why our current social set-ups exist, even when they don’t, to my ignorant eyes, seem to actually support the kind of reproductive strategies that are supposed to drive evolution.

      Our environment is very different form the environment of evolutionary adaptation. The whole history of human civilization is only ~10% of the history of behaviorally modern Homo sapiens, ~3% of the history of anatomically modern Homo sapiens, and less than 0.1% of all hominids. Don’t think about how some trait promotes fitness in our modern society, think about how it used to promote fitness in a population of hunter-gatherers living in the African plains.

      I mean, it seems to me that if the optimum strategy for men is to mate with many women, and the optimum strategy for women is to mate with one strong man who will support her in pregnancy, then we should tend towards polygamy, yeah?

      Some societies and subcultures (from the rationalist polyamorists to the ghetto African-Americans) do, even today. And even in ostensibly monogamous cultures, infidelity and paternity fraud are common, even when there are various kinds of social punishments against them.

      So why did monogamy appear in the first place? This is actually a good question.

      A possible hypothesis is that when sedentary agriculture was invented, it enabled wealth, in the form of land ownership, to be accumulated and inherited, creating the potential for huge inequalities that couldn’t exist in previous societies. In principle, this allowed a very small number of men who where at the top of the social hierarchy to monopolize all the women, except that land is unproductive if not worked on, especially by the people with strong arms unencumbered by child rearing duties, i.e. men. But men were unwilling to work if they weren’t being rewarded with sex and kids, they tended to become lazy and/or violent instead, thus monogamous marriage was established as a kind of redistributive policy for sex in order low-class men happy enough to maintain a functioning agrarian society. Men on the top of social hierarchy could still get multiple women, but up to a certain limit, while low-class peasant men who worked hard and played by the rules got their slice of the cake.

      Whether this explanation is true, it seems quite clear that monogamy is indeed a social construct that is not universal, needs to be actively enforced, and is not strictly observed, while polygamous mating is always happening all over the place, overtly or covertly.

      Why doesn’t the dude just have two wives?

      Who said he hasn’t? In many societies it is possible to have two wives.

      What is the evolutionary benefit to being the girlfriend next door, rather than the second wife?

      Maybe she is married to some dupe who spends his resources to raise the “boyfriend”‘s kid. Or maybe he even knows. I mean, it’s not like single mothers never find husbands. As long as the woman has some man to provide for her kid, she get the evolutionary benefit of having the “superior” (in the prehistoric setting) man’s genes in her kid.

      The man knows the encounter probably won’t produce a successful offspring, and probably won’t support the offspring if it does produce one.

      Prostitution is older than paternity testing and reliable birth control, so it may have been a viable reproductive strategy for men, depending the opportunity costs. It wasn’t a particularly good reproductive strategy for women, but starving to death was a worse reproductive strategy.

      Prostitutes who aren’t poor or enslaved are most likely mentally abnormal (e.g. they have cluster B personality disorders).

    • mlogan says:

      The worst thing Cochran cites (obliquely) is the “Vervet Monkey Study”. The excellent debunking of this study at http://mixingmemory.blogspot.com/2006/04/monkeys-playing-with-boys-and-girls.html was practically my introduction to the world of unbelievably bad evolutionary psychology research.

      The vervet monkey study did some very bad statistics, but let’s instead assume that their entire analysis was impeccable in every way. Look at the methodology: They use the fact that female monkeys play with cooking pans while male monkeys play with trucks as evidence for their hypothesis. But… monkeys do not know how to cook or drive!!!! As a species, they can’t have evolved an adaptive preference for something that is not used by their species. It would be like saying “Male monkeys showed a preference for playing with calculus textbooks, while female monkeys showed a preference for playing with novels. Therefore, men are better suited for math, and women are better suited for verbal pursuits.”

      It really makes my head spin.

      • J Mann says:

        To be fair, Cochran refers to studies on “vervet and rhesus monkeys”, which presumably means the 2002 Alexander and Hines paper and the 2008 Hassett/Seibert/Wallen paper.

        https://www.ncbi.nlm.nih.gov/pmc/articles/PMC2583786/

        Hassett and her co-authors claim that toy preference in humans is a robust and well known effect, and that they found a similar result in rhesus monkeys.

        You’re right that Cochran citing the Alexander study without discussion suggests a lack of famility – Hassett et al thought that their results contradicted Alexander and Hines.

        The researchers are on top of your criticism, though – they generally hypothesize that if the result is replicable, then there is something about wheeled toys that make them attractive to male monkeys and less attractive to female monkeys, such as the mechanical aspects or the shape. If the effect is valid and replicable (which I think we don’t confidently know yet), then presumably there is SOME explanation, albeit presumably not an absurd one.

      • AnonYEmous says:

        Sorry, but that article is if anything worse than the study it’s critiquing. I mean:

        But… monkeys do not know how to cook or drive!!!!

        Whereas baby humans do?

        Obviously not. So why the truck? Well, a quote from the study, that the monkeys moved the truck back and forth, like humans kids do. It’s an active toy. The study even notes this and so does the article.

        Of course, the cooking pot is a poor choice – the color is apparently doing a good amount of heavy lifting. But “truck” and “doll” seem like solid categories to base gender roles on. Certainly the male vervets didn’t like dolls as much as the females.

        And check out this justification for dismissing female preferences:

        “According to them, the females weren’t playing with the cooking pot and baby for any reason associated with the objects’ human “femininity,” but for reasons associated with species-specific gender roles (nurturing infant vervets with reddish faces)”

        Don’t you think “femininity” has a lot to do with female gender roles? So you’re just left with the term “species-specific”, basically saying that female vervets like to nurture infant vervets. Which is so different to female humans liking to nurture human infants, right?

        Bottom line: the study proves that female monkeys didn’t use the openly masculine toys nearly as much as the male monkeys, and male monkeys didn’t use the doll nearly as much as the female monkeys. That’s solid data. Something which feminists have very little of.

    • Edward Scizorhands says:

      it seems to me that if the optimum strategy for men is to mate with many women, and the optimum strategy for women is to mate with one strong man who will support her in pregnancy, then we should tend towards polygamy, yeah?

      This isn’t that hard if you just calm down and think about it.

      A society with surplus males means that men need to compete and take extreme risks to become above average, because the average male doesn’t reproduce. Historically, this meant war.

      In primitive societies, war between tribes would consume extra males. If the men go off to war and 1 in 4 makes it back, then that one takes 4 wives and everything proceeds. Over time only the “best” men make it back from war.

      (Does war lead to polygamy or does polygamy lead to war? Yes.)

      Monogamy is a cultural institution that societies create to make there be peace despite our natures, just like they create all sorts of other things to overrule our nature. The main principle of science, for example, is “how we can rationally assess evidence despite the fact that we are emotional creatures.”

      Monogamy is not optimal but it is compatible with human sexuality.

      You cannot have all three of

      1. equal birth ratios
      2. lack of war (or something else that kills a bunch of men)
      3. polygamy

      If we could snap our fingers and make birth ratios 1:4 men:women, then maybe we could have the best of all worlds, I guess. But there is no or minimal evolutionary pressure to bring that about.

  36. habu71 says:

    As for the 737-sized electric plane, this is simply absurd. The power density of current battery technology is simply nowhere near what would be required for this.
    And according to the article,

    They don’t plan on developing their own battery technology,

    They are simply spending money planning for someone else to provide the unobtainium.

    And if you need one more reason why this idea is ridiculous both now and the foreseeable future, consider the example of Boeing. For their new 787, they decided to use LiCoO2 batteries – which were pretty much state of the art in 2005 – rather than rely on older NiMH batteries. In aerospace, weight is everything, and the LiCoO2 had a significant power to weight advantage. Mind you, the power to weight ratio of the LiCoO2 batteries is still an order of magnitude or so too low for actually powering the engines, but they are significantly lighter than NiMH batteries.

    The end result? The new batteries caught fire and resulted in the grounding of the entire fleet of 787’s.

    • hlynkacg says:

      Don’t forget that the batteries will also need to power everything that will no longer be powered by the engines as well.

      • youzicha says:

        This is true, but it doesn’t really matter. E.g. a Boeing 737 is roughly this size (depending on the model it may have between 80 and 200 seats). It is powered by two CFM56 engines. The fuel consumption at full power is 0.9 kg/s per engine. The energy content of kerosine is 40 MJ/kg. So the power output of the engines is about 70 MW, completely dwarfing any non-propulsion need. (Each engine drives a 125 kW electrical generator, two orders of magnitude less.)

        • hlynkacg says:

          Those generators are for lighting and avionics. Things like hydraulics and air conditioning are powered directly either by bleed air or mechanically through the accessory gearbox.

          • bean says:

            The 787’s AC is all-electric, and most of the hydraulics, too. Total standard generator power is 1 MW. A bit of math suggests that at full power, the engine’s fuel consumption gives something like 160 MW.

          • hlynkacg says:

            So you’re saying that electrical auxiliaries increase electrical requirements by 500% compared to traditional mechanical auxiliaries? I’d say that sounds about right.

            Top of the line lithium batteries have a density of 200 – 300 watt hours per kg or there about. That means 1 KWH worth of battery will weigh approx. 4000 kg and a MWH worth of battery will weigh 1000 times that. That’s 4000 tons of battery needed to run the 787s auxiliaries (not the engines, just the auxiliaries) for 1 hour. To give you a sense of scale, a fully loaded 787 weighs “only” 230 tons, and a 737 less than 100.

            TL/DR barring some major breakthroughs, the battery needed to run a airliner’s basic systems and amenities for an hour would weigh many times more than the airliner itself.

          • bean says:

            Your math is off. If the battery has an energy density of 250 Wh/kg, then a 1 KWH battery will weigh 4 kg. So a battery for a 787 for an hour would be 4 tons, not 4000 tons.
            And if we assume the 787 has similar requirements to the 737, then it’s more 200% increase. That said, it could be that the batteries/APU provide peaking power that I don’t know about. I’m not familiar with the electrical system of either plane.

          • hlynkacg says:

            *facepalm*

            That’s what I get for posting drunk.

        • habu71 says:

          And according to wikipedia, Lithium-ion batteries possess an energy density of about .875 MJ/kg.
          Rough calculations show that at full load, the 737 has about 16000 kg of fuel aboard. To use batteries to store an equivalent amount of energy to that stored by the kerosene, you would need more than 650000 kg of lithium ion batteries – which is an order of magnitude more mass than the entire 737 itself.
          And this doesn’t even consider volumetric energy density or discharge rate.

          In other words, looking to improved battery technology to allow a battery powered 737 is exactly as ridiculous as telling someone that you are working out and taking steroids so that you can flap your arms fast enough to fly.

          • gbdub says:

            You don’t get to count the full energy density of kerosene though, since efficiency losses involved in burning kerosene in a jet engine to produce thrust are quite a bit higher than a battery to electric motor drivetrain. And they aren’t trying to get equivalent range or speed, so their total energy needs will be quite a bit lower.

            Then again, “an order of magnitude” seems about right for the difference in weight between a Tesla battery pack and the full gas tank of an equivalent car.

            Overall I agree this thing is pretty bonkers at current or near term technology levels.

    • Deiseach says:

      Call me stupid, but if they want silent airships, what’s wrong with going back to re-designing a better dirigible? Yes, the Hindenburg, but if they can’t improve on safety in eighty years, what have they been doing?

      I suppose the problem there is weather and storms interfering with schedules so even short hops could potentially be delayed for hours?

      • bean says:

        The problem is basically that airships don’t work. It’s primarily a matter of speed. Basically, you’re paying to get from A to B, and a 7#7 is going to get you there a lot faster than an airship. So for the zeppelin to make sense as anything other than a luxury (and I will say that I’m a bit surprised that nobody has built a zeppelin ‘cruise ship’) it has to be a lot cheaper to buy, so you can move a similar number of seat-miles per year per dollar. Actually, that’s not enough either because I’d rather be where I’m going than stuck in the air. Or the accommodations will need to be a lot better than an airplane for a similar price.
        Also, they’re very vulnerable to weather, which hinders dispatch reliability. Another thing that airlines prize. And there’s no infrastructure for them.

        • Glen Raphael says:

          Airships also make a lot of sense for slow, large cargo transport to and from areas where there aren’t good airports. Lockheed currently plans to build a dozen of that sort for delivery and use starting in 2018.

          • bean says:

            In theory. But I’m old enough to remember CargoLifter, and that failed rather spectacularly. Why is this time different? Lockheed is somewhat more reputable, yes, but big aerospace companies get involved in some rather wacky things, and LockMart’s management isn’t going to keep pouring money in if the market isn’t there to support it.

          • Glen Raphael says:

            The main thing that’s different is they have a specific customer willing to spend a half-billion dollars buying the specific design they think they can deliver in less than a year. The design has some unusual features too. (notably: it’s less buoyant than most designs)

            There’s more detail (including pretty rendered flyby shots) in the source article here.

        • JayT says:

          There is also the issue that dirigibles really can’t carry very much weight. The Lockheed one mentioned in Glen’s link can carry 20 metric tons of cargo while a 747 cargo plane can carry about 112 metric tons.

        • Deiseach says:

          I was thinking re: airships if they’re selling (amongst other things) “our electrically powered plane will be quieter” that for short hops between nearby cities, dirigibles would be the thing (and certainly would cut down on the aircraft take-off and landing noise which is one of the major complaints that people living in the vicinity of airports have).

          But if they are so vulnerable to weather, you can’t guarantee that the 9:00 a.m. flight from Brownville to Majorton will arrive at 11:15 a.m. every day and that would kill any route.

          • bean says:

            The comments on noise levels were Scott’s, and not the responsibility of the electric airplane people. It’s not like they haven’t said enough stupid things for us to not take them seriously.
            The problem with the dirigible for short hops is that it isn’t likely to be that much faster than land transport. The official airship speed record is 70 mph (a value often exceeded on freeways) and while the unofficial record is closer to 90 mph, it’s still not that fast. Once you take the overhead of air travel into account (and good luck getting that down unless you can entirely remove the TSA or its equivalent) it’s really hard to see any advantage in the short-haul market. Trains would eat its lunch, as they’re probably faster and almost certainly cheaper. The only possible advantage would be over ferries, but those are often ro-ro and leave you with a car on the other end. I suppose it could connect train systems across, say, the Irish Sea, but that’s a pretty limited market.
            And that seems like a market that’s very weather-sensitive. The only place that you can actually use a blimp is SoCal, and this is the home of the car.
            Edit:
            Actually, if I was going for a faster vehicle for moving people across a medium-sized body of water than existing ferries, the airship seems unlikely to be competitive with things like ground-effect vehicles, hovercraft, and hydrofoils.

    • eh says:

      What probability of success (defined however you want) would you assign to “ridiculous”? 10% chance of success? 0.01% chance of success? Lower?

      The problem of venture capital is the difficulty of distinguishing between “ridiculous”, “ridiculous marketing promises but will turn into something profitable”, “defies fundamental physical laws”, “sounds too good therefore all the value has already been extracted”, etc. Maybe this company will reinvent itself as manufacturing something unforeseeable like electric ground effect vehicles for tourists or better engines for fume cabinets or something.

      • hlynkacg says:

        I think it’s safe to conclude that this is just a marketing stunt for the reasons covered in the earlier thread.

        If they are in fact sitting on one or more of the technological breakthroughs they’d need to build and fly an all-electric medium-to-heavy weight aircraft within the next 20 years there are fantastically more profitable things they could be doing with that tech than trying to compete with Airbus and Boeing.

      • habu71 says:

        How about any prediction/plan so insane that I would instantly and without hesitation bet my entire life savings against its occurrence?

        And if anyone would like to take this bet…

        • suntzuanime says:

          At what odds? If the odds were right I’d bet my entire life savings against a substantial likelihood. At even odds I’d probably bet against something that was, like, 20% to happen.

  37. MawBTS says:

    If you ever find yourself in Searle’s Chinese Room, make sure the door’s ajar. He hasn’t been found guilty of wrongdoing just yet, but still, yikes.

    • Deiseach says:

      I honestly don’t know what to think of this; the one sentence quoted about something sexually harassing that he allegedly said is so unlike something I’d imagine an 80 year old philosopher to say that I have trouble believing it as anything said outside of an episode of Family Guy:

      “American imperialism? Oh boy, that sounds great, honey! Let’s go to bed and do that right now!”

      Though again, people do say stupid embarrassing crap so it could be true. I’m really glad that I don’t ever have to try and disentangle the truth out of this kind of a case, because unless you have an impartial witness or a recording, or it happened in public, how can you decide between “he said/she said”?

      • The original Mr. X says:

        I honestly don’t know what to think of this; the one sentence quoted about something sexually harassing that he allegedly said is so unlike something I’d imagine an 80 year old philosopher to say that I have trouble believing it as anything said outside of an episode of Family Guy:

        That was pretty much my reaction too. Whilst you no doubt get a few randy 84-year-old sex pests, I don’t think they’re all that numerous.

        • quanta413 says:

          That was pretty much my reaction too. Whilst you no doubt get a few randy 84-year-old sex pests, I don’t think they’re all that numerous.

          On the other hand, I think being very famous in an insular field but almost unknown outside of it is pretty optimal for using power to get sex. Men so routinely do dumb shit because they are horny including powerful men who have the power to do even dumber shit because they are horny, that I wouldn’t be surprised if it was was true.

          I’d take 50/50 odds on it if I was a gambler.

        • Scott Alexander says:

          If he did it, I’d be concerned about dementia. Lot of dementias that lower sexual inhibiations.

          • Protagoras says:

            Dementia could also explain his insistence that he’s never heard any rebuttal of the chinese room argument; maybe he actually doesn’t remember any of them. Anyway, the philosophical gossip I’ve been hearing doesn’t have the usual divide between the believe the victim and the suspend judgment until more evidence groups; everybody seems to be sure he’s guilty. Apparently he’s been known for doing this sort of thing for a long time.

          • MawBTS says:

            Reminds me of when the NYRoB ran a piece by the (then) 84 year old Richard Lewontin, who badly confused mitochondria with ribosomes.

            I wonder how many public intellectuals are out there with amyloid plaques quietly building up in their brains?

          • Eponymous says:

            Our president is 70 and says crazy things often. Politics aside, I’m somewhat concerned about his mental fitness.

            (In fairness, I was concerned about Hillary too. Did we have to swing so far from Obama to the geriatrics?)

          • Philosophisticat says:

            Brian Leiter, who is strongly disposed to come down sympathetically to the accused in these cases, has mentioned that rumors of Searle’s sexual pursuit of students have been around for at least thirty years.

      • Matt M says:

        because unless you have an impartial witness or a recording, or it happened in public, how can you decide between “he said/she said”?

        Call me old fashioned, but we USED to have an accepted cultural norm where innocence was assumed and guilt had to be proven with actual evidence beyond “he said/she said”

        • suntzuanime says:

          Well, I don’t think that’s true. Guilt has to be proven beyond a reasonable doubt, but witness testimony is accepted as one form of evidence that can be used to establish guilt. It comes down to things like credibility and plausibility.

        • BBA says:

          Depends on the stratum of society the accused came from. Some got the norm you describe, while others got the alternative norm of extrajudicial killing based on a bare unsubstantiated accusation.

        • wintermute92 says:

          Legally, fine (in the United States, for middle class whites, for a while). But even within that caveat list, unsubstantiated allegations have been ruining people’s lives since forever. (As have true-but-unprovable abuses that get ignored, I’m just reacting to the one point.)

          The idea that people 50 years ago didn’t suffer from unproven rumors is truly bizarre. The society which mistreated bachelors and spinsters for being presumed-gay certainly didn’t have a cultural norm where innocence was assumed without evidence.

    • Reasoner says:

      I know Joanna Ong a little bit, and she seems a little crazier even than your average SJW. So that makes me sympathetic to Searle.

      • MawBTS says:

        Well, just because you’re paranoid doesn’t mean they’re not really out to get you, I guess. It’ll be interesting to follow this.

    • Machina ex Deus says:

      I’m not a fan of “believe every accuser”; I think “sexual assault” is a rhetorical ploy to blur the line between rape and overly-long hugs; I paid attention to the Duke Lacrosse and UVA rape cases (guessed wrong on the first, right on the second). I 100% believe in getting more facts before making up my mind.

      Yet my instinct here is to completely believe Ong, and think of Searle as a randy old goat.

      The fuck is wrong with me?

      • The Nybbler says:

        Based on his age and some of the descriptions of his behavior I was leaning to dementia (and therefore he’s a randy old goat, but at least with an excuse), but various accusers have claimed he’s done these things for 10 years, so now I just don’t know.

        • HeelBearCub says:

          Do you really take the position that this is uncommon behavior?

          I think one thing that makes the UVA case and the Duke Lacrosse case typical of the incidents that are known about is that the normal everyday stuff is so commonplace as to be generally unremarkable, and therefore generally not marked or remarked upon.

          • AnonYEmous says:

            i’m sorry but I can’t parse your second paragraph, is it written correctly ?

          • The Nybbler says:

            > Do you really take the position that this is uncommon behavior?

            The behavior I’ve most heard claimed as common (and used to believe was) is along the lines of a male professor pressuring a student into a sexual relationship in return for various favors (or in return for not doing harm). The second one I’ve heard claimed as common is an awkward Don Quixote-like declaration of love. This is neither; this is just absolutely bizarre.

          • HeelBearCub says:

            @AnonYEmous:
            UVA and Duke Lacrosse are allegations of grossly aberrant behavior.

            Whereas this sounds like everyday sexual harassment where you constantly turn the talk in the professional environment towards the topic of a sexual relationship, and then maybe take retribution via managerial power if you are rebuffed.

            And this second kind of the sexual harassment is the the typical example, everyone knows this kind of stuff happens, but there usually isn’t any way to prove it, and it’s not a very exciting story, so no one actually talks about it except perhaps around the water cooler or over a pint at the local bar.

            @The Nybbler:
            I’m really not seeing this as very bizarre. The declaration of how they are fated to become sexual partners and a concurrent declaration of how well she will be taken care of professionally seems pretty typical.

            I’d guess in his mind (assuming it’s true), he thinks he is sort of a “Don Juan” whose still got it. Then he gets irritated when rebuffed. Perhaps he is less subtle about it than he used to be, for various reasons.

          • AnonYEmous says:

            OK, are you entirely sure that you wrote it correctly though

            if not I might have to look at it much, much, much more closely

          • HeelBearCub says:

            @AnonYEmous:
            I could probably add one comma? But otherwise, I wrote what I meant.

            To rephrase and make more “Hemingway”:
            We notice the unusual cases. Everyday sexual harassment is not unusual. It is therefore not made note of nor mentioned.

          • AnonYEmous says:

            so then how do you define

            the incidents that are known about

            i’m not even trying to make a political argument here i just can’t even parse it

          • HeelBearCub says:

            @AnonYEmous:

            I meant known outside the immediate circle of those affected and the very local community (like the department in question and some professionals in that field). By “known about” I mean famous, or perhaps infamous, or “well known”.

          • AnonYEmous says:

            but don’t you think some words are missing in that paragraph as a connector or similar

            you’re saying that the UVA case is typical because it is fake, and also infamous like other fake cases

            whereas the normal everyday stuff isn’t like that because it’s not infamous, but rather commonplace

            what is doing the work of “whereas”? or even “because”?

          • HeelBearCub says:

            @AnonYEmous:
            The UVA case isn’t a typical case, it’s typical of those which are widely reported.

        • random832 says:

          but various accusers have claimed he’s done these things for 10 years, so now I just don’t know.

          He wasn’t exactly young 10 years ago. Or is it that you expect if he had dementia then he would have deteriorated further?

  38. p duggie says:

    New Culture is a project of ‘Michael Rios and Jonica Hunter’

    googling them they are poly advocates.

    So yeah.

  39. Wrong Species says:

    On Tim Pool:

    Needless to say what I found was interesting and in reality “closer to the middle” in regards to the left/right narrative.

    I was surprised to find that there actually are gangs of orphan migrant youths going around robbing people and sexually assaulting women. Many people don’t want to talk about it but an MP in Uppsala recently came out and said it.

    I don’t want to know what really bad would be.

    • Scott Alexander says:

      Yeah, I got the impression he was conspicuously signaling centrism so that people wouldn’t write him off as One Of Them.

    • Philosophisticat says:

      There actually are italian mafia organizations, and criminals who are illegal immigrants from mexico, and asian american gangs going around robbing people and worse. That by itself tells us next to nothing about whether any of these issues are “really bad” or whether the corresponding “this is something we should be very concerned about!” narrative is accurate.

    • Desertopa says:

      As I understood it, he was saying “yes, these bad things are actually happening, but the magnitude isn’t that great compared to the background levels of crime that other countries already experience all the time.” Bad things are happening, but how much they’re happening also matters.

  40. IvanFyodorovich says:

    Does Uber have crappier technology than Google or Tesla, or are they just better about logging interventions?

    • Protagoras says:

      From what I’ve read about Uber, I would lean toward the crappier technology theory, though the reports that happen to pop up in my usual feed could for any number of reasons represent a biased sample.

    • JayT says:

      Their technology is younger, as far as I know. Google’s been working on their car for almost ten years, whereas Uber started their self driving car division two years ago. They hired a lot of really knowledgeable people to run the department, but I would still guess having a lot more research all under the same roof would lead to a better technology.

      • wintermute92 says:

        Yeah, this. Uber hired a lot of high-end experience in the field, but their project is very new and lacks both the age and ground-up design of Google vehicles. Adding self-driving to existing cars still seems to be vastly underperforming building focused self-driving vehicles.

        • The Nybbler says:

          The focused self-driving vehicles Google has built are fairly recent (and not full-capability); most of the work was done with a fleet of modified Priuses (and I think a Lexus and an Audi)

    • Douglas Knight says:

      To be clear, Uber has an intervention every mile, Tesla every 3 miles, and Google every 5,000 miles. So if you’re going to group two of them together, it wouldn’t be Google and Tesla.

      One expensive signal of quality is that Google vehicles don’t have a steering wheel. On the other hand, that means that Uber drivers probably intervene more, because they can make small interventions. “Critical” interventions are only once every 200 miles. Also, Google restricts itself to California, so it doesn’t have to worry about the weather. And it restricts itself to roads that it knows very well.

      • Glen Raphael says:

        Another expensive signal of quality is that Google gives demo rides to civilians. My parents got to ride a Google car; their cars are boringly competent drivers on Mountain View surface streets.

        (If Google can drive *in Mountain view* they can probably drive anywhere in California given appropriate business incentive to build out the maps.)

        • Douglas Knight says:

          Yes, Google gives demos, but so does Uber—to paying customers.

          Yes, I believe that Google’s official position is that Mountain View is as difficult as roads get (though not as weather gets), so it is fine to restrict to that case as they gather more data. Whereas Uber is testing on faster streets that might not be algorithmically more difficult, but are more dangerous, causing them to err on the side of intervention.

          • The Nybbler says:

            Speed aside, I’d say Pittsburgh streets are harder than the streets of downtown Mountain View.

            No self-driving car company has yet, as far as I know, had the nerve to try it in NYC and environs. Survive the NJ Turnpike, Lincoln Tunnel, surface streets of Manhattan, George Washington Bridge, both elevated and covered parts of NJ-139, and then get out of town on the LIE and maybe I’ll believe you can handle US roads.

          • Douglas Knight says:

            No one brags about 40 year old accomplishments.

          • wintermute92 says:

            @The Nybbler

            Similarly, make it through the Boston tunnels and bridges and I’ll be deeply impressed. Hell, only crash occasionally there and you’ll still be ahead of human drivers – though I don’t know how these cars handle places where street signs and traffic signals are actively misleading.

        • John Schilling says:

          As I understand it, Google has invested heavily in creating and maintaining extensively detailed maps of the relevant Mountain View streets, with everything that might confuse an idiot-savant robodriver being marked, catalogued, and if necessary assessed by human judgement. It is unclear whether Uber has done the same, but I would suspect that there is at least a qualitative difference that may be significant.

          Google also doesn’t drive on Mountain View streets during inclement weather or other adverse conditions; Uber’s paying customers will evaporate if they can’t get rides in the rain, so Uber may be pushing to roll out that aspect of the technology before its time.

      • IvanFyodorovich says:

        What I wonder about Tesla interventions are how many are overcautious/bored drivers taking over vs. how many are preventing accidents or major errors. I feel like they must be far better than one critical intervention every 200 miles. I mean, if Autopilot tried to crash you every 200 miles we would probably be much higher than one death by now and no one would trust it.

        • Douglas Knight says:

          The Tesla numbers are not for the currently existing product, but for testing a full-fledged autonomous car. And these are official numbers about testing on public roads, which Telsa only did for one week last year.

          (Also, this specifically does not include bored people taking over, only either the computer giving up or the tester deciding intervention is necessary, but that’s probably obvious once you know that this is only testing, not consumers who might want to drive.)

          • IvanFyodorovich says:

            I guess where I’m confused is that I thought Autopilot was basically autonomous (even if you weren’t supposed to use it as such). Somehow I thought Autopilot was a much better product than everything I read about these Ubers, which inspired the original question.

          • Douglas Knight says:

            Telsa is close to fully autonomous on limited access highways. It is useable on divided highways. But of course it chose those tasks because they are the easiest. Until last month, Autopilot refused to operate on local roads at all. Whereas, Uber pretty much only runs on local roads.

  41. mupetblast says:

    “His voice raised in a British lilt, Robinson announces, ‘One of the things alt-right guys are good at it is making all these weird, young, disaffected teens feel like they’re the cool ones and the left side is the boring side. We need to have that kind of thing.'”

    Indeed. But how the hell did the right get away with this? The corporate colonization of diversity rhetoric is probably one answer. Here’s a good Jacobin piece proffering another: https://www.jacobinmag.com/2017/02/paleocons-for-porn/

    Anyway, the comments section on that Ringer piece is pretty brutal.

    • martinw says:

      And here is our esteemed host with a blog post from three years ago, offering another, more high-level explanation.

    • suntzuanime says:

      Part of the problem is that the “alt-left” is sitting down and going “how can we make teens think we’re cool” while the alt-right is concocting heists to steal Shia LaBoeuf’s protest flag.

      • Scott Alexander says:

        The “alt-left” seems cool enough, it’s just the centrists who aren’t.

      • Conrad Honcho says:

        This is how you end up with Christian rock. 80s preachy moralists thump bibles -> kids draw pentagrams on their notebooks and listen to death metal -> preachy moralists invent Christian rock so they can “rap about Jesus like the cool kids.” Today we’ve got preachy moralists screeching about diversity -> kids draw swastikas on their notebooks -> out of touch moralists come up with….something we’ll cringe at next year.

        • Le Maistre Chat says:

          If the kids are listening to metal, inventing a type of rock so you can rap sounds like the most out-of-touch thing possible.

        • Eli says:

          Power metal was basically invented so you could have all the edgy Satanist pretend-play, all the glories of metal, a positive attitude, and actual melodies.

          AND IT WAS GREAT.

    • Eponymous says:

      At the risk of offending left-leaning readers…

      There’s a clear male/female split in politics. I think it’s fair to say that more conservative views are more appealing to men in general, and more liberal views are more appealing to women.

      I think this extends to concepts of gender as well. Simply put, I suspect that more masculine men are more likely to be conservative, and more feminine men are more likely to be liberal.

      And it’s no surprise then which side is “cooler” to disaffected teens.

      • suntzuanime says:

        That doesn’t explain why this is a recent phenomenon, though. For most of the past 50 years the left has been cooler than the right, right?

        • Nornagest says:

          I guess it’s conceivable that emphasis is shifting away from age-mediated values and toward gender-mediated values — the stuff geezers like is never cool. This would predict right-wing politics becoming more popular among the young (true, I think) and less popular among women (dunno).

        • Eponymous says:

          A fair point. My perception is that conservatives were more cool during the Reagan / fall of communism period, but not for the rest.

          Though I still think conservatives were perceived as more masculine throughout that entire period. Maybe masculinity isn’t as cool as I think it is?

          There’s probably an age component here. Young people skew left, and young people pretty much define what is “cool”.

        • JayT says:

          I think for much of the last 50 years the image of a manly man was for the most part people’s fathers, and as such, being a sensitive guy was a form of rebellion, which is always the “cool” choice. Nowadays, those sensitive guys are the dorky fathers telling dad jokes.

          • suntzuanime says:

            IOW, masculinity is cool, so long as it’s not associated with men.

          • Whatever Happened To Anonymous says:

            IOW, masculinity is cool, so long as it’s not associated with men.

            Yeah, I think OP’s theory only makes sense if we separate the giant enemy crab that we call the left into its smaller components.

            It’s feminism that is, almost by definition, unmasculine, not the left as a whole. It’s really only as the former has taken a larger role within the latter that a lot of “cool stuff” has been either abandoned or condemned by “”””the left””””.

            Still, I think the “Rage Against The Establishment” hypothesis makes more sense. Hell, there are reports that Generation Z is more conservative/right-wing than prior ones. Not sure how true this is, but it’d be a huge departure from folk wisdom.

          • Randy M says:

            Feminisim isn’t unmasculine. It’s ostensibly anti-masculinity, but apart from the rhetorical war on words related to men it tends to see as normative traits and behaviors associated with men–independence, assertiveness, authority, strength, coarseness, promiscuity.

          • eyeballfrog says:

            Feminism encourages those qualities in women while simultaneously decrying them in men.

          • Randy M says:

            Right. So feminism, in encouraging it’s members to take on the qualities of men, can’t be said to be unmasculine, any more than I would evince gentleness by overpowering you.
            Of course, it is against the concept of segregating virtues, against the concept of distinctive masculinity and femininity.

          • Machina ex Deus says:

            @Whatever:

            It’s really only as the former has taken a larger role within the latter that a lot of “cool stuff” has been either abandoned or condemned by “”””the left””””.

            Wait, I’m confused: are you signaling that the left is controlled by midget Jews?

          • Montfort says:

            Machina ex, I’m not sure if you actually don’t know this, but for anyone confused: in some modern dialects of internet english, successive double quotes are used to indicate increasing sarcasm – on the theory that if saying someone is “cool” means you think they’re lame, saying they’re “””cool””” must be three times as scornful.

        • Acedia says:

          Hard left positions on social issues like feminism, racism, immigration etc. became establishment, so being against them became anti-establishment, hence cool.

          Hard left economic positions are still anti-establishment, which is why is the cool kids of leftist social media are the ironycommunists.

        • The original Mr. X says:

          That doesn’t explain why this is a recent phenomenon, though. For most of the past 50 years the left has been cooler than the right, right?

          Back in the old days the establishment was generally conservative, so the left was the cool rebels overturning the system. Now they’ve largely succeeded, the establishment is generally left-wing, and it’s now the left who’ve become the censorious killjoys telling people what they can and can’t do.

          • Aapje says:

            I would also argue that the reason why some people here perceive that the left has won is not just because their ideals have been accepted, but also because the parts of left has become considerably more right wing. One way to get your way is to change the world, another is to change your desires to be more like reality.

        • Yakimi says:

          Simple: the First World Left was allied with and inspired by Third World revolutionary movements, the aesthetic of which was implicitly masculine. They styled themselves as revolutionaries, and even revolutions need to be organized on a “conservative” basis: you need military discipline, appealing uniforms, and violent males, all of which are cool. “Join us, and you, too, can be like Che Guevera.” Now that those movements have withered, so has the Left’s aesthetic and the result is this.

          The Dirtbag Left/Swole Left thing is kind of interesting because they’re trying to make leftism implicitly masculine and cool again (in part by signalling against social justice types), a strategy for which left-liberals deride them as brocialists.

          • The Nybbler says:

            Now that those movements have withered, so has the Left’s aesthetic and the result is this.

            That’s actually a group calling themselves the Pastel Front, and they’re a group aligned against the antifa. The antifa themselves are much more masculine, in the same way the fascists are.

          • Montfort says:

            I see, but I also have pictures.

          • dndnrsn says:

            Relevant: one of the internal criticisms of left-wing radical movements in the 60s and 70s was their machismo and misogyny. There’s Stokeley Carmichael’s famous statement (he was joking but, as the saying goes, intent is not magic) that “the position of women in the movement is prone.” The same criticism continues today – you’ll occasionally see stuff about how antifa etc groups feature a lot of machismo and some of the people attracted to that stuff are more into the punching and breaking things than they are to the politics. None of this is surprising.

            Related is the occasional scandal where a vocal Male Feminist ™ turns out to have a habit of doing all sorts of sketchy shit. There’s a problem with abuse, etc in activist circles – it doesn’t even go away when there are no men involved! – to the degree that there’s a book about it. (I read the book, and a common thread was that people who are socially adept are really really good at getting away with shit in environments that are supposed to have minimum hierarchy. Of course, any group is going to have some abusers in it, and hierarchy is prone to abuse too, but the observation is not hard to find that groups with no official hierarchy can end up with really nasty unofficial hierarchies.)

            (my conclusion is that hierarchies are inescapable, and it’s basically a frying pan/fire situation – do you have a formal hierarchy, which is open to abuse, or do you attempt to eschew hierarchy and end up with an informal hierarchy, which is open to abuse?)

          • HeelBearCub says:

            @dndnrsn:
            Well said.

            One of the things that constantly amazes me is the assumption that because government, hierarchy, organization, etc. has not achieved perfection, that it is therefore useless or inherently corrupt.

            It’s why I view the far left as doing the work of the (current) right for them.

          • dndnrsn says:

            @HeelBearCub:

            I think there is a tendency, independent of right/left, to take the position that the good is the enemy of the perfect, so to speak.

          • HeelBearCub says:

            @dndnrsn:

            Sure, it’s a human nature thing. Everyone is susceptible to being seduced by utopia.

            But it would be hard to find a more central figure to the current American right than Grover Norquist. And Grover Norquist wants to whittle the federal government down to a size where it “can be drowned in a bathtub.”

            The central position on the right is to reduce as much of the federal government as possible, and the state governments too, while they are are at it. So, in this case, the mainstream left is left to do all of the lifting when it comes to keeping the government functional.

          • Matt M says:

            “But it would be hard to find a more central figure to the current American right than Grover Norquist. And Grover Norquist wants to whittle the federal government down to a size where it “can be drowned in a bathtub.””

            And not only has it not shrunk to that level, it continues to grow. Despite people who have pledged loyalty to him controlling 2/3 branches of government (and basically tied for the third).

            He’s either lying about his intentions, or is insanely incompetent at achieving them. Which do you suppose it is?

          • cassander says:

            @HeelBearCub says:

            The central position on the right is to reduce as much of the federal government as possible, and the state governments too, while they are are at it. So, in this case, the mainstream left is left to do all of the lifting when it comes to keeping the government functional.

            If this is the central position on the right, why have they never, ever done this when in power? the last time the budget was lower at the end of a presidency than the beginning was eisenhower’s first term. What agencies did GWB abolish, what regulations did he repeal (I said repeal, mind you, not delay, or modify before implementation)? that all the republicans want to do is take a hatchet to things almost pure myth.

          • HeelBearCub says:

            @cassander:
            Partly because Republicans have been disingenuous. But as more and more primary victories are won by true believers, this becomes less true.

            But also because the position is based on utopian/fantasy thinking.

            But it’s not like I am misdescribing Norquist, and I believe it true that he is as central a figure to the modern right as exists. Do you dispute that?

          • cassander says:

            @HeelBearCub says:

            Partly because Republicans have been disingenuous. But as more and more primary victories are won by true believers, this becomes less true.

            if they were being disingenuous then, by definition, cutting government is NOT their core position.

            But also because the position is based on utopian/fantasy thinking.

            how on earth is reducing the size of government a fantasy? You realize it’s been done before, right?

            But it’s not like I am misdescribing Norquist, and I believe it true that he is as central a figure to the modern right as exists. Do you dispute that?

            I think you are misunderstanding and caricaturing his position based on a sound bite. And I think you are vastly overstating his importance to the modern right. He’s an issue advocate, but not one with an enormous amount of money and organization behind him. His organization has a budget of around 12 million a year. I was going to say he was more like the head of Greenpeace than, say, the NEA, but even greenpeace has a budget of 20 times what he has.

          • dndnrsn says:

            The American (or Canadian) right is a funny case. There’s a lot of rhetoric of small government, but the powerful interests don’t want small government – they want government that serves their interests. Laissez faire capitalism and crony capitalism have entirely different supporters. Of course, some of the latter use the rhetoric of the former, because it’s more appealing rhetoric.

            How many people, having struck down the evil wizard, wouldn’t try on the magic ring, just once, just a little bit, just to see how it feels? That’s what’s going on. “Disingenous” implies some level of intent. It’s more… I’m sure that when they’re out of power, a lot of them really do intend to shrink the government. It just never turns out that way.

          • AnonYEmous says:

            if they were being disingenuous then, by definition, cutting government is NOT their core position.

            yes, but it’s the core position they pretend to have, because that’s what the right’s voters and intellectuals want. Or if that’s not the case, address that argument instead.

            how on earth is reducing the size of government a fantasy? You realize it’s been done before, right?

            seems like an uncharitable way to interpret the argument when it’s most likely that the amount of reduction is fantastical, rather than the idea of reduction itself

          • HeelBearCub says:

            @cassander:
            Norquist’s “Taxpayer Protection Pledge” was signed by 238 of 242 Republican House Members prior to the 2012 election. That’s not Greenpeace.

            Norquist doesn’t need a huge budget precisely because he isn’t trying to move Republican’s very far off their extant positions.

            The fantasy is that you can shrink the federal budget by cutting foreign aid. The idea that you can increase military spending and not touch SS, Medicare, or debt payments while cutting top end taxes and not grow the federal debt usually involves fantastical assumptions. Not to mention all the other government functions that they hate by name, but like by function. Like the Department of Energy.

            When “Taxed Enough Already” tea party voters think that Repubicans are proposing to cut their taxes by any substantive amount, that also involves a certain amount of misdirection.

            Nonetheless, the true believers who think that failing to increase the federal debt ceiling is a good idea have grown powerful enough in the Republican caucus that Boehner couldn’t get debt ceilings passed by his own caucus, and needed almost every single Democrat and a handful of Republicans to pass vital legislation.

          • Aapje says:

            It seems to me that the American right have a tendency to cut taxes, but not cut spending, because the latter hurts them with voters. The idea is that when the Democrats get into power, they then either have to raise taxes or cut spending, both of which hurt them in the eyes of voters.

          • suntzuanime says:

            The very simplified version is that the Republicans cut taxes and the Democrats raise spending, because people like spending and don’t like taxes, so they elect the two parties to advance those goals.

          • HeelBearCub says:

            @suntzuanime:
            Are you trying to say that simplified version is reality?

            As a rhetorical device used by a deficit hawk, it’s punchy, but it’s not true.

          • Matt M says:

            It seems to me that the American right have a tendency to cut taxes

            They move the top bracket from 39% to 35% and maybe fiddle with the thresholds for various brackets a bit. Does that strike you as emblematic of people who want to wholly dismantle the government?

          • suntzuanime says:

            Are you trying to say that simplified version is reality?

            At least as much reality as Aapje’s model. Generally when things are simplified, they do not reflect all the nuances of reality, because that’s what it means to simplify something.

          • cassander says:

            @HeelBearCub

            >Norquist’s “Taxpayer Protection Pledge” was signed by 238 of 242 Republican House Members prior to the 2012 election. That’s not Greenpeace.

            I swear, the way the left fetishizes that damned pledge as if it meant something. What of it? taxes have gone up since then. By contrast, greenpeace offered its support to politicians who pledged to increase environmental protections, and low and behold, they did go up. you’re right, they aren’t greenpeace. greenpeace gets shit done, norquist doesn’t.

            Norquist doesn’t need a huge budget precisely because he isn’t trying to move Republican’s very far off their extant positions.

            The point of the money isn’t to move politicians, but to move voters to support or oppose politicians. Norquist’s ability to do that is basically zero, as witnessed by his total failure to actually prevented. If were republicans were half as set on lowering taxes as you claim then you’d think that taxes would be lower than they were 30 years ago. they aren’t.

            The fantasy is that you can shrink the federal budget by cutting foreign aid. The idea that you can increase military spending and not touch SS, Medicare, or debt payments while cutting top end taxes and not grow the federal debt usually involves fantastical assumptions. Not to mention all the other government functions that they hate by name, but like by function. Like the Department of Energy.

            Sure, but that’s no more a fantasy then the trillions in savings promised by advocates of single payer healthcare. Almost all voters are totally delusional about policy, that’s part for the course.

            When “Taxed Enough Already” tea party voters think that Repubicans are proposing to cut their taxes by any substantive amount, that also involves a certain amount of misdirection.

            the taxed enough party earned it’s chops by primarying people they didn’t think were republican enough, for precisely that reason.

            Nonetheless, the true believers who think that failing to increase the federal debt ceiling is a good idea have grown powerful enough in the Republican caucus that Boehner couldn’t get debt ceilings passed by his own caucus

            well, one, that’s not what happened. but more importantly, the assertion that the only way to force the government to cut spending is to stop it from borrowing is hardly delusional. Indeed, it seems strongly borne out by the evidence.

          • Cerebral Paul Z. says:

            The important point is that cutting taxes (to the extent that that’s even been done) is not at all the same thing as whittling down government to a size where it can be drowned in a bathtub.

          • onyomi says:

            In favor of this simplification being broadly true: it’s what incentives would lead us to expect: people like spending but they don’t like taxes. These two facts, taken together, should lead us to expect for them to vote for politicians who spend more but don’t raise taxes enough (if at all) to compensate and/or politicians who cut taxes but don’t cut spending enough (if at all) to compensate. It would be quite surprising if this weren’t what happens.

            And as far as I can tell this is what plays out in reality, with the past few Democrat administrations doing option A, and the past few Republican administrations doing option B.

            The result is that spending and deficits nearly always increase, albeit at different rates, with spending increasing faster under Democrats and deficits increasing faster under Republicans.

            Insofar as some Republicans really mean it when they say they want to cut the size of government, I think they are right to feel betrayed in that simply cutting taxes without cutting spending does not meaningfully reduce the burden of government; in fact, it arguably makes it worse, by shifting the burden more to e.g. those without the resources and sophistication to hedge against inflation.

            Conversely, cutting spending would reduce the burden of government on those most vulnerable even without tax cuts: if the government took in large amounts of money in taxes but didn’t spend it, the result would be to increase the purchasing power of dollars in circulation. But they do the opposite, meaning that even those too poor to explicitly pay much in taxes nevertheless lose purchasing power.

            But “unsophisticated savers on fixed incomes who’d like to see the value of their incomes and savings gradually increase” is not a concentrated interest group, so they always lose out to the narrower groups currently benefiting directly from the spending.

          • Aapje says:

            I think that an element that is missing in the discussion is that there is a game theory element to it.

            There are ways to to gain short term growth at the expense of the long term. For example, not maintaining infrastructure, removing regulation that hampers boom/bust cycles, borrow and spend (if the lenders don’t gradually increase the interest rates), etc.

            If you are forever in power, it is not sensible to do these things, but if you are not, you can gain by getting the advantages of these strategies for yourself and letting the chickens come home to roost for someone else.

            For example, President Trumlinton ensures that he has a strong economy by borrowing from the future. He is reelected after 4 years (win #1). The economy starts to tank just after 8 years, when a new president is elected, for which most people blame the incoming president, not Trumlinton (win #2). Americans seem to get tired of a party after 8 years, so even with a good economy it will be hard for Trumlinton’s party to win after 8 years, but in case they do win, the strong economy probably helped a lot (win #3). That president then is set up to do badly and may only govern 4 years, but:
            – he still gets to implement his agenda
            – that actually makes Trumlinton look better by comparison (win #4)
            – it’s fairly unlikely that his party gets such a bad reputation from this that it really hurts their chances to win in the future.
            So this strategy should give the party that uses it more election wins and be especially good for the legacy of the president that uses it.

          • onyomi says:

            If you are forever in power, it is not sensible to do these things, but if you are not, you can gain by getting the advantages of these strategies for yourself and letting the chickens come home to roost for someone else.

            This is basically Hans Hoppe’s argument for monarchy being better than democracy: the monarch treats the nation like his private property (which he can bequeath to his children) which he has an incentive to take care of in the long run, whereas the politician has an incentive to do whatever will get him reelected in the short run, even if it’s long-run disastrous for the country.

            I’m not really convinced that this advantage of monarchy over democracy is enough to outweigh all the ways in which democracy is probably better than monarchy, but it’s something (better solution, of course, is ancap, where everything is private property, but not of one inbred guy).

          • HeelBearCub says:

            @Cassander:
            I’m perfectly willing to concede that environmental protection is a core principle of the Democratic party.

      • Montfort says:

        Then how come all the cool disaffected teens of the 70s chose the left?

        • hlynkacg says:

          Well my knee-jerk reply would be “because back in the 70s the popular image of “the left” was Che Guerva and Mic Jagger now it’s Lena Dunham.”

          I’m honestly not sure if I’m joking.

          • Montfort says:

            Then how come such masculine men chose the left?

            Is there a Che of the alt-right? We’re running low on charismatic paramilitary types in general (besides ISIS/Al Qaeda/etc – and I don’t think either the left or the alt-right are going to be putting out al-Baghdadi t-shirts).

          • quanta413 says:

            I think this sort of thing is more true than we’d all like.

            Milo is a lot flashier and edgier than anybody on the right was in the 70s. And it’s much harder to be edgy and left wing now; the collapse of the soviet union has really lowered the opportunities for kicking ass in the name of the proletariat. Less money and soviet backing not to mention the repeated horrifying failures of communists have really hollowed out the most violent (and thus coolest) archenemies of the capitalist rightists.

        • Eponymous says:

          Being a rebel is cooler than being establishment?

          • hlynkacg says:

            That too, especially when you’re a teenager.

          • Montfort says:

            Of course. But establishment has little to do with masculinity/femininity.
            Additionally, one can rebel to the left or to the right – hence the proliferation of exciting new forms of race/gender theory

          • John Nerst says:

            Rebellion is masculine, and being right wing is masculine. For decades the two were on opposite sides of the political spectrum, resulting in relative balance among men. But recently things have changed and left wing social attitudes has become the establishment so “rebellion” is now right wing. Suddenly all the masculinity is on one side, resulting in a hypermasculine right-wing rebelliousness combination.

            Something like that?

      • Salem says:

        At the risk of disturbing the slumber of the Old Ones, the married/single split dominates the male/female split.

      • NostalgiaForInfinity says:

        The gender voting gap isn’t consistent over time in either the US or the UK – generally women have shifted further left (and men further right) in both countries over the past 30 years. Women in the UK used to vote consistently Conservative:

        https://www.ncpolitics.uk/2015/03/history-voting-patterns-gender.html/

        Which suggests it’s not inherent in conservative/liberal positions; the parties have just adopted policies that tend to appeal more or less to women in recent years. Unless you think either modern party is more truly right/left than their predecessors 40 years ago.

        There’s a good section in Democracy for realists that addresses how the Republican/Democrat stances on abortion changed their members’ positions on the issue (and filtered out people who felt strongly opposed to their parties’ policies, IIRC).

        • AnonYEmous says:

          The biggest divide is apparently married women vs. single women. I’d look to that as explaining the shift.

      • Eli says:

        I always wonder how these stereotypes are supposed to apply to me. Everyone who meets me thinks I’m a particularly overtly masculine male. They also attribute various traits to me which would possibly involve high-testosterone alleles.

        I honestly feel closer to the “center” of the gender spectrum, very mildly masculine if anything. I mostly think the gender standards of the people around me are skewed as hell.

        Oh, and I’m a socialist, which we all know is dominated by “bros” nowadays. So, uh, how does the model fit?

    • hatkat12 says:

      Pro-tip: neither the alt-right nor dirtbag left are cool.

      The popular stereotype of both groups, to the extent that a popular stereotype even exists for them, is of a socially awkward nerd who spends a lot of time indoors.

      • suntzuanime says:

        The battle for the soul of the socially awkward nerds who spend a lot of time indoors is going to determine the winner of the next war, just like the Nazis doomed themselves by getting every Jew who could build an atom bomb to try to flee to the US. So the real question is, what does the socially awkward nerd who spends a lot of time indoors think is cool?

        • The Nybbler says:

          So the real question is, what does the socially awkward nerd who spends a lot of time indoors think is cool?

          Social Justice, vidya, or the Chinese government (these are actual Chinese nerds from China). I suspect the last is the largest group.

        • Civilis says:

          Decades ago, I was the socially awkward nerd, in part because my contrary nature rebelled against what was popular. My tastes haven’t changed since then, but a number of the things I enjoyed at the time have become, if not universally cool, popular enough to be ‘cool’ to some percentage of the mainstream. Socially awkward nerds are now a significant portion of the market, and not just a niche.

          What I think has changed is that as technology has advanced, marketing and even manufacturing has changed the way things become cool. It’s most visible with the internet, where anyone can reach a world-wide audience with their cultural product, but other advances paved the way. One example is likely the move for TV from a limited broadcast spectrum to the multiplicity of cable channels, both providing more options, making the options cheaper, and making it easier to target a specific audience.

          The iconic image of Che Guavera is a cultural product put out by a taste-maker. That photo was chosen to make him and his cause look good. It becoming popular wasn’t a spontaneous development. On the other hand, the modern ‘meme’ era is an era of various random people with the power to publish thousands of images to see which resonate and which don’t. Somehow, a random green frog resonated enough to catch on and reach the public consciousness.

  42. tayfie says:

    Re: Rare Pepe Currency

    I first saw that story at Crypto Insider.

    The gist of it is that people are creating trading cards featuring the meme which, after being certified by the Rare Pepe Foundation, use coin coloring to ensure that each card is backed by some amount of bitcoin. This ensures the collectibles are limited. Apparently, some of these cards are being traded for “thousands of dollars worth of XCF on the Counterparty platform”.

    A full game using the cards is in the works.

  43. simoj says:

    i noticed that the context for the “exportability of nordic democracy” post was an argument between the author and matt breunig. i seem to have MB filed in my head as a person who argues in bad faith and should be muted – is this fair and/or should i reconsider?

    • MawBTS says:

      My most vivid memory: about 2 years ago he was involved in an Twitter slapfight about SAT scores and income mobility. He made some math errors that implied the max SAT score was 1800 (or something). When this was pointed out he was basically a smarmy dipshit and didn’t apologize or admit he was wrong.

      Sorry for not having more details. I can’t show you the tweets because he’s since purged his Twitter account.

    • Did anyone understand that article? He seems to have rounded off the Nordic Model to public healthcare, and hr seems to think there is some negstive relationship between rurality and public healthcare, and hr argues that the Nordics are peculiarly well placed to implement publlic Healthcare because they are city states, and he doesnt notice that placres like France which are nothing like city states have public healthcare..

    • Did anyone understand that article? He seems to have rounded off the Nordic Model to public healthcare, and hr seems to think there is some negstive relationship between rurality and public healthcare, and hr argues that the Nordics are peculiarly well placed to implement publlic Healthcare because they are city states, and he doesnt notice that placres like France which are nothing like city states have public healthcare..

      • 1soru1 says:

        I think the takeaway point is that economics, not geography, drives politics.

        Geographically, the US does have actual deserts, which Europe doesn’t, unless you count Greenland. But, as approximately no-one actually lives in deserts, it’s hard to see that mattering much, any more than the exact ratio of rural to urban areas. What does matter is the relationship of the rural areas to the state.

        Unlike the US, all European countries, via the Common Agricultural Policy, massively subsidize rural areas, whether they have a lot of it or a little. The rural population see themselves, especially in France, as guardians and protectors of the environment, and naturally guardians expect to be paid to do their job.

        Whereas the US rural populace see themselves as pioneers who can’t expect anything from the state, except perhaps protection against Apache raids. And those have been notably rare recently, so what is the state even for?

        • Aapje says:

          So we need the Apache’s to start raiding again.

          I propose the 28th amendment: forbidding all tribes from running casino’s.

  44. Prof. Quincy Adams Wagstaff says:

    “Some of Trump’s crueler policies might be interpretable in the context of trying to scare people out of illegal-immigrating.”

    Oh stop it with the constant sniping. You are smarter, better, and less boring than this. Control your amygdala.

    • Scott Alexander says:

      Consider for example the first incarnation of the travel ban, where people who were vacationing outside the country were forbidden from coming back in. The news at the time said this wasn’t a simple mistake – people had pointed out to Bannon that this would happen, and he said to go ahead anyway without changing the law to prevent it. Of course, eventually they changed that part of the law, and it never would have survived the courts anyway.

      The going narrative is that Bannon was just hopelessly confused and kind of an idiot and had no idea how to write laws. The other possibility is that he was deliberately planning that every media source in the world would cover how terrible and disruptive this was to immigrants, and immigrants would get the message and not come.

      I think I’m trying to be more charitable than anyone else here. If you disagree, give me an actual argument why.

      • Nornagest says:

        My pet theory is that the travel bans (at least, the first one) were essentially stunts and/or boundaries-testing exercises: Bannon et al. probably wouldn’t have cared much if they had been upheld, but their real purpose was to produce a lot of buzz, stir up the base, keep the immigration issue in the news, and possibly help an inexperienced executive team calibrate what it could get away with. They look bizarrely reckless and slapshod because they weren’t designed to last, and because they are executive orders, they can easily be modified later if by some miracle they don’t get struck down.

        • IvanFyodorovich says:

          I did wonder about the “boundary testing” aspect to the executive order. I always felt like the White House back in the Bush days couldn’t possibly have cared so much about whether they got to indefinitely detain Jose Padilla (a minor terrorist who was eventually easily convicted in the regular courts) but just wanted to see exactly how far they could go.

          If that’s what Trump and company were trying for though, it probably backfired. The crazy first ban made it easier for judges to strike down the second, less crazy ban.

      • tmk says:

        I just assumed Bannon really hates immigrants and wants them to suffer.

    • Ilya Shpitser says:

      Sanity check: out of the three explanations (incompetence, evil, something else) on the details of the ban in question, how likely is each of these, by your lights? Why?

  45. Ozy Frantz says:

    Jerry Coyne’s review of Cordelia Fine’s new book on the biology of sex/gender. Stuart Ritchie’s review. Greg Cochran’s review. Since everyone with any subject-area expertise agrees it’s total bunk, you won’t be surprised to see the New York Times, etc, giving it glowing reviews.

    This seems unfair to me. Jerry Coyne links to P Z Myers giving Fine a glowing review, and it seems to me that he does have subject-area expertise in biology, since he’s a professional evolutionary biologist.

    I have yet to read Cordelia Fine’s new book, though I have a copy, and I do not have an opinion on the accuracy of her book.

    • Scott Alexander says:

      Myers gets so many things about basic biology weirdly and bizarrely wrong that I’m not sure I trust him on more general issues.

      Nevertheless, I admit he is technically a biologist and have edited the post.

      • Eponymous says:

        Everything I’ve read by Myers suggests he’s mind-killed on anything political. Based on a brief sampling, I’d pretty much written him off as a source of information.

        But since you linked to him, and since he is sort of an area expert, and because I like to give people second chances and be charitable, and because I like to read people with views different than mine….I decided to give him a read.

        And it’s even worse than I could have imagined. His “review” consists almost entirely of the analysis of a mathematical case that is easy to show to be complete baloney. Honestly, if I saw a review like that by an anonymous person online, I would think they were an innumerate high school student. It’s such an obvious rhetorical sleight of hand that it backfires heavily for any reader with a reasonable faculty with numbers.

        I suggest you no longer consider him a reliable source on anything. He is simply an idiot, and I do not say that lightly.

        • xXxanonxXx says:

          Your assessment is spot on, but in the spirit of charity I’d add that he wrote a very good rebuttal to Michael Behe’s idea of irreducible complexity. Despite the snarky language, I’ve seen it change people’s minds.

      • sympathizer says:

        he is technically a biologist

        He’s not in bad company: See Matthew Garrett, and Richard Dawkins.

        • Mitch Lindgren says:

          I don’t know who Matthew Garett is, but Dawkins has so many more accomplishments to his name than Myers that I don’t see how the two can even be compared.

      • Ozy Frantz says:

        I can buy “P Z Myers is mindkilled about politics-y topics and is therefore not trustworthy, in spite of his subject-matter expertise.”

      • wintermute92 says:

        Myers is at this point apparently editing people’s critical comments on his piece to make them look bad. I’m filing this one under “interesting demonstration of maliciousness and political bias”, but not much more.

    • stillnotking says:

      That’s the same post where a commenter demonstrated with simple math and realistic assumptions that multiple mating is the best reproductive strategy for males, and asked for a double check, to which PZ responded:

      “Your math is fine. It’s your humanity that is broken.”

      Ah, PZ. Never change.

      • Eponymous says:

        I rest my case.

        Anyone with an interest in forming an accurate view of the world should ignore anything written by this man.

      • suntzuanime says:

        I mean, he’s not wrong. A properly-functioning humanity would kick in with some hypocrisy to avoid believing a socially disadvantageous truth.

      • Anatoly says:

        FWIW, he got unusually strong pushback from his own commenters, and later apologized.

        • stillnotking says:

          He apologized for being uncivil. Props for that, but it really wasn’t what I had a problem with. He’s still refusing to engage with the well-established facts and theories around this issue, in favor of attacking strawmen and waving his hands vaguely about “reductionism”.

          No one seriously thinks a man is likely to impregnate 100 women a year. That’s an extreme limit case used in a thought experiment. If Cordelia Fine and PZ Myers don’t understand that, it’s because they don’t want to understand it.

          • J Mann says:

            Yeah, the quote Fine was debunking was that a man with 100 partners can have “as many as” 100 children in a year – it’s obviously an extreme example, but Fine and Myers both treat the conclusion that 100 children from 100 partners is very unlikely as a complete debunking of the reproductive case for promiscuity.

            Myers should have titled his piece “Fine totally destroys case for promiscuity with bad ass math.”

          • Randy M says:

            That was the quote? Because partners =/= couplings, and it seemed her math was assuming one attempt with each.

          • Anatoly says:

            I went and read the paper by Schmitt that Fine quotes from. The quotation is not out of context. There’s a reason Schmitt chose 100 women a year, and not say 10 or 1000. Schmitt is trying to establish that a short-term mating stragegy of indiscriminate mating with numerous partners is more reproductively advantageous for males than it is for females. “100 women a year” seems possible with an extreme case of such a strategy: basically a new one-night stand twice a week.

            How many women would you impregnate with such a strategy? Taking Fine’s 3% rate of success per encounter, perhaps three or so. If that rate is pessimistic, perhaps a bit more but certainly not 100. Why does Schmitt then say “up to 100” – technically correct, but misleading?

            There’s an interesting question hiding behind the ridiculous strawman of 100 impregnations a year. If indeed you’d only expect to sire 3-4 offspring, then *indiscriminate mating with numerous partners* is not actually a lot more advantageous reproductively then, say, “keeping a harem of 4 women”, “having a wife and two mistresses” or even “being serially monogamous and dumping your girlfriend when she gets pregnant”. You only need 100 *different* women if you expect to get *a lot* of them pregnant. If you don’t, having the same amount of sex with few women will work just as well. And that means that Schmitt’s “indiscrimate mating with numerous partners” is not the obviously right strategy for males to evolve to maximize reproductive success. Although it may seems so before you start thinking of rates of success and such.

            I think it’s fine of Fine to point out that Schmitt’s rhetorics were misleading, but it seemed silly to estimate the uninspiring odds of impregnating all 100 women with one encounter each.

            P.S. Schmitt’s paper is at http://www.bradley.edu/dotAsset/163311.pdf, the excerpt quoted by Fine is on the third page. The paper itself analyzes the results of International Sexuality Description Project, by which they mean that they gave a questionnaire about sex to college students in 50 different countries.

          • At a bit of a tangent, that 3% figure has to assume random timing. With a little organization so as to have most of your coupling in or near the woman’s fertile period, you push it up above 10%.

          • stillnotking says:

            The assumption is that the influence of genes isn’t specific enough to produce behaviors like “maintain a harem of four women”. That assumption may not be correct — we’re just beginning to scratch the surface of this field, and courtship/mating patterns in other animals can be incredibly specific — but the real point is that whatever the optimal male strategy, it differs from the optimal female strategy in the direction of seeking out more mates. Fine’s objection is that male reproductive potential is realistically bounded at much less than 100 offspring per year, which is true, but it only needs to be bounded at something greater than the ~1.1/yr female limit for sexually dimorphic selection pressure to occur. If a promiscuous man has 1.3 offspring on average while his less promiscuous rival has 0.9, that’s a significant advantage, and it can be achieved just by sneaking into the rival’s tent a few times per year. Their respective mates are stuck at 1.1 each no matter what.

          • 1soru1 says:

            One key question is what are the evolutionary origins of the 3% figure?

            Is it a case of evolution doing as much as it can to maximize that number in that face of hard physical constraints?

            Or it it a case of it tuning it for an optimal value, neither too high nor too low?

            Because if the latter, it will presumably be downstream of culture, adapting to it as much as vice versa.

            Also, the corresponding figure for IVF is now up to 35%, presumably with still scope anf motive for improvement. Biology is, as usual, more easily mutable than culture.

            What are the implications?

          • HeelBearCub says:

            that’s a significant advantage, and it can be achieved just by sneaking into the rival’s tent a few times per year.

            This should be obviously not true. It makes the exact same mistake, and several others, that the original example of 100 does.

            One key question is what are the evolutionary origins of the 3% figure?

            Well, given that most (IIRC) female mammals go into heat, we should be asking the opposite question, which is why do we have sex outside of the fertile period?

          • stillnotking says:

            This should be obviously not true. It makes the exact same mistake, and several others, that the original example of 100 does.

            I don’t think it’s the same error, but fine — that line was tongue-in-cheek anyway.

            The basic question is: Can promiscuity improve the upper limit on male reproduction more than it can female reproduction? Since a promiscuous female cannot have more than ~1.1 children/year under any circumstances, while a promiscuous male obviously can, the answer to this question must be “yes”, regardless of any other numbers involved, with whom they are mating, etc.

          • Spookykou says:

            @HBC

            This should be obviously not true. It makes the exact same mistake, and several others, that the original example of 100 does.

            Could you clarify what you are saying here.

            Is this just a flaw with the particulars of the example, like bad math, or is the common problem here that the underlying idea is fundamentally wrong?

          • J Mann says:

            @Spookykou – depending on your perspective and your preference for snark, I think the key error is either (1) grossly overestimating the likelihood of success per coupling or (2) using imprecise language in the presence of pedants.*

            In this case “a few” extra sexual encounters per year wouldn’t raise expected offspring from .9 to 1.3, especially if the extra partner had a regular partner herself.

            * I kid! **

            ** Well, not really, but I identify as a pedant myself, if that helps.

          • Protagoras says:

            In assessing the long-term viability of a strategy, I believe it is standard to treat the number of grandchildren as more informative than the number of children (a bunch of kids who don’t survive to have children of their own are obviously meaningless in the long term). But the clean, simple numeric comparisons all involve children. If high-promiscuity strategies for men tended to produce children who themselves were relatively unsuccessful, they would not be very good strategies. Conversely, if high promiscuity strategies for women improved their ability to produce successful offspring (perhaps by increasing the number of men willing to assist them; there seems to be some evidence of some such phenomenon in bonobos), then looking only at the fact that promiscuity doesn’t do much to increase the size of the next generation for women would give an equally misleading view of the role of promiscuity in female mating strategies. Note that I am not endorsing these, or any other, particular stories, just pointing out that the “Myers is obviously wrong” seems to be based on simplified models of a situation that is complex. Maybe he’d still be wrong according to an analysis that examined all the complexities, but nobody here is providing that (and in any event such an analysis would hardly show him to be obviously wrong).

          • Brad says:

            Note that I am not endorsing these, or any other, particular stories, just pointing out that the “Myers is obviously wrong” seems to be based on simplified models of a situation that is complex. Maybe he’d still be wrong according to an analysis that examined all the complexities, but nobody here is providing that (and in any event such an analysis would hardly show him to be obviously wrong).

            I hate to be a buzzkill, but wouldn’t we expect at some point this or any other model to be subjected to some sort of empirical test?

            Unless this is supposed to be a branch of math (game theory?) where we posit our axioms and then see what kind of hypothesis we can prove from them.

          • stillnotking says:

            @Protagoras:

            All this theorizing is intended to explain the observation that men are more likely to be promiscuous than women across a wide range of cultures. Schmitt made an exhaustive case for that, and AFAIK no one really disagrees (if they do, they never bother to mention it).

            There may be some other reason for it; for instance, one could argue that women are prevented by sexist expectations and institutions from seeking promiscuity, but would if those were removed. Not relevant, though. Myers and Fine aren’t proposing an alternative theory, they’re attempting to debunk the evopsych one.

          • J Mann says:

            @Protagoras

            1) If Coyne, Cochran, et al. are accurate, then Myers is pretty close to obviously wrong to the extent that he argues that Fine has debunked the theory that selective pressure on males to compete for mating led to some of the behavioral differences we see today between men and women. They make a pretty strong case that Fine debunks the most easy arguments, then ignores any evidence and arguments she can’t debunk.

            2) I think in particular, Myers is in fact obviously wrong when he quotes the following math exercise from Fine and comments that based on that exercise, monogamy “seems to be the best evolutionary strategy.”

            Indeed, a promiscuous man would need to have sex with more than 130 women just to have 90 percent odds of outdoing the one baby a monogamous man might expect to father in a year.

            Suddenly, my preferred reproductive strategy, monogamy and paternal investment in offspring, seems to be the best evolutionary strategy.

            Monogamy might be the best evolutionary strategy today (which would be irrelevant), it might even have been the dominant evolutionary strategy during the period when selective pressures are hypothesized to give rise to some of the behavioral differences observed today (although I doubt it), but Myers is obviously wrong that Fine’s math example demonstrates that that seems to be the case – let me know if you want to discuss.

          • random832 says:

            All this theorizing is intended to explain the observation that men are more likely to be promiscuous than women across a wide range of cultures. Schmitt made an exhaustive case for that, and AFAIK no one really disagrees (if they do, they never bother to mention it).

            Er… if all those 100 women are only having sex with the one guy who is having sex with 100 women, then who are the other 99 guys having sex with?

            It doesn’t seem like it’s mathematically possible for the mean lifetime number of heterosexual sex partners to be different by any more than the difference in the total number of “sexual marketplace” participants. Presumably then we’re somehow measuring promiscuity differently, but the “observation” doesn’t seem terribly rigorous as-is.

          • Aapje says:

            @random832

            Er… if all those 100 women are only having sex with the one guy who is having sex with 100 women, then who are the other 99 guys having sex with?

            1. Why do you assume that they are having sex? There is no law that everyone gets to have sex.
            2. Prostitutes
            3. Why do you assume a 1:1 ratio of men to women? If men are encouraged to engage in dangerous activities (hunting, war, discovering the unknown, etc) then they may have died a lot more (then again, child birth was very risky).
            4. Isn’t this a thought experiment, where a more likely historical ratio is way lower than 100 women having sex with one man?

            It doesn’t seem like it’s mathematically possible for the mean lifetime number of heterosexual sex partners to be different by any more than the difference in the total number of “sexual marketplace” participants.

            1. Why does this matter? If 1 guy has reproductive sex with 100 women and 99 guys have sex with 0 women, the average guy has 1 sex partner. The median is 0 though and the one outlier gets to spread his genes exclusively.

            2. Prostitutes.

          • random832 says:

            1. Why do you assume that they are having sex? There is no law that everyone gets to have sex.

            If they are not having sex (or if they are not having enough sex to qualify as promiscuous) they are bringing down the average. It’s entirely reasonable to say that “average lifetime number of partners” is not actually the basis for saying men are more promiscuous than women, but it’s then not clear what the basis is other than intuition, and calling that an “observation” is a stretch.

            3. Why do you assume a 1:1 ratio of men to women?

            Is there not? We’re talking about what’s actually true of the societies being discussed, not what can be assumed.

            4. Isn’t this a thought experiment, where a more likely historical ratio is way lower than 100 women having sex with one man?

            Regardless of what the actual ratio is, I’m still not sure how the ratio between the averages can be anything other than the population gender ratio. The distributions could look very different, but it’s not clear what it means for one distribution to mean its population is “more promiscuous”.

            1. Why does this matter?

            I was responding to “All this theorizing is intended to explain the observation that men are more likely to be promiscuous than women across a wide range of cultures.” – my point is that it’s not clear A) that it is true or B) what it means.

            1. Why does this matter? If 1 guy has reproductive sex with 100 women and 99 guys have sex with 0 women, the average guy has 1 sex partner. The median is 0 though and the one outlier gets to spread his genes exclusively.

            The median being 0 certainly can’t be the basis for saying “men are more promiscuous”. Assigning characterizing adjectives based explicitly on the presence of outliers feels wrong to me.

            2. Prostitutes.

            Why does this count for men but not for women?

          • J Mann says:

            @Random832

            Stillnotking has a source (Schmitt) for the statement that it’s been observed that men are more promiscuous than women across a wide range of cultures, and further argues that there’s no significant source contradicting that.

            To the extent that you’re arguing that it’s mathematically impossible for Schmitt to be correct, I disagree.

            To the extent that you’re arguing that Schmitt is wrong, I think you need some more sources explaining why.

          • Brad says:

            The linked paper’s abstract says “In this article, findings from a cross-cultural survey … demonstrates that sex differences in the desire for sexual variety are culturally universal …”

          • J Mann says:

            A little orthagonally, here is Schmitt writing for a popular audience – I thought it was interesting in how he sees the field.

            https://www.psychologytoday.com/blog/sexual-personalities/201202/men-women-and-interplanetary-promiscuity

          • Aapje says:

            @random832

            2. Prostitutes.

            Why does this count for men but not for women?

            We know that male prostitutes with female clientele are very uncommon today and they seem to have been uncommon in the past as well (even in ancient Rome). In Afghanistan, the patriarchal protection of women is so strong that men rape boys (bacha bazi), which seems to be a case of situational homosexuality.

            I think that it is fair to say that an imbalance in sexual desire can be solved in part by
            – one sex paying members of the other sex for intercourse, so the latter do it even when they ‘are not feeling it.’
            – men having sex with men even if they are not really homosexual

            I think that people who are paid to be promiscuous are not naturally so. So a prostitute is promiscuous in behavior, but not necessarily in desire (and her desire is almost certainly not at the level of her behavioral promiscuity). I think that we need to distinguish between promiscuous desires and promiscuous behavior.

            When looking at reproduction, I also think that it is fair to assume that female prostitutes will produce offspring for a man who has sex with her far less often than other sex partners (for various reasons).

            So it seems to me that a society where 1 man has sex with 99 women and 1 woman has sex with 99 men, where both genders have an average of 1 partner, the harem guy will have a lot of offspring, while the 99 other men will have far less success.

          • random832 says:

            @J Mann

            To the extent that you’re arguing that it’s mathematically impossible for Schmitt to be correct, I disagree.

            To the extent that I’m arguing anything like that at all, it’s that one particular rigorous definition, the most obvious one, is mathematically impossible, and no other definition other than the non-rigorous “it intuitively feels like men are more promiscuous” has been presented (At least it has not been presented here in the comments. I feel like it’s important enough to the discussion that if there is one it should have been repeated here rather than requiring one to read through a paper).

            The problem with this “observation” is that there’s no clear statement of what has supposedly been observed.

            @Aapje

            We know that male prostitutes with female clientele are very uncommon today and they seem to have been uncommon in the past as well (even in ancient Rome).

            I didn’t think it was necessary to put such a fine point on it, but I meant why don’t female prostitutes count as promiscuous women, if male clients of prostitutes count as promiscuous men? The sex is presumably understood by all to be non-reproductive, so on those grounds it shouldn’t count for either.

          • Aapje says:

            @random832

            One can argue that prostitutes are a response to the evolutionary development that made men more promiscuous. Men have that innate desire, much more than women, who need to be paid before they play that game. One can assume that nature didn’t make a special rule that drives men to sex only with women who don’t use birth control, because birth control is a fairly recent development. So that these men are having sex that is poor at reproducing can be said to be humanity ‘hacking’ human desire to avoid the outcomes that those desires are meant to result in.

            As the prostitutes apparently don’t have those desires, but make a far more rational decision, it doesn’t count for them, since they are not ‘hacking’ their own desires.

          • J Mann says:

            @random832 – thanks for sticking with this conversation. It’s interesting, and I feel like I’m learning something.

            I’m not an expert, so take this with a grain of salt, but:

            1) IMHO, you’re right that it’s difficult, probably impossible, in a heterosexual environment for the average number of sexual partners to be off by more than differences in the sexually active populations of men and women.

            2) Of course, you could have differences in attitudes, and you could have differences in the average number of sexual participants is uneven. (So if a given community has 10 sexually active men and 100 sexually active women, it’s easy. Alternately, the community has 10 men who are engaged in relationships with several women each, and 90 men who alternate between short monogamous relationships and long periods of singledom, compared to 100 women who are single much less often, but only have sex with 1 man at a time).

            3) If I’m reading Schmitt correctly, he says that he gets robust and cross-cultural results that men have more promiscuous attitudes than women – that they are willing to engage in short term relationships with less hesitation, etc.

            4) Assuming 3 is correct, the next question is to what extent that difference is cultural/environmental, and to what extent it’s biological. The biological hypothesis is that human behavior evolved during a period where selective pressure rewarded men for having multiple mates more than it rewarded women.

            I think the biological hypothesis still unproven (generally, I think the jury is out whether humans evolved in circumstances more like chimps or more like bonobos, or for a specific example, when males are likely to commit infanticide against rival’s children, there is an incentive for females to mate widely enough that males can’t identify which infants to kill).

            But it’s not mathematically impossible for humans to have evolved in circumstances more like chimps or gorillas than bonobos. Your math is much better than Fine’s or Myer’s – it eliminates a misunderstanding about modern promiscuity findings – but it still doesn’t foreclose the underlying question.

            (To be fair, you might not have been saying it did. If this is what you’ve been saying all along, then I apologize for taking so long to get it!)

  46. Wander says:

    While Sarah Jones argued that the “alt-left” doesn’t exist, as the phrase groups together too diverse a political crowd to be unified by a single label

    There’s something really funny about this.

    • wintermute92 says:

      I will accept her argument as soon as major new sites stop grouping an-caps and fascists together under “alt-right”. (So, probably never.)

  47. HedonicRegression says:

    It being Spring Break, shouldn’t you have called this “urls gone wild”?

  48. nolemonnomelon says:

    I don’t think that NYT headline is as misleading as it seems. As I recall, in previous years foreign applicants were increasing more consistently across the board, so that 40% might be a significant negative change.

    • suntzuanime says:

      That sounds like a Gettier case.

    • JayT says:

      The main problem with the NYT headline and article is that it doesn’t give any useful information at all, and uses one statistic to try and make Trump look bad.

      Telling us that 40% of schools have fewer foreign applicants gives basically no information unless you talk about how many applicants there are overall. We don’t know if those schools are losing applicants because they have decided not to come to America or if they just chose other schools. We don’t know how big of a drop in applicants those schools had. Did they get one less student? 10? 100? Without those numbers the 40% statistic is meaningless. Reading this article gives no information about whether or not there is actually a significant change of any kind.

      • Conrad Honcho says:

        Also you could have just as easily titled the article “In anticipation of roaring Trump success 35% of US colleges see increased foreign student enrollment.”

  49. meh says:

    Re below link (). I can’t decide if that is dishonest without knowing prior years data. At best I can say it is incomplete, or they are drawing a conclusion based on data they don’t present. Calling it dishonest assumes the baseline is no change. I couldn’t find great numbers online, but if the trend had been year over year increase then flat to slightly down could be considered a dip.

    Marginal Revolution: Last year, 35% of colleges saw international student numbers go up, 26% saw no change, and 39% saw them go down. New York Times publishes this with the headline “Amid Trump Effect Fear, 40% of Colleges See Dip in Foreign Applicants”. This may be even more dishonest than that other NYT headline.

    • Catlick says:

      The study they cite states that the trend has been year-over-year increases in foreign student applications. So this would indeed be evidence of a cool-down. But there are a hundred ways the Times could’ve gotten that point across, without being dishonest.

      Not to be beaten, NPR writes this article, titled “Travel Ban’s ‘Chilling Effect’ Could Cost Universities Hundreds of Millions”. From the piece: “Four in 10 of the colleges that responded say they had seen a drop in applications from around the globe”.

  50. Whatever Happened To Anonymous says:

    I’m surprised Robinson is so young, considering that part of his criticism of Vox was that they were a bunch of very green dudes acting like they were experts on a variety of subjects.

    • Scott Alexander says:

      Yeah, it was pretty funny seeing him make fun of Yglesias for being an undergrad studying philosophy at Harvard, when he was a postgrad studying sociology at Yale.

      The reason I didn’t go ahead and make the joke/insult/observation is that I think Robinson took some time off education to serve as a lawyer for poor people, and I think part of his position is that things like that give you the real-life experience that makes you more knowledgeable than Yglesias.

  51. Matt M says:

    Charity, you say? I thought the whole point of the glorious European welfare state was that everyone was guaranteed food such that they would never have to depend on the worthless and unreliable scourge that is private charity?

    • Scott Alexander says:

      I don’t think Britain is usually considered part of the Glorious European Welfare State.

    • Whats your argument? That a continuing role for private charity means public welfare is completely ineffective?…not that private charity can fill in gaps and provide specialised services?

      Foodbanks are getting big in the UK because among other reasons the right wing government instituted rules that nean claimants keep getting their welfare cut off for minor infringements.

      • Matt M says:

        That the extent and/or effectiveness of the European welfare state is wildly (and intentionally) exaggerated, among Americans at least.

  52. J Mann says:

    P.Z Myers is surprisingly unreflective for someone in science. In his review, he approvingly quotes Fine and adds the bit at the end.

    Indeed, a promiscuous man would need to have sex with more than 130 women just to have 90 percent odds of outdoing the one baby a monogamous man might expect to father in a year.

    Suddenly, my preferred reproductive strategy, monogamy and paternal investment in offspring, seems to be the best evolutionary strategy. It’s much less exhausting, too, and has given me time to do other things in my life.

    It’s just weirdly innumerate. I can’t find Fine’s math, but she seems to be assuming that a promiscuous man is going to have sex with each of his additional mates only once per year, and that this strategy is only useful if it has a 90% chance of producing more than one (I don’t know if that means 1.01 expected value or 2!) offspring per year.

    Even without knowing her specific math, i can’t see why you would use confidence intervals instead of expected values, why you would require a 90% confidence interval, or why you would assume that the expected reproduction rate from monogamy is one child per year(!) But Myers just chortles that her math demonstrates that monogamy “seems to be the best evolutionary strategy.”

    Then when one of his commenters points out that the expected value of promiscuity is higher than monogamy for basic assumptions, Myers doesn’t go back and correct his review or consider how he made a mistake – he accuses the commenter of lacking “humanity” for doing the math, and eventually falls back on a bunch of evasions. It’s very sad.

    http://freethoughtblogs.com/pharyngula/2017/03/08/cordelia-fine-is-doing-the-math/

  53. Jordan D. says:

    The anti-Buzzfeed brief title is an excellent joke. It’s always a risk to try to employ humor in a legal filing, but I think the reward is worth the risk if you’ve got good judgment and taste.

    (I guess the trick is that people who don’t have those things are ill-equipped to ascertain that)

    The case itself is very interesting, too- it’s about the unverified “Trump Dossier” that Buzzfeed published a while back. While Trump himself is a public figure and, despite all of his threats, unlikely to succeed at any libel case against them, the dossier also contains allegations against less-public figures. This suit is by three of those people. Whether they’ve got a case is a complex question, but they could very well win.

    But this particular motion is probably less good.

  54. bysstah rhymes says:

    The article about the exportability of the nordic model.

    Basically, his question is, if you dropped Kela or Vero in the U.S., would they be able to do the same stuff. My answer is no, because of institutions. Americans resist giving money to the government in a way Scandinavians don’t. Nobody flies planes into Vero’s offices. They do for the IRS. Vero doesn’t need SWAT team support. The IRS sometimes does. Scandinavians don’t think of Kela as “their” retirement, i.e. they don’t have a mental image of a personal retirement account as the norm, Americans do.

    As a swede, I find this quite amusing, as he is basically saying im just paying my taxes out of good will. This is not why swedes pay their taxes. Just like in the U.S. if u don’t pay your taxes, there will be consequences. The Swedish IRS can, through a different governing body, seize your assets. If it gets to this, youll also be put in a registry that’s used by banks and such, and it will be much harder for you to get a loan, credits, a contract for an apartment, ect.

    This is the case in the US as well I would assume? And if not, why not enact laws that make this possible?

    About the plane flying in to the IRS and them having to use SWAT to seize assets.
    In the town I live in, in 2013 a bomb was set of outside the local IRS office, during night, so nobody was injured, but swedes are ill tempered as well. About having to use swat, if u don’t pay your taxes in Sweden, the cops will eventually come and assist in the seizing of the possible assets you have in your possession. So I don’t see the difference here other than americans have more guns, and therefor need heavier police equipment when seizing assets.

    But yes, its most probably a cultural thing and the reason americans doesn’t have a bigger welfare state is because americans doesn’t want to have a bigger welfare state. Plus their political system is different as well, which might play a part in it.
    And of course. America is really big as well.

  55. Steve Sailer says:

    “Stuart Buck on how some of the hype about rising white-working-class mortality comes from graph that exaggerates its point by using two different y-axes.”

    That’s a trivial flaw out of the dozens of graphs in Deaton and Case’s new paper.

    The really interesting question is why the media didn’t notice The White Death until less than 18 months ago. If Deaton hadn’t won the (quasi) Nobel in Economics in October 2015 and then released a paper on The White Death in November 2015, would the media have noticed it yet?

    Here’s my estimate of how big The White Death is: about 40,000 incremental deaths per year. That’s gigantic. “I had not thought death had undone so many.”

    You can read my methodology here:

    http://www.unz.com/isteve/the-white-death-proven/

    If you come up with a different estimate for the magnitude of excess deaths, please let me know.

  56. taradinoc says:

    Prison brutality: guards throw mentally ill inmate in scalding hot shower; leave him there until he dies. No punishment given.

    That’s one perspective. If you read the official report, there are several points that aren’t mentioned in that Reason article:

    * The water temperature was measured several times, with analog and digital thermometers, at various points between the source and the shower, at a normal 120ish degrees. Only one measurement was at a dangerously high level, and that measurement wasn’t taken in the shower. (None of the measurements were taken on the same day as the inmate’s death.)

    * Statements from the maintenance staff suggest the water temperature couldn’t have reached the temperature alleged. One boiler was offline; the remaining one was set to 120. Accessing the boiler to change the setting meant requesting a key, and access was logged.

    * The medical examiner stated there were no burns on the body consistent with the alleged scalding, and the skin “peeling off” was due to soaking in a pool of water after death.

    * The inmate’s medical conditions (schizophrenia plus an undiagnosed heart condition) and medications predisposed him to heart problems brought on by stress and even slightly elevated temperatures.

    * Video evidence contradicts some claims made by the main person who alleges the death was intentional: he physically could not have seen events he claims to have seen.

    * Perhaps most damning: the shower in question didn’t have a normal shower head that covered a large area. It had a hose that produced a sad-looking trickle of water, horizontally, across the far end of a large stall. The report includes a photo, and it also states that the inmate was seen standing out of the water’s path.

    • AnonYEmous says:

      As a personal resolution, this is a reminder that libertarians, too, have an ideological worldview and are willing to view events a certain way if they advance that worldview. If an event seems to confirm libertarian viewpoints, then it should be re-examined for clarity, to make sure it isn’t just red meat thrown to the comments section without regards to its truth.

    • J Mann says:

      I’m ideologically inclined to agree with Reason, and know and like some of their writers personally, but most of their writers are fairly one-sided. If they tell you that it’s outrageous that cop so and so wasn’t punished for a shooting or that teen so and so was punished for banging a 14 year old or that honors student so and so was punished for a “science experiment” where she blew up a bottle bomb outside of school while students were entering for the day’s classes, it’s extremely unlikely that they researched and disclosed the other side of the story.

    • Matt M says:

      Thanks for this. Despite being an AnCap who is highly skeptical of the police, when I first read the headlines here, my immediate reaction was, “This CAN’T be the whole story…. there has to be more to this.” Nice to see that’s true.